format .pdf, 3.0 MB

102
RECREA Ţ II MATEMATICE REVIST Ă DE MATEMATIC Ă PENTRU ELEVI Ş I PROFESORI Universitatea “Al. I. Cuza” din Iaşi (1860 – 2010) Asocia ţ ia “Recrea ţ ii Matematice” IA Ş I - 2010 Iulie Decembrie 2010 Anul XII, Nr. 2 1 = π i e

Transcript of format .pdf, 3.0 MB

RECREAŢII MATEMATICE

RE V IS TĂ DE M AT E M AT I CĂ P E NT R U E L E V I Ş I P R O F E S O R I

Universitatea “Al. I. Cuza” din Iaşi

(1860 – 2010)

A s o c i a ţ i a “ R e c r e a ţ i i M a t e m a t i c e ” I AŞ I - 2 0 1 0

Iulie – Decembrie 2010Anul XII, Nr. 2

1−=πie

Semnificaţia formulei de pe copertă: Într-o formă concisă, formula 1−=πie leagă cele patru ramuri fundamentale

ale matematicii: ARITMETICA reprezentată de 1 GEOMETRIA reprezentată de ALGEBRA reprezentată de i ANALIZA MATEMATICĂ reprezentată de e

Redacţia revistei :

Petru ASAFTEI, Dumitru BĂTINEŢU-GIURGIU (Bucureşti), Temistocle BÎRSAN, Dan BRÂNZEI, Alexandru CĂRĂUŞU, Constantin CHIRILĂ, Eugenia COHAL, Adrian CORDUNEANU, Mihai CRĂCIUN (Paşcani), Paraschiva GALIA, Paul GEORGESCU, Mihai HAIVAS, Gheorghe IUREA, Lucian-Georges LĂDUNCĂ, Mircea LUPAN, Gabriel MÎRŞANU, Alexandru NEGRESCU (student, Iaşi), Gabriel POPA, Dan POPESCU (Suceava), Florin POPOVICI (Braşov), Maria RACU, Neculai ROMAN (Mirceşti), Ioan SĂCĂLEANU (Hârlău), Ioan ŞERDEAN (Orăştie), Dan TIBA (Bucureşti), Marian TETIVA (Bârlad), Lucian TUŢESCU (Craiova), Adrian ZANOSCHI, Titu ZVONARU (Comăneşti)

COPYRIGHT © 2010, ASOCIAŢIA “RECREAŢII MATEMATICE” Toate drepturile aparţin Asociaţiei “Recreaţii Matematice”. Reproducerea integrală sau

parţială a textului sau a ilustraţiilor din această revistă este posibilă numai cu acordul prealabil scris al acesteia. Se consideră că autorii materialelor trimise redacţiei revistei sunt, în mod implicit, de acord cu publicarea lor, îşi asumă responsabilitatea conţinutului lor şi cedează Asociaţiei “Recreaţii Matematice” dreptul de proprietate intelectuală asupra acestora.

TIPĂRITĂ LA BLUE SIM & Co IAŞI Bd. Carol I, nr. 3-5 Tel. 0332 111021, 0721 571705 E-mail: [email protected]

ISSN 1582 - 1765

Anul XII, Nr. 2 Iulie – Decembrie 2010

RECREAŢ I I MATEMATICE

RE V IS TĂ DE M AT E M AT I CĂ P E NT R U E L E V I Ş I P R O F E S O R I

1−=πie

Revistă cu apariţie semestrială

EDITURA “RECREAŢII MATEMATICE”

IAŞI - 2010

.

Universitatea ieseana – la aniversare

Anul curent este considerat ca fiind cel de-al 150-lea an de existenta aUniversitatii ”Al.I. Cuza” si este sarbatorit prin mai multe actiuni academice,dedicate evenimentului.

Desigur, chiar cu acceptarea ideii ca anul 1860 este anul de nastere al venerabileiinstitutii, universitatea ieseanaeste tot cea mai veche din tara,iar progresul pe care l-a realizateste confirmat si de includerea eiın grupul ”Coimbra”. Poate chiarmai semnificativ decat aceastaapartenenta este faptul ca maimulti tineri profesori ce si-auınceput pe bancile ei uceniciasunt acum titulari de catedreuniversitare la unele universitatide mare faima ın lume: Paris,Oxford, Brown (Providence), Berkeley (California), Stanford si altele. Situatia ımieste cunoscuta numai pentru matematicieni si banuiesc ca si alte domenii pot daexemple similare.

Din trecut putem indica nume sonore, care au dus reputatia universitatii iesenedincolo de hotarele Romaniei. Istoricul A.D. Xenopol este primul roman care a fostales membru al Academiei Franceze, dupa Dimitrie Cantemir (acesta la AcademiaPrusiana de stiinte). Horia Hulubei, pe cand era profesor la Iasi, a devenit membrucorespondent al Academiei Franceze. Alt fizician de la Iasi, Stefan Procopiu, a facutun nume bun pentru universitatea din Iasi, precum si Gheorghe Bratianu, ale caruilucrari sunt frecvent utilizate ın cercetarea istorica mondiala contemporana.

Cu toate acestea, Universitatea din Iasi, ınfiintata prin decret al domnitoruluiAlexandru Ioan Cuza si purtand semnatura primului sau ministru,

Mihail Kogalniceanu, fost pro-fesor al Academiei Mihailene, nueste prima institutie din tarileromane care sa-si poata apropriatitlul universitar. Ea este o a douatreapta ın evolutia AcademieiMihailene, ıntemeiata la 1835,sub domnia lui MihalacheSturza, cu aportul substantial alparintelui educatiei moderne dinMoldova –Gheorghe Asachi. Indiscursul pronuntat la deschidereaAcademiei Mihailene, acesta

spunea ca ınfiintarea acestei institutii are menirea de a ridica la un rang superiorscoala din Principatul Moldovei ”ın acord cu nivelul luminatei Europe”.

93

La Academia Mihaileana, a carei existenta a fost marcata de evenimentelepolitico-sociale ale timpului, a rostit Mihail Kogalniceanu discursul sau istoric,ın deschiderea primului curs tinut vreodata la noi de Istorie Nationala. Tot aici,Dimitrie Asachi, fiul lui Gh. Asachi, a tiparit primul curs de topografie ın limbaromana. El a fost si unul dintre primii romani care a facut cercetare matematicaoriginala.

In cartea Invatamantul romanesc ın date de Mihai Bordeianu, fost director alBibliotecii Universitare ”Mihai Eminescu” din Iasi, si Petru Vladcovschi, cercetator,se poate vedea pe baza de documente ca Academia Mihaileana a fost o institutiede rang universitar, comparabila cu multe alte institutii similare din Europa. Existaın aceasta lucrare o lista detaliata a diferitelor discipline de ınvatamant, cu indicatiiasupra continutului cursurilor, care arata fara ındoiala faptul ca aceasta Academieservea aceleasi teluri ca si universitatile europene din alte tari, avand multi profesoriformati ın marile centre universitare apusene, ei fiind atat din Moldova cat si dinMuntenia sau Transilvania.

Infiintarea universitatii iesene, la un an dupa Unirea Principatelor, a fost o simplaformalitate regizata de Mihail Kogalniceanu. De fapt, drapelele mai multor facultatiexistente ın Muzeuluniversitatii sunt da-tate cu multi aniınainte de 1860. Maimult, studentii ceexistau la AcademiaMihaileana, cat siun mare numar din-tre profesorii sai,au devenit automatstudenti sau profe-sori ai acesteia. Asaıncat, la un an dupaınfiintare, Universi-tatea a putut acorda prima diploma din istoria sa trunchiata.

Primii profesori ai universitatii, ın cea mai mare parte fara titlu de doctor, i-aupropus lui Simion Barnutiu, cu doctorat la Viena, sa devina primul rector. Acestaa declinat politicos oferta, invocand faptul ca este cetatean al Imperiului Austriac.Simion Barnutiu provenea de la Academia Mihaileana, unde predase cursuri de niveleuropean.

Trebuie mentionat faptul ca ın anul 1935, a fost sarbatorit de catre universi-tarii ieseni Centenarul ”Universitatii Mihailene”. Printre profesorii din acea vremese numarau personalitati ca: Alexandru Myller, Grigore Moisil, Octav Mayer, StefanProcopiu, Petre Andrei, Iorgu Iordan, Petru Caraman, Ioan Placinteanu, Mihai Ralea,I. Borcea, a caror opera stiintifica se bucura de recunoastere internationala.

Peste 25 de ani, ın anul 1960, s-a sarbatorit din nou ”centenarul” universitatiiiesene, fireste, prin ignorarea sau minimalizarea rolului Academiei Mihailene, prinnerecunoasterea acesteia ca prima universitate a tarii cu limba de predare romana.

94

Acest prejudiciu adus, ın ultima instanta, primei universitati romanesti, nu poatefi acoperit de manifestarea oarecum pompoasa, presarata cu alocutiuni pronuntateıntr-un veritabil ”limbaj de lemn” de multi vorbitori. Rectorul Universitatii ”V.I.Lenin” din Chisinau ne-a salutat la sfarsitul mesajului sau rostit ın limba rusa cu”Traiasca Univerzitetul din Iasi”, aceasta ın ”limba romana”. Ca participant la des-chiderea acestor solemnitati, mi-au procurat adevarate satisfactii mesajele rostite,ıntr-o limba romana de ınalta calitate, de Alf Lombard (Suedia) si Carlo Tagliavinide la Universitatea din Padova.

Argumentul invocat de cei ce au hotarat repetitia sarbatoririi Centenarului Uni-versitatii din Iasi se reduce, ın fond, la faptul ca Mihalache Sturza a emis, ın 1841, undecret de desfiintarea Academiei, proba-bil ıngrijorat de ac-tivitatea studentilorcare erau sub in-fluenta ideilor re-volutionarilor de la1848, ca peste totın Europa. Facul-tatile acesteia si-au pastrat totusiexistenta, devenindapoi parti compo-nente ale Universitatii iesene. Intreruperi si alte dificultati au aparut si la alte institutiisimilare din alte tari. Cu toate acestea, multe institutii de ınvatamant superior si-aupastrat identitatea si continuitatea. De ce s-a petrecut altfel la noi, contrar pareriilui A.D. Xenopol, figura de prim rang ın istoria universitatii iesene? Cred ca valul”sovromculturii”, dominanta ın acea perioada a contribuit la aceasta situatie oarecumanormala, contrar traditiilor academice si intereselor noastre culturale.

Luand ın consideratie Academia Mihaileana si rolul ei de a promova ınvata-mantul superior din Moldova ın limba romana, trebuie sa concludem ca ın anul 2010sarbatorim cea de a 175-a aniversare a primei universitati romanesti.

Cu ocazia Centenarului Seminarului Matematic ”Al. Myller”, organizat la Iasi ınzilele de 21-26 iunie 2010, s-au putut constata progresele realizate ın acest sector alcunoasterii. Fireste, acest lucru a fost posibil datorita faptului ca ın anul 1835, apoiın anul 1860, oameni luminati au pus bazele necesare construirii unui viitor fecundeducatiei superioare a tinerilor.

Constantin CORDUNEANUUniversity of Texas, Arlington

95

CentenarulSocietatii de Stiinte Matematice din Romania

In vara anului 1909, la via lui Ion Ionescu de la Valea Calugareasca, a avut loc osedinta a redactorilor Gazetei Matematice ın care s-a hotarat ınfiintarea unei societati”pentru ındeplinirea unor forme legale”, adica a unei societati care sa dea o acoperirejuridica ıntregii activitati desfasurate ın jurul revistei mentionate mai sus.

O comisie formata din Vasile Cristescu, Andrei Ioachimescu, Ion Ionescu, TraianLalescu, I.D. Teodor si Gheorghe Titeica s-a ocupat de redactarea documentelor nece-sare recunoasterii oficiale a noii societati. Proiectul depus si apoi votat ın camereleparlamentului devine lege. In cuvantul luat ın Senat, de sprijin pentru noua soci-etate, Spiru Haret spune: ”A contribuit mai mult decat orice alta institutiune pentrudezvoltarea si ıntarirea ınvatamantului matematic”. Regele Carol I promulga legeaprin Decretul 3798 din 18 decembrie 1910. Aceasta este data ınfiintarii Societatii”Gazeta Matematica”.

Societatea este administrata de o Delegatie compusa din doi membri alesi pentrudoi ani, fara dreptul de a fi realesi imediat, si un casier ales pentru cinci ani, care poatefi reales. Prin delegati erau coordonate diversele activitati: delegat pentru aritmetica,delegat pentru examinarea notelor si articolelor etc. De mentionat, Gh. Titeica a fost,pana la sfarsitul vietii sale, delegat pentru ıntocmirea rapoartelor. In 1924, ca urmarea Legii persoanelor juridice, se aduc modificari statutului, iar societatea trece subtutela Ministerului Instructiunii, Cultelor si Artelor.

In anul 1935 este inaugurata Casa ”Gazetei Matematice”, rezultat al unor contri-butii particulare felurite (teren de amplasare, executia lucrarii, donatii etc.) si spri-jinului statului. (Acest local a fost confiscat abuziv ın 1949 de puterea comunista sinici astazi lucrurile nu s-au clarificat.) Tot ın 1935 a fost organizat un mare jubileu la40 de ani de la aparitia Gazetei Matematice. Semicentenarul acesteia a fost sarbatoritın 1945, ın conditiile grele de dupa razboi.

Cu instalarea puterii comuniste, s-a instaurat ın tara o atmosfera apasatoare denesiguranta si abuzuri; o serie de membri marcanti ai Societatii ”Gazeta Matematica”sunt arestati si sfarsesc ın ınchisori, iar Societatea ınsasi dispare. In locul ei esteınfiintata Societatea de Stiinte Matematice si Fizice din R.P.R., iar revista GazetaMatematica devine Gazeta Matematica si Fizica. Au urmat si alte schimbari ınstructura organizatorica a noii societati si a publicatiei (publicatiilor) tutelate.

In 1964, prin separarea sectiei de matematica de cea de fizica, ia fiinta Societateade Stiinte Matematice, care publica Gazeta Matematica ın doua serii: A si B.Vor mai urma si alte schimbari si ”rationalizari” ...

Din anul 1980, a fost reluat Concursul Gazetei Matematice sub numele de Concur-sul anual al rezolvitorilor. S.S.M.R. este partener, alaturi de Ministerul Invataman-tului si Cercetarii ın organizarea olimpiadelor pentru elevi, nationale si internationale.

In 1995, o ampla manifestare a fost dedicata Centenarului Gazetei Matematice.Statul Roman a conferit, ın anul 2005, Ordinul ”Meritul Cultural” ın grad de

Ofiter revistei Gazeta Matematica.

Redactia

96

Strofoida – cateva proprietati elementare

Temistocle BIRSAN 1

Abstract. In this note, some elementary properties of the strophoid are given. The wholecontent is accessible to highschool students and section 1 can be understood by junior highschoolstudents as well.

Keywords: strophoid, ellipse, hyperbola, parabola.

MSC 2000: 53A04.

Scopul propus este de a prezenta, ın limitele cunostintelor de geometrie dobanditeın cursul scolii, ıncepand cu cele din clasa a VI-a, cateva proprietati ale strofoidei.In mod necesar, selectia acestora a fost severa, dar speram ca faptele retinute suntsuficiente pentru introducerea cititorului ın universul fascinant al strofoidei.

Nume ilustre sunt legate de studiul acestei curbe: E. Torricelli (1645), G.P. Rober-val (1645), I. Barrow (1970), Jean Bernoulli, A.L.J. Quetelet (1910) s.a. Denumireade strofoida i-a fost data de E. Montucci ın 1846 (στρoφoς (strophos)=cordon/sfoararasucit(a), ειδoς (eidos)=aspect). Alte denumiri: pteroides torricellana, logociclica,focala lui Quetelet.

1. Fie date un punct A si o dreapta ∆ care nu-l contine. Fie O proiectia

P

S

S

AO

D

d

Fig. 1

punctului A pe ∆. O dreapta variabila d ce trece prin A in-tersecteaza ∆ ın P. Vom numi strofoida locul geometric alpunctelor S si S′ situate pe d si verificand conditia PS = PS′ =PO. Punctul A se va numi varful strofoidei, O-centrul sau, iar∆-axa sa (fig. 1). Cand d variaza astfel ıncat P parcurge axa∆ de sus ın jos, punctul S descrie ramura de strofoida indi-cata de sagetile pline, iar S′ descrie ramura indicata de sagetileıntrerupte; S se ındreapta catre A, iar S′ porneste din A (vomstabili riguros forma curbei ın sectiunea 2).

Evident, conditia PS = PS′ = PO implica faptul ca O,S, S′

se afla pe cercul cu centrul ın P si tangent la OA. Ca urmare, are locPropozitia 1. Strofoida este locul geometric al intersectiilor cercurilor tangente

A

MS

O

M

S

D

Fig. 2

ın O la OA cu diametrii lor care trec prin varful A.Vom utiliza mai jos notatiile CA si CO pentru cercurile

C(A,AO) si C(O,OA). O noua caracterizare a strofoidei vadecurge imediat din lema urmatoare (demonstratia careiautilizeaza relatia ıntre unghiurile cu laturile perpendiculare):

Lema. Fie M intersectia perpendicularelor ın A pe ASsi ın O pe OS. Atunci, S satisface conditia PS = PO daca sinumai daca M ∈ CA. Afirmatie similara relativa la puncteleS′ si M ′ (fig. 2).

1Prof. dr., Catedra de matematica, Univ. Tehnica ”Gh. Asachi”, Iasi

97

Propozitia 2. Strofoida este locul punctelor S cu proprietatea ca perpendiculareleın A si O pe dreptele AS si respectiv OS se intersecteaza pe CA.

Propozitia 3. Strofoida (de varf A si centru O) este locul ortocentrelor H aletriunghiurilor MAO, unde M este un punct mobil pe CO.

Demonstratie. Daca M ∈ CO si H este ortocentrul triunghiului MAO,

A

M

L

H

P

D

O

Fig. 3

atunci OL este axa de simetrie a triunghiului isoscel OAM

si avem ÕAHL ≡ ÖMHL. Rezulta ca ÕPHO ≡ ÕPOH sauPH ≡ PO, adica H este pe strofoida.

Invers, daca H este un punct pe strofoida (pe arcul su-perior al buclei ın fig. 3), notam cu M intersectia paraleleiprin H la axa ∆ a strofoidei. Se arata, pe cale inversa, ca

PH ≡ HO implica ÕAHL ≡ ÖMHL si apoi ca AHO ≡MHO (cazul LLU !) si se obtine ca OL ⊥ AM. Cum siMH ⊥ AO (prin constructia punctului M), rezulta ca Heste ortocentru ın MAO, ceea ce ıncheie demonstratia.

Propozitia 4. Dreptele ∆ si ∆′ trec prin punctele O si respectiv A si sunt per-pendiculare pe AO. Fie M ∈ ∆ mobil si N = pr∆′M . Locul geometric al simetricelorpunctelor N fata de dreapta AM este strofoida de varf A si centru O.

Demonstratie. Fie P = AS∩OM (fig. 4). Daca S se obtine dinM ca ın enunt,

A O

PS

N M

D D

Fig. 4

atunci triunghiurile ANS si PAM sunt isoscele si avem AS =AN = MO si AP = MP. Deci PS = AS−AP = MO−MP =PO, adica PS = PO si punctul P apartine strofoidei.

Invers, daca S este pe strofoida, fie M punctul obtinut in-tersectand ∆ cu paralela prin A la OS. Din PS = PO siPSO ∼ PAM deducem AP = MP si apoi AS = OM.Daca N = pr∆′M , urmeaza ca triunghiul ANS este isoscel.Din AM∥OS si ∆∥∆′ rezulta usor ca (AM este bisectoarea

unghiului ÕNAS, deci si mediatoarea segmentului [NS], adica Seste simetricul lui N fata de AM si demonstratia este completa.

Cititorul va putea rezolva, cu mijloacele la fel de elementare, problemele:

Problema 1. Fie date punctele A si O si fie ∆,∆′ perpendicularele ın O sirespectiv A pe AO. Se considera un punct M ∈ ∆′ mobil si fie S punctul ın caredreapta OM retaie cercul de centru A ce trece prin M. Aratati ca locul punctului Seste strofoida de varf A si centru O.

Problema 2. Fie B punctul diametral opus lui A ın cercul CO si M ∈ CO mobil.Aratati ca strofoida de varf A si centru O este locul punctelor S de intersectie aperpendicularei prin M pe AO cu paralela prin O la MB.

Problema 3. Aratati ca, daca M este un punct mobil pe CO, atunci locul centruluicercului ınscris ın triunghiul MAO este bucla strofoidei de varf O si centru A.

2. Unele proprietati ale strofoidei necesita cunostinte de geometrie mai avansate.Vom prezenta cateva dintre acestea, cu grija de a ramane ın limitele programelor.

98

Propozitie 5. Transformata strofoidei prin inversiune fata de cercul CA estecurba ınsasi.

Demonstratie. Sa aratam ca punctele S si S′ (fig. 1) sunt inverse ın raport cuCA. Intr-adevar, punctele A,S si S′ sunt coliniare si avem AS · AS′ = (AP + PS) ·(AP −PS′) = (AP +PO) · (AP −PO) = AP 2 −PO2 = AO2, deci AS ·AS′ = AO2.

In continuare, vom abandona abordarea sintetica a problemelor, preferand uti-lizarea metodei coordonatelor. Vom lua originea axelor de coordonate ın punctulO-centrul strofoidei (fig.1), dreapta AO cu sensul de parcurs de la A la O ca axaa x-lor si ∆ ca axa a y-lor. Considerand AO = a, avem A(−a, 0). Ca parametruluam ordonata t a punctului variabil P , adica P (0, t), t ∈ R. Fie S(x, y) pe dreapta

AP : y =t

a(x+a). Conditia PS = PO se scrie

Èx2 + (y − t)2 = |t|. Sistemul acestor

doua ecuatii se mai scrie: x2 + y2 − 2yt = 0 si t =ay

x+ a. Introducand t ın prima

ecuatie, obtinem, dupa cateva calcule simple, ecuatia carteziana a strofoidei:

(1) x(x2 + y2) = a(y2 − x2)

sau, ın forma explicita,

(2) y = ±x

ra+ x

a− x, x ∈ [−a, a).

Pentru ramura strofoidei cu ”+” avem: limx→a

y = +∞ (dreapta x = a este asimp-

tota), y′ =−x2 + ax+ a2

(a− x)√a2 − x2

cu x0 =a

2(1 −

√5) singura ei radacina ın [−a, a) si

tabelul de variatie

x −aa

2(1−

√5) 0 a

y′ -∞ – 0 + 1 + +∞y 0 ≈−0, 3a 0 +∞

Graficul strofoidei este compus din graficul acestei ramuri si al simetricului saufata de axa x-lor (fig. 1).

Tangentele ın origine la strofoida sunt bisectoarele axelor de coordonate.

Pentru calculul ariei buclei de strofoida avem (cu substitutiile

Éa+ x

a− x= t si

t = tg u) succesiv:

A1 = 2

Z 0

−a|x|r

a+ x

a− xdx = −8a2

Z 1

0

t2(t2 − 1)

(t2 + 1)3dt =

= 8a2Z π

4

0sin2 u cos 2u du =

2− π

2

a2.

Pentru aria domendiului cuprins ıntre strofoida si asimptota sa obtinem, ın urma unui

calcul similar, A2 =2 +

π

2

a2. Aria totala a domeniului marginit de strofoida si

asimptota sa este A=A1+A2=4a2, adica de patru ori aria patratului de latura AO.Intersectand strofoida data prin (1) cu dreapta y = tx dusa prin nodul ei, obtinem

99

P

AO x

y

Fig. 5

ecuatiile parametrice:

(3) x = a1− t2

1 + t2, y = a

t(1− t2)

1 + t2;

tot din (1) se obtine si ecuatia polara a strofoidei

(4) ρ = −acos 2θ

cos θ

(ρ2=x2+y2=ay2 − x2

x=aρ

sin2 θ − cos2 θ

cos θ=−aρ

cos 2θ

cos θ).

Proprietatile urmatoare indica legaturi ale strofoidei cu curbele de ordinul doifamiliare: parabola, elipsa si hiperbola.

Propozitia 6. Fie P o parabola de varf A si O intersectia axei cu directoarea sa.Locul proiectiilor lui O pe tangentele la P ıntr-un punct mobil al ei este strofoida devarf A si centru O.

Demonstratie. Alegem O ca origine, axa parabolei ca axa x-lor si directoareaca axa a y-lor. Atunci P are ecuatia y2 = −4a(x + a). Fie (α, β) ∈ P un punctmobil. Impunand conditiile din enunt obtinem ecuatiile: β2 = −4a(α+ a) – punctul

(α, β) ∈ P, βy = −2a(x + a + α + a) – ecuatia tangentei ın (α, β) si y =β

2ax

– ecuatia perpendicularei ın O pe tangenta. Eliminam α si β din sistemul acestor

ecuatii: β =2ay

xse introduce ın a doua ecuatie si se obtine α+a = −

y2

x+ x+ a

.

Inlocuind acestea ın prima ecuatie, vom gasi ca ecuatie a locului tocmai ecuatiacarteziana (1) a strofoidei.

Observatie. Date o curba Γ si un punct fix A, se numeste podara curbei Γ fatade polul A curba descrisa de proiectia lui A pe tangenta la Γ ıntr-un punct mobil alei. Proprietatea perecedenta se poate enunta astfel: podara parabolei fata de punctulde intersectie a directoarei cu axa sa este strofoida cu centrul ın acest punct si varfulın varful parabolei.

Propozitia 7. Curba inversa a hiperbolei echilatere de varfuri O si A fata decercul CO este strofoida de varf A si centru O.

Demonstratie. Cum A(−a, 0) (fig. 1), centrul hiperbolei este punctul−a

2, 0,

iar ecuatia ei estex+

a

2

2− y2 =

a2

4sau x2 − y2 + ax = 0.

Fie H(α, β) pe hiperbola, deci α2 − β2 + aα = 0 (*). Inversul acestui punct,fie el S(x, y), este coliniar cu O si H si satisface relatia OS · OH = a2 (cercul COavand raza OH = a). Asadar, avem y =

β

αx si (x2 + y2)(α2 + β2) = a4. Rezolvand

sistemul acestor doua ecuatii ın α si β, obtinem α =a2x

x2 + y2, β =

a2y

x2 + y2. Scriind ca

(α, β) verifica ecuatia hiperbolei echilatere, adica introducand ın (*) aceste expresii,

100

obtinem ecuatia locului punctului S:

a2x

x2 + y2

2

a2y

x2 + y2

+

a3x

x2 + y2= 0, de

unde x(x2 + y2) = a(y2 − x2) – strofoida de varf A si centru O.

Intr-un plan π consideram punctele A si O si dreapta ∆ ce trece prin O si este

A

F

F

O

B

q

P

.

.

.

Fig. 6

perpendiculara pe AO. Notam cu C∆ cilindrul de rotatie deaxa ∆ ce trece prin A.

Propozitia 8. Locul focarelor elipselor de sectiune acilindrului C∆ cu planele care trec prin A si sunt perpendic-ulare pe π este strofoida situata ın π si avand varful A sicentrul O.

Demonstratie. Notam cu θ unghiul pe care-l face unplan oarecare (ce ındeplineste cerintele propozitiei) cu cel alsectiunii circulare. De asemenea, notam AB axa mare a elip-sei de sectiune rezultate, F si F ′ focarele si P ∈ ∆ centrulei. Evident, PF = PF ′ si ramane sa aratam ca PO = PF. In triunghiul dreptunghic

OAP avem PO = a tg θ si AP =a

cos θ. Elipsa de sectiune avand semiaxele

a

cos θsi

a, rezulta ca PF 2 = a

cos θ

2− a2 = a2 tg2 θ. Ca urmare, PF = a tg θ = PO, ceea ce

ıncheie demonstratia.4. Aducand modificari elementelor ”de temelie” ale strofoidei: A (varful),

..

.A

S

O

S

d

G

.P

.Fig. 7

O (centrul) si ∆ (axa) ın privinta pozitiei sauformei lor, se vor obtine generalizari importante.

Fie date o curba Γ si doua puncte A si O.Curba strofoidala a curbei Γ relativ la puncteleA si O este locul geometric al punctelor S si S′

ale unei drepte variabile d ce trece prin A si careındeplinesc conditia PS = PS′ = PO, unde P esteun punct de intersectie (altul decat A) al drepteid cu curba Γ. Daca Γ este o dreapta, notata d, O ∈ d si AO ⊥ d, vom obtine stro-foida dreapta (pe scurt, strofoida), curba la care ne-am referit ın sectiunile precedente.Renuntand la conditia AO ⊥ d, obtinem strofoida oblica, cu proprietati asemanatoarecu ale strofoidei drepte. Daca Γ este un cerc, O este centrul sau si A se afla pe cerc, seobtine melcul trisector. Daca Γ este o dreapta, O nu-i apartine si A este ”la infinit”,curba strofoidala corespunzatoare este hiperbola.

Bibliografie

1. C. Climescu - Strofoida (Logociclica), Recreatii Stiintifice, 2(1884), 141-143.

2. I. Creanga si colab. - Curs de geometrie analitica, Ed. Tehnica, Bucuresti, 1951.

3. R. Ferreol - Strophoıde droite (ou strophoide de Newton), http://www.mathcurve.com/courbes2d/strophoiddroite/strphoiddroite.shtml.

4. R. Ferreol - Courbe strophoıdale, http://www.mathcurve.com/courbes2d/strophoidale/strophoidale.shtml.

5. J. Lemaire - Hyperbole equilatere et courbes derivees, Vuibert, Paris, 1927.

101

O generalizare a teoremei lui Cosnita

Ion PATRASCU 1

Omagiu adus lui Cezar Cosnita,

la centenarul nasterii sale

Abstract. This note is a tribute to the memory of the Romanian mathematician CezarCosnita, as this year there are 100 years since his birth. A theorem of concurrence publishedby C. Cosnita in 1941 is extended.

Keywords: Cosnita point, Kariya point, barycentric coordinates.

MSC 2000: 51M04.

In luna octombrie, 2010, se ımplinesc 100 de ani de la nasterea profesorului uni-versitar Cezar Cosnita (1910-1962). Nota de fata este dedicata acestui evenimentsi contine generalizarea unei teoreme ce apartine lui C. Cosnita.

Teorema lui Cosnita. Fie O centrul cercului circumscris triunghiului ABC siA1, B1, C1 centrele cercurilor circumscrise triunghiurilor BOC, COA, respectiv AOB.Atunci dreptele AA1, BB1, CC1 sunt concurente [4].

Punctul de concurenta a dreptelor AA1, BB1, CC1 se numeste punctul lui Cosnita.

Teorema. Fie P un punct ın planul triunghiului ABC, nesituat pe cercul circum-scris sau pe laturile acestuia, A′B′C ′ triunghiul podar al lui P si A1, B1, C1 puncteastfel ıncat PA′ · PA1 = PB′ · PB1 = PC ′ · PC1 = k, k ∈ R∗. Atunci drepteleAA1, BB1, CC1 sunt concurente.

Demonstratie. Fie α, β, γ coordonatele baricentrice ale lui P . Din α = aria(PBC),

avem PA′ =2α

asi din PA′ · PA1 = k, gasim PA1 =

ak

2α(am considerat P ın

interiorul triunghiului ABC, vezifigura). Notam cu D si respectiv P1

proiectiile ortogonale ale lui A1 pe AD

si P pe A1D. Deoarece ÖPA1D ≡ ÕABC(unghiuri cu laturile perpendiculare)

avem: A1P1 = PA1·cosB =ak

2αcosB.

Din γ = aria(PAB), rezulta ca

PC ′ =2γ

csi A1D = A1P1 + P1D =

ak cosB

2α+

c. Notam A1(α1, β1, γ1)

si avem: γ1 =kac cosB + 4γα

4α, α1 =

aria(A1BC) =1

2BC · A1A

′ =a2k − 4α2

4αsi, analog ca ın calculul lui γ1, β1 =

kab cosC + 4βα

4α. Adica

1Profesor, Colegiul National ”Fratii Buzesti”, Craiova

102

A1

a2k − 4α2

4α,kab cosC + 4βα

4α,kac cosB + 4γα

.

In acelasi mod aflam ca

B1

kab cosC + 4βα

4β,b2k − 4β2

4β,kcb cosA+ 4βγ

,

C1

kab cosB + 4γα

4γ,kbc cosA+ 4γβ

4γ,c2k − 4γ2

.

In conformitate cu sectiunea 28 din [1] sau [2], cevienele AA1, BB1, CC1 sunt con-curente daca si numai daca α2β3γ1 = α3β1γ2. Cum ın cazul nostru aceasta relatie severifica, dreptele AA1, BB1, CC1 sunt concurente. Teorema se demonstreaza ın modanalog ın cazul cand punctul P este ın exteriorul triunghiului ABC.

Vom numi punctul lor de concurenta punctul generalizat al lui Cosnita.

Observatii. 1. Conditiile din enuntul teoremei au interpretarea geometrica:punctele A1, B1, C1 se afla pe perpendicularele din P pe laturile triunghiului si suntinversele punctelor A′, B′, C ′ ın raport cu cercul de centru P si raza |k|.

2. Teorema lui Cosnita se obtine ın cazul particular P = O si k =R2

2(O si

R sunt centrul si respectiv raza cercului circumscris). Intr-adevar, OA′ = R cosA

si cu teorema sinusurilor aplicata ın BOC avema

sin 2A= 2OA1 (A1 fiind centrul

cercului circumscris BOC). De unde OA′ ·OA1 =R2

2; analog gasim OB′ ·OB1 =

OC ′ ·OC1 =R2

2.

3. Se verifica usor ca luand P = I (centrul cercului ınscris) si k = r(r + a), a > 0dat, iar r raza cercului ınscris, dreptele AA1, BB1, CC1 sunt concurente ın punctullui Kariya. Asadar, teorema prezentata este o generalizare si a teoremei lui Kariya(vezi [4]).

Bibliografie

1. C. Cosnita - Coordonnees barycentriques, Bucarest, Paris, Librairie Vuibert, 1941.

2. C. Cosnita - Geometrie analitica ın coordonate baricentrice, Editura Reprograph,Craiova, 2005.

3. C. Barbu - Variatiuni pe tema punctului lui Cosnita, Gazeta Matematica nr. 4/2010,180-185.

4. C. Barbu - Teoreme fundamentale din geometria triunghiului, Editura Unique, Bacau2008.

103

Criteriu pentru calculul unor limite de functii

Valentina BLENDEA1, Gheorghe BLENDEA2

Abstract. The theorems of Cesaro-Stolz and Cauchy-d′Alembert in the theory of sequences areextended to the case of positive real functions. The results thus obtained are use to solve certaincalculus problems.

Keywords: Cesaro-Stolz theorem, Cauchy-d′Alembert theorem.

MSC 2000: 26A03.

Scopul acestei note este de a extinde unele rezultate din teoria limitelor de sirurila limite de functii. In acest sens vom aminti:

Teorema. a) (Cesaro-Stolz). Fie (an)n≥1 si (bn)n≥1 doua siruri de numere reale

cu proprietatile: i) 0 < bn ↑ +∞; ii) limn→∞

an+1 − anbn+1 − bn

= l ∈ R. Atunci exista

limn→∞

anbn

= l.

b) (Cauchy-d ′Alembert). Daca (an)n≥1 este un sir de numere strict pozitive si

limn→∞

an+1

an= l ∈ R, atunci exista lim

n→∞n√an = l.

In continuare prezentam o varianta a teoremei de mai sus pentru functii si catevaaplicatii.

Propozitie. 1) Fie a si t > 0 si functiile f, g : [a,+∞) → R, cu proprietatile:a) f este marginita pe orice interval marginit din [a,+∞);b) g ia valori strict pozitive, este strict crescatoare si nemarginita.

Daca exista limx→+∞

f(x+ t)− f(x)

g(x+ t)− g(x)= l ∈ R, atunci exista lim

x→+∞

f(x)

g(x)= l.

2) Fie a > 0, b > 0, t > 0 si functia h : [a,+∞) → [b,+∞) care are proprietatea ca

este marginita pe orice interval marginit din [a,+∞). Daca exista limx→+∞

h(x+ t)

h(x)=

l ∈ R∗+, atunci exista lim

x→+∞[h(x)]

1x = l

1t .

Demonstratie. 1) Cazul I: l ∈ R. Pentru orice ε > 0, exista δε > a astfel ıncat

∀x > δε ⇒ l − ε

2<

f(x+ t)− f(x)

g(x+ t)− g(x)< l +

ε

2⇔

⇔l − ε

2

[g(x+ t)− g(x)] < f(x+ t)− f(x) <

l +

ε

2

[g(x+ t)− g(x)].

Fie x > δε+3t, nx ∈ N∗ astfel ıncat x−nxt ∈ (δε+t, δε+3t). Aplicand inegalitateaprecedenta pentru x− t > x−2t > . . . > x−nxt > δε+ t > δε si adunand inegalitatileobtinute, deducem ca

l − ε

2

[g(x)− g(x− nxt)] < f(x)− f(x− nxt) <

l +

ε

2

[g(x)− g(x− nxt)].

1Profesor, Colegiul ”National”, Iasi2Profesor, Colegiul National ”M. Eminescu”, Iasi

104

Din stricta monotonie si nemarginirea lui g rezulta ca limx→+∞

g(x) = +∞. Se obtine:

l − ε

2+

1

g(x)

hf(x− tnx)−

l − ε

2

g(x− tnx)

i<

f(x)

g(x)<

< l +ε

2+

1

g(x)

hf(x− tnx)−

l +

ε

2

g(x− tnx)

i.

Fie functiile h, h′ : (δε+t, δε+3t) → R, h(y) = f(y)−l − ε

2

g(y), h′(y) = f(y)−

l +ε

2

g(y), care sunt marginite din ipotezele a) si b), deci exista M = Mε > 0 astfel

ıncat |h(y)| < M, |h′(y)| < M, ∀y ∈ (δε + t, δε + 3t). Din limx→+∞

1

g(x)= 0 ⇒ ∃δ′ε > a

astfel ıncat 0 <1

g(x)<

ε

2M, ∀x > δ′ε. Deducemf(x)g(x)

− l

< ε, ∀x > maxδε, δ′ε, deci limx→+∞

f(x)

g(x)= l.

Cazul II: l = ±∞. Se trateaza asemanator.2) Daca luam ın 1) f(x) = ln(h(x)) si g(x) = x, x ≥ a, acestea verifica ipotezele

a) si b) din 1), deci

limx→+∞

f(x)

x= lim

x→+∞

f(x+ t)− f(x)

t= lim

x→+∞

lnh(x+ t)

h(x)

t=

=ln l

t= ln l

1t ⇒ lim

x→+∞[h(x)]

1x = l

1t .

Aplicatii.1. Daca functia f : [1,+∞) → R este marginita pe orice interval marginit din

[1,+∞) si exista limx→+∞

f(x+ 1)− f(x)

2x+ 1= l ∈ R, atunci exista lim

x→+∞

f(x)

x2= l.

Solutie. Functia f din enunt si g : [1,+∞) → R, g(x) = x2 verifica ipotezele dinPropozitiei, punctul 1), cu t = 1, de unde rezulta concluzia.

2. Se considera numarul a > 0 si o functie f : [a,+∞) → R care este marginita pe

orice interval marginit din [a,+∞), cu proprietatea ca limx→+∞

f(x)

x2= 0. Sa se arate

ca, daca exista limx→+∞

f(x+ 1)− f(x)

x, atunci aceasta este egala cu zero.

Solutie. Functiile f si g : [a,+∞) → R, g(x) = x2 verifica ipotezele Propozitiei,punctul 1), si avem:

limx→+∞

f(x+ 1)− f(x)

x= l ∈ R ⇒ lim

x→+∞

f(x+ 1)− f(x)

2x+ 1=

= limx→+∞

f(x+ 1)− f(x)

x· x

2x+ 1= l

1

2.

105

Conform aplicatiei precedente, rezulta ca limx→+∞

f(x)

x2=

l

2, deci l = 0.

3. Fie f : [1,+∞) → [1,+∞) o functie continua si neconstanta astfel ıncat

limx→+∞

f(x+ 1)

f(x)= l ∈ R∗

+. Atunci exista limx→+∞

Z x

1f(t)dt

1x= l.

Solutie. Fie a > 0 si functia F : [1,+∞) → [a,+∞), F (x) =

8<:a, x = 1Z x

1f(t)dt, x > 1

,

care este derivabila pe (1,+∞). Functia F este marginita pe orice interval marginit

din [1,+∞) si limx→+∞

F (x+ 1)

F (x)= lim

x→+∞

f(x+ 1)

f(x)= l (se utilizeaza regula lui l′Hospital).

Aplicand Propozitia, punctul 2), se obtine rezultatul.

4. Sa se arate ca limx→+∞

[x]1x=1, unde [x] reprezinta partea ıntreaga a numarului

x.Solutie. Considerand functia f : [1,+∞) → [1,+∞), f(x) = [x], observam ca

f este marginita pe orice interval marginit din [1,+∞) si limx→+∞

[x+ 1]

[x]= 1, deci,

conform Propozitiei, punctul 2), rezulta ca limx→+∞

[x]1x = 1.

Rodolph Neculai Raclis (1896-1966), doctor ın stiinte matematice la Sorbonaın 1930, a fost directorul Institutului Matematic Roman si fondatorul revistei Numerus(v. articolul de la pag. 128).

Ca un omagiu adus lui, ın Numerus vol. 3, caietul 29, pag. 196, este publicataurmatoarea problema (reprodusa fidel!):

3.3309 (Ion Armean). Sa se faca urmatoarea operatie:√R a c l i s = ∗s∗∗∗ ∗ ∗∗ ∗ s∗ ∗ ∗ ∗∗ ∗ ∗ s

/

notand prin litere anumite cifre si prin ∗ diferite cifre ce trebuie determinate prinrationament.

N.B. Raspunsul se gaseste la pag. 136.

106

Asupra unor puncte de concurenta ale unui triunghi

Dan POPESCU1

Omagiu adus lui Cezar Cosnita,

la centenarul nasterii sale

Abstract. Some remarkable points in the geometry of triangle are presented in a unified way,as being Jacobi points in particular positions.

Keywords: Jacobi point, Fermat-Torricelli point, Napoleon′s point, Vecten′s point, Morley′spoint, Cosnita′s point.

MSC 2000: 51M04.

Nota ısi propune o abordare metodologica unitara si accesibila a catorva problemede concurenta din geometria elementara a triunghiului.

Urmatorul rezultat este atribuit matematicianului german Carl Gustav JacobJacobi (1804-1851). Mai este numit si teorema Fermat-Torricelli generalizata ([2]).

Teorema 1. Fie triunghiul ABC si punctele A′, B′, C ′ din planul sau ıncatÖA′BC ≡ ÖC ′BA, ÖB′CA ≡ ÖA′CB si ÖC ′AB ≡ ÖB′AC. Atunci dreptele AA′, BB′ siCC ′ sunt concurente ıntr-un punct J , numit punctul lui JACOBI.

Demonstratie. Sa analizam doar cazul AA′ ∩ [BC] = A1 si BC ∩ (AA′) = θ,A

A

C

B

B

C

aa

b

b

g

g

A1

B1

C1

celelalte cazuri presupunand rationamente similare.

Daca α = m(ÖB′AB), β = m(ÖA′BC), γ = m(ÖB′CA),

atunciA1B

A1C=

A[ABA′]

A[ACA′]=

AB ·A′B sin(B + β)

AC ·A′C sin(C + γ). Se

obtin relatii analoage relativ la punctele B1 ∈ AC si

C1 ∈ AB si se verifica usor caA1B

A1C· B1C

B1A· C1A

C1B=

1. Conform reciprocei teoremei lui Ceva, drepteleAA1, BB1 si CC1 sunt concurente.

De observat ca punctul J poate fi si ın exteriorultriunghiului ABC.

Particularizand α, β, γ, vom obtine mai multe puncte remarcabile ale triunghiului.

I. Punctul lui Fermat. In exteriorul triunghiului ABC se construiesc triunghiu-rile echilaterale A′BC,B′AC si C ′AB. Evident, perechile de semidrepte ([AB′, [AC ′),

([BA′, [BC ′) si ([CA′, [CB′) sunt izogonale pentru unghiurile triunghiului bA, ÒB si,

respectiv, ÒC. Atunci, dreptele AA′, BB′ si CC ′ se intersecteaza ıntr-un punct F ,punctul lui FERMAT.

O chestiune conexa acestui rezultat o constituie problema gasirii unui punct Pdin interiorul unui triunghi ABC cu toate unghiurile de masura mai mica decat 120,astfel ca suma PA + PB + PC sa fie minima. Acest punct a fost cercetat si deE. TORRICELLI. Daca P ∈ Int(ABC) si triunghiul BPC se roteste cu 60 ınjurul lui B, atunci PA+ PB + PC = PA+ PP ′ + P ′C ′. Asa ca suma este minima,

1Profesor, Colegiul National ”Stefan cel Mare”, Suceava

107

daca si numai daca P, P ′ sunt pe CC ′. Astfel, punctul lui TORRICELLI coincide cupunctul lui FERMAT.

II. Punctul lui Napoleon. Daca pe laturile triunghiului ABC si ın exte-riorul lui se construiesc trei triunghiuri echilaterale cu centrele B,E si F , atuncidreptele AD,BE si CF sunt drepte concurente ıntr-un punct N , numit punctul luiNAPOLEON, dupa numele ımparatului Frantei NAPOLEON BONAPARTE.

Triunghiul echilateral cu centrul D are o latura [BC], cel cu centrul E are o latura[AC] si cel cu centrul F are o latura [AB]. Deci si punctul lui NAPOLEON este totun punct JACOBI. Totodata, triunghiul DEF este si el echilateral, dupa cum sededuce prin calculul unei singure lungimi DE.

III. Punctul lui Vecten. Daca pe laturile triunghiului ABC si ın exteriorullui se construiesc patrate cu centrele D,E si F atunci dreptele AD,BE si CF suntdrepte concurente ın punctul V , numit punctul lui VECTEN, fiind pus ın evidenta deprofesorul francez de liceu M. VECTEN (1812).

IV. Punctul lui Morley. Acesta este un alt punct de tip JACOBI. In 1899,profesorul FRANK MORLEY (1860-1937) a aratat ca trisectoarele unghiurilor unuitriunghi ABC determina un triunghi echilateral A′B′C ′, iar AA′, BB′ si CC ′ suntdrepte concurente ıntr-un punct, numit punctul lui MORLEY.

V. Punctul lui Cosnita si dualul sau. Spre deosebire de dualul sau, acestanu-i punct de tip Jacobi; ınsa, ın [2] se da rezultatului lui Cosnita o demonstratie pebaza Teoremei 1. Vom prezenta o alta, directa si simpla.

Teorema 2 (Cosnita). Fie triunghiul ABC, O centrul cercului circumscris tri-unghiului, OA, OB si OC centrele cercurilor circumscrise triunghiurilor OBC,OACsi OAB, respectiv. Atunci dreptele AOA, BOB si COC sunt concurente ın punctulK1, numit punctul lui COSNITA.

A

B C

O

OC O

B

C

A

B

OA

Demonstratie. Se considera A′ = AOA ∩ BC

si punctele B′, C ′ definite analog. Avem m×(BOC) =

2A etc. Se deduce ca mØ(BOAC) = 360 − 4A.

Astfel, mØ(CBOA) = mØ(BCOA) = 2A − 90. Apoi,

sinØ(ABOA) = sin(B + 2A− 90) = cos(A−C). Analog,

sinØ(ACOA) = cos(A − B). Deci,A′B

A′C=

A[ABOA]

A[ACOA]=

AB ·BOA sin(ÕABOA)

AC · COA sin(ÕACOA)=

c cos(A− C)

b cos(A−B). Analog,

B′C

B′A=

a cos(B −A)

c cos(B − C)si

C ′A

C ′B=

b cos(C −B)

a cos(C −A)si concurenta este evidenta, conform reciprocei teoremei lui Ceva.

Teorema 3. Daca I este centrul cercului ınscris ın triunghiul ABC, IA, IBsi IC sunt centrele cercurilor ınscrise triunghiului IBC, IAC si, respectiv, IAB.Atunci dreptele AIA, BIB si CIC sunt concurente ıntr-un punct K2, numit punctullui COSNITA dual.

108

Demonstratie. Se observa ca K2 este un punct Jacobi cu α =A

4, β =

B

4,

γ =C

4.

In [1], articol recent aparut, sunt date un numar mare de rezultate ınrudite cuteorema lui Cosnita (printre care si Teorema 3, cu o demonstratie diferita).

Bibliografie

1. C. Barbu - Variatiuni pe tema punctului lui Cosnita, GM(B)-4/2010, 180-185.

2. M. de Villiers - A Dual to Kosnita′s Theorem,, Math. and Inf. Quart., 6(1996),169-171.

IMPORTANT

• In scopul unei legaturi rapide cu redactia revistei, pot fi utilizate urmatoareleadrese e-mail: t [email protected] si [email protected] . Peaceasta cale colaboratorii pot purta cu redactia un dialog privitor la ma-terialele trimise acesteia, procurarea numerelor revistei etc. Sugeram cola-boratorilor care trimit probleme originale pentru publicare sa le numerotezesi sa-si retina o copie xerox a lor pentru a putea purta cu usurinta o discutieprin e-mail asupra acceptarii/neacceptarii acestora de catre redactia revistei.

• La problemele de tip L se primesc solutii de la orice iubitor de matematicielementare (indiferent de preocupare profesionala sau varsta). Fiecare dintresolutiile acestor probleme - ce sunt publicate ın revista dupa un an - va fiurmata de numele tuturor celor care au rezolvat-o.

• Adresam cu insistenta rugamintea ca materialele trimise revisteisa nu fie (sa nu fi fost) trimise si altor publicatii.

• Rugam ca materialele tehnoredactate sa fie trimise pe adresa redactieiınsotite de fisierele lor (de preferinta ın LATEX).

• Pentru a facilita comunicarea redactiei cu colaboratorii ei, autorii materi-alelor sunt rugati sa indice adresa e-mail.

109

Cateva proprietati caracteristiceale triunghiului isoscel

Razvan CEUCA1

Abstract. Let P be the intersection point of the Cevian lines AA′, BB′, CC′ in the triangleABC. A couple of remarkable points of the triangle (denoted G,H,K,Γ, N) are identified, such thatthe congruence of two segments among [PA′], [PB′] and [PC′] implies the property of the triangleto be isosceles.

Keywords: simedian, Gergonne′s point, Nagel′s point.

MSC 2000: 51M04.

Fie dat un triunghiABC si fie P punctul de concurenta a trei cevieneAA′, BB′, CC ′

cu A′ ∈ (BC), B′ ∈ (CA) si C ′ ∈ (AB). Ne punem urmatoarea ıntrebare:daca doua dintre segmentele [PA′], [PB′], [PC ′] sunt congruente, triunghiul ABC

este isoscel?Cu un contraexemplu, vom arata ca raspunsul este negativ. In acest scop, avem

nevoie de cateva pregatiri. Notam cu m =A′B

A′C, n =

B′C

B′Asi p =

C ′A

C ′Brapoartele

care determina pozitia cevienelor AA′, BB′ si respectiv CC ′ fata de triunghi. Mai jos,vom stabili formulele care dau lungimile segmentelor [PA′], [PB′] si [PC ′] functie dea, b, c,m, n, p.

A

BC

A

B

C

P

Intr-adevar, conform teoremei lui Van Aubel, avemPA

PA′ =B′A

B′C+

C ′A

C ′B=

1

n+ p, de unde

(1) PA′ =n

1 + n+ npAA′.

Cum aAA′ 2 = b2A′B+ c2A′C − aA′B ·A′C (teorema lui

Stewart) si cum A′B =m

m+ 1a, A′C =

1

m+ 1a, obtinem

(2) AA′ 2 =1

m+ 1(mb2 + c2 − m

m+ 1a2).

Din (1) si (2), deducem formula pentru PA′ 2; apoi, prin analogie, deducem PB′ 2 siPC ′ 2. Avem:

(3)

PA′ 2 =

n

1 + n+ np

2

· 1

m+ 1

mb2 + c2 − m

m+ 1a2,

PB′ 2 =

p

1 + p+ pm

2

· 1

n+ 1

nc2 + a2 − n

n+ 1b2,

PC ′ 2 =

m

1 +m+mn

2

· 1

p+ 1

pa2 + b2 − p

p+ 1c2.

1Elev, Colegiul ”National”, Iasi

110

Contraexemplul urmator arata ca nu are loc implicatia PA′ = PB′ = PC ′ ⇒ ABC

isoscel. Luand a =3√15

20c, b =

√31

4c, obtinem un triunghi scalen pentru orice c > 0

(ıntr-adevar, a < c < b si b < a + c). Pentru m =1

2, n =

2

3, p = 3 avem mnp = 1,

deci cevienele AA′, BB′, CC ′ sunt concurente. Utilizand formulele (3), dupa calcule,

gasim: PA′ 2 = PB′ 2 = PC ′ 2 =9

220.

Vom indica, ın continuare, ceviene particulare pentru care raspunsul la ıntrebareapusa este pozitiv.

I. Mediane (P ≡ G). Avem: GB′ = GC ′ ⇒ 3GB′ = 3GC ′ ⇒ BB′ = CC ′

(BB′, CC ′ mediane) ⇒ ABC isoscel cu varful A.

II. Inaltimi (P ≡ H). Avem HB′ = HC ′ ⇒ HB′A ≡ HC ′A ⇒ AB′ =AC ′ ⇒ AB′B ≡ AC ′C ⇒ AB = AC (triunghiurile ce intervin sunt drept-unghice).

III. Bisectoare (P ≡ I). Este un caz mai complicat.Daca numai doua dintre segmentele [IA′], [IB′], [IC ′] sunt congruente, atunci

ABC poate sa nu fie isoscel, dupa cum arata exemplul: ABC cu A = 60,B = 100, C = 20 nu este, evident, isoscel, dar se constata ca IB′ = IC ′ (ıntr-

adevar, avem m(ÕIB′A) = C +B

2= 70, m(ÕIC ′A) = B +

C

2= 110 si, cu teorema

sinusurilor ın AIB′ si AIC ′, obtinem 2 · IB′ =AI

sin 70=

AI

sin 110= 2IC ′, adica

IB′ = IC ′).Daca ınsa IA′ = IB′ = IC ′, atunci ABC este echilateral. Intr-adevar, tinand

seama de formula (1) si analoagele sale, ın carem =c

b, n =

a

c, p =

b

a, conditia noastra

revine la aAA′ = bBB′ = cCC ′. Cu formulele ce dau lungimile bisectoarelor, obtinem1

b+ ccos

A

2=

1

c+ acos

B

2=

1

a+ bcos

C

2. Punand aici b + c = 2R(sinB + sinC) =

4R cosA

2cos

B − C

2etc., deducem egalitatile cos

B − C

2= cos

C −A

2= cos

A−B

2.

Consideratii de rutina ne conduc la A = B = C = 60.

IV. Simediane (P ≡ K). Vom avea ın vedere triunghiuri ascutitunghice (a2 +

b2 − c2 > 0 etc.). Intrucat m =c2

b2, n =

a2

b2, p =

b2

a2, cu formula (1) obtinem

KA′ =a2

a2 + b2 + c2AA′, iar cu (2) deducem ca lungimea simedianei AA′ este data

de AA′ 2 =b2c2

b2 + c2

2− a2

b2 + c2

. Ca urmare, vom avea: KB′ = KC ′ ⇔ b2BB′ =

c2CC ′ ⇔ b2

c2 + a2

2− b2

c2 + a2

=

c2

a2 + b2

2− c2

a2 + b2

b2

c2 + a2− 1

2

=c2

a2 + b2− 1

2

b2

c2 + a2− c2

a2 + b2

b2

c2 + a2+

c2

a2 + b2− 2

= 0 ⇔ b2

c2 + a2=

111

c2

a2 + b2(ultima paranteza scrisa fiind nenula ın conditia caABC este ascutitunghic)

⇔ (b2 − c2)(a2 + b2 + c2) = 0 ⇔ b = c.Afirmatia si calculul precedente raman valabile si ın cazul triunghiurilor obtuzunghi-

ce ın A (nu ınsa daca unghiul obtuz ar fi ÒB sau ÒC).

V. Ceviene Gergonne (P ≡ Γ). Stim ca AB′ = AC ′ = p − a, BC ′ = BA′ =p− b, CA′ = CB′ = p− c, p = semiperimetrul triunghiului.

Vom arata ın fapt echivalenta conditiilor: 1) b = c, 2) cevienele Gergonne ce pleacadin varfurile B si C sunt congruente, 3) segmentele [ΓB′] si [ΓC ′] sunt congruente.

Intr-adevar, utilizand teorema cosinusului ın ABB′ si ACC ′, avem:

BB′ = CC ′ ⇔ c2 + (p− a)2 − 2c(p− a)(cosA = b2 + (p− a)2 − 2b(p− a) cosA

⇔ b2 − c2 − 2(b− c)(p− a) cosA = 0 ⇔ (b− c)[b+ c− 2(p− a) cosA] = 0

⇔ b = c

(paranteza patrata nu se anuleaza: ın caz contrar, am avea cosA =b+ c

b+ c− a> 1,

fals).

Presupunem acum ca ΓB′ = ΓC ′. Atunci ΓB′C ′ este isoscel, deci ÖBB′C ′ ≡ÖCC ′B′. De asemenea, deoarece AB′C ′ este isoscel, avem si ÖBC ′B′ ≡ ÖCB′C ′.Asadar, BB′C ′ ≡ CC ′B′ si deducem ca BB′ = CC ′ si, ca urmare, b = c.

VI. Ceviene Nagel (P ≡ N). In acest caz, BC ′ = CB′ = p− a, CA′ = AC ′ =p− b, AB′ = BA′ = p− c. Urmam calea parcursa ın cazul precedent.

Cu teorema cosinusului, aplicata ın ABB′ si ACC ′, relativ la cevienele NagelBB′ si CC ′, avem:

BB′ = CC ′ ⇔ c2 + (p− c)2 − 2c(p− c) cosA = b2 + (p− b)2 − 2b(p− b) cosA

⇔ b2 − c2 + [(p− b)2 − (p− c)2]− 2 cosA[b(p− b)− c(p− c)] = 0

⇔ (b− c)[b+ c− a+ cosA(b+ c− a)] ⇔ b = c.

Fie acum NB′ = NC ′, iar ABC va fi considerat ascutitunghic. Observam carelativ la NBC ′ si NCB′ avem: NC ′ = NB′, BC ′ = CB′ = p− a si au unghiuri

opuse ın varful comun N . Atunci, ÖNBC ′ ≡ ÖNCB′, fiind ascutite; triunghiurile suntcongruente si vom avea BN = CN. Rezulta ca BB′ = CC ′ si, apoi, b = c.

Prin ce proprietate sunt ınrudite urmatoarele numere:1, 3, 13, 85, 781, 9 331, 137 257, 2 396 745, 48 427 461 ?

N.B. Raspunsul se gaseste la pag. 127.

112

Valeurs et vecteurs co-propresd′une application semi-lineaire

Adrien REISNER1

Abstract. The notions of a semi-linear mapping, of a co-eigenvalue and co-eigenvector areintroduced. A number of properties thereof outline the similarities/differences of these notions withrespect to the corresponding classical ones.

Keywords: semi-linear, co-eigenvalue, co-eigenvector.

MSC 2000: 15A18.

Soit E un espace vectoriel complexe de dimension n ≥ 1. Pour u ∈ L(E), λ ∈ Cest une valeur propre s′il existe x ∈ E non nul tel que u(x) = λx; le vecteur x estun vecteur propre de u associe a la valeur propre λ. Enfin, le spectre de u, noteSp u, designe l′ensemble des valeurs propres de l′endomorphisme u, tandis que Sp Mu

designera le spectre de la matrice Mu representant u dans une base quelqonque de E.

Remarque. E etant un C–espace vectoriel (et plus generalement si E est unK – espace vectoriel ou K est un corps algebriquement clos) tout endomorphisme ude E admet n valeurs propres, les n racines du polynome caracteristique χu(X) =det(Mu − IX).

Le but de cet article est l′etude de certaines proprietes des applications semi-lineaires de E dans lui-meme.

Definitions. a) Une application u de E dans lui-meme est semi-lineaire si ellepossede la propriete suivante: ∀a ∈ C, ∀x, y ∈ E, u(ax + y) = au(x) + u(y). (udesignera desormais une application semi-lineaire de E dans lui-meme.)

b ) α∈C est une valeur co-propre de u s′il existe un vecteur x = 0 tel que: u(x)=αx.Un tel vecteur x est un vecteur co-propre associe a la valeur co-propre α.

x etant un vecteur co-propre de u, il existe au plus une seule valeur co-propre deu associee a x. En effet, si u(x) = λx = µx, alors (λ−µ)x = 0, d′ou λ = µ car x = 0.

De facon evidente, la composee de deux applications semi-lineaires est une appli-cation lineaire En particulier, si u est une application semi-lineaire alors l′applicationu2 est lineaire.

Proposition 1. Si α est une valeur co-propre de u, pour tout reel θ le nombrecomplexe αeiθ est encore une valeur co-propre de u. Si x est un vecteur co-propreassocie a α, alors e−iθ/2x est un vecteur co-propre associe a α eiθ.

Demonstration. Soit x un vecteur co-propre associe a la valeur co-propre α,i.e. u(x) = αx avec x = 0. Il vient alors pour tout reel θ: u(e−iθ/2) = eiθ/2 · αx =

1Centre de Calcul E.N.S.T., Paris; e-mail: [email protected]

113

(eiθα) (e−iθ/2x) et par suite eiθα est une valeur co-propre de u associee au vecteurco-propre e−iθ/2x.

Remarque. En particulier, α est valeur co-propre de u si et seulement si |α| estvaleur co-propre de l′application semi-lineaire u.

Proposition 2. Si α = 0 est une valeur co-propre de u, l′ensemble Eα desvecteurs co-propres associes est un espace vectoriel reel mais pas un espace vectorielcomplexe. E0 est aussi bien un R-espace vectoriel qu′un C-espace vectoriel.

Soit B = (e1, e2, . . . , en) et B1 = (f1, f2, . . . , fn) deux bases de l′espace vectorielE et P la matrice de passage de la base B a la base B1 : P = Pass(B,B1).

Proposition 3. a) A toute application semi-lineaire u est associee dans la baseB une matrice carree complexe A d′ordre n telle que la relation y = u(x) s′ecrive:Y = AX. On notera: A = Mat(u,B).

b) Si A = Mat(u,B) et B = M(u,B1) alors on a: B = P−1AP .

Demonstration. a) Pour x =nP1

xiei on a u(x) =nP1

xiei. Posant alors u(ei) =

nP1

akiek il vient y = u(x) =Pk

(Pi

akixi)ek soit yk =Pi

akixi et finalement Y = AX.

b) Avec des notations evidentes (X et X1 attache a x dans les bases B et B1

respectivement) X = PX1, Y = PY1, Y = AX,Y1 = BX1. Donc APX1 = AP X1 =PBX1. Cette egalite etant realisee pour tout X1, on en deduit: AP = PB soitB = P−1AX1 – formule de changement de base.

La relation AℜB ⇔ ∃P ∈ GLn(C) telle que B = PAP−1

est de facon evidenteune relation d′equivalence. Deux matrices A et B appartenant a la meme classe sontdites co-semblables. Compte tenu de la proposition precedente l′application semi-lineaire u est representee dans deux bases differentes par des matrices co-semblables.Deux matrices co-semblables ont le meme co-spectre. [En effet si AX = αX et

B = PAP−1

on a avec Y = PX : BY = PAP−1

PX = PAX = αY .] Avec lesnotations de la Proposition 3, a), si α est une valeur co-propre de u on dira que αest une valeur co-propre de la matrice A. Si x est un vecteur co-propre associe a α,on a alors: AX = αX. La matrice A est co-trigonalisable (resp. co-diagonalisable)si cette matrice est co-semblable a une matrice trigonale (resp. diagonale) – voircomplements.

Exemple numerique. Soit a chercher valeurs co-propres de la matrice A =0 −11 0

. Le vecteur X =

a

b

etant un vecteur co-propre associe a la valeur co-

propre α on a l′equivalence suivante: AX = αX ⇔ −b = αa, a = αb. On en deduit:a = α (−αa) soit a = 0, car 1 + αa = 1 + |α|2 = 0 et finalement a = b = 0, i.e. lamatrice A n′a pas de valeur co-propre. C′est une difference essentielle avec les valeurspropres – voir la premiere remarque.

114

Proposition 4. a) Si une matrice reelle A admet une valeur propre reelle λ,alors cette matrice admet au moins une valeur co-propre.

b) Si α est valeur co-propre de la matrice carree complexe A, alors le nombre reel

|α|2 est valeur propre de la matrice AA.

Demonstration. a) Il existe un vecteur X reel, non nul verifiant AX = λX.Donc AX = λX, i.e. λ est aussi valeur co-propre, c.q.f.d.

b) Soit X = 0 tel que AX = αX ou A est une matrice complexe. On a alors:

AX = αX et donc AAX = αAX = |α|2 X, i.e. |α|2 est valeur propre de la matriceAA.

Corollaire. Pour que le reel positif ou nul µ soit valeur co-propre de la matriceA il faut et il suffit que le reel µ2 soit valeur propre de la matrice AA.

Demonstration. (⇒) resulte de l′assertion b) de la proposition precedente.(⇐) Soit µ ≥ 0, µ2 ∈ Sp AA et X = 0 un vecteur propre associe, i.e. AAX = µ2X.

Montrons que µ est valeur co-propre de la matrice A en envisageant deux cas:I. AX et X sont lies. Etant donne que X est non nul, on a: AX = αX. Donc

A(AX) = A(αX) = αAX = |α|2 X soit µ2 = |α|2, i.e. µ = αeiθ. Par suite AX =αX ⇒ AX = µe−iθX, c′est a dira que µe−iθ est une valeur co- propre de la matriceA et la proposition 1 permet de conclure que µ est aussi une valeur co-propre de A,c.q.f.d.

II. les vecteurs AX et X sont independants. Dans ce cas le vecteur non nulY = µX + AX verifie: AY = A(µX + AX) = µAX + µ2X = µY , i.e. µ est valeurco-propre de la matrice A.

Considerons maintenant le cas particulier d′une matrice triangulaire. Etant donneeune matrice triangulaire superieure A, on a la

Proposition 5. a) Si λ est valeur propre de A, alors pour tout reel θ le nombrecomplexe λeiθ est une valeur co-propre de la matrice A.

b) Si α est une valeur co-propre de A alors il existe un reel θ tel que le nombrecomplexe αeiθ soit valeur propre de la matrice A.

Demonstration. a) Si A =

λ1 ∗ ∗ ... ∗0 λ2 ∗ ... ∗... ... ... ... ...0 ... ... 0 λn

, alors

AA =

|λ1|2 ∗ ... ... ∗0 |λ2|2 ∗ ... ∗... ... ... ... ...

0 ... ... 0 |λn|2

.

Les reels |λ1|2 , |λ2|2 , . . . , |λn|2 sont les valeurs propres de la matrice AA et, comptetenu du corollaire precedent, les reels |λ1|, |λ2|, . . . , |λn| sont valeurs co-propres deA. La remarque de la proposition 1 montre alors que λ1, λ2, . . . , λn sont valeurs

115

co-propres de la matrice A et enfin la Proposition 1 permet de conclure: ∀θ ∈R, (λje

iθ)j:1,...,n sont valeurs co-propres de la matrice A.b) Soit α une valeur co-propre de A. |α| est aussi une valeur co-propre de la matrice

A. Le Corollaire montre alors que |α|2 est valeur propre de la matrice AA. Donc, ilexiste j ∈ [1, n] tel que |α| = |λj | soit (puisqu′il existe θ′ tel que αeiθ

′ > 0) : α = |λj|e−iθ′

= λje−iθ ou λj = αeiθ ∈ Sp A.

Exemple numerique. Etant donnee la matrice triangulaire superieure A =i 10 i

, i ∈ Sp A. En particulier, |i| = 1 est valeur co-propre de A. Le vecteur

X =

a+ ibc+ id

etant un vecteur co-propre associe a la valeur co-propre 1, on a les

equivalences suivantes: AX = X ⇔i 10 i

a− ibc− id

=

a+ ibc+ id

⇔ ia+b+c− id =

a + ib et ic + d = c + id. Par suite: c = d, b + c + i(a − d) = a + ib ou b + c = a.

Le vecteur

1 + i0

est vecteur co-propre associe a la valeur co-propre 1. D′ailleurs

l′ensemble E1 des vecteurs co-propres associes a la valeur co-propre 1 est l′espacevectoriel reel – voir Proposition 2 –

E1 =

§k1

1 + i0

+ k2

1

1 + i

k1, k2 ∈ Rª.

Remarques concernant la co-diagonalisabilite. Si P−1AP = diag(λj) on a,en designant par cj la j-eme colonne de la matrice P : Acj = λjcj pour j : 1, . . . , n.Par suite, de meme que pour les matrices diagonalisables, cj est vecteur co-proprede la matrice A associe a la valeur co-propre λj . Compte tenu de la remarque dela Proposition 1, on peut meme choisir la matrice diagonale diag (λj) telle que pourj : 1 . . . n, λj ∈ R+.

Soit A = B + iC une matrice carree complexe d′ordre n (ou les matrices B etC sont reelles). Concluons alors avec le theoreme suivant qui indique une conditionnecessaire et suffisante pour qu′un complexe α soit valeur co-propre d′une matrice A:

Theoreme 6. α est valeur co-propre de la matrice A si et seulement si |α| estvaleur propre de la matrice carree reelle d′ordre 2n,D, definie par blocs par D =B CC −B

.

Demonstration. Le complexe α est valeur co-propre de A si et seulement si |α|est aussi valeur co-propre de la matrice A – voir remarque de la Proposition 2. Ilvient alors, X et Y etant reels: (B + iC)(X − iY ) = (BX +CY ) + i(CX −BY ). Levecteur X + iY est vecteur co-propre de la matrice A = B + iC associe a la valeurco-propre |α| si et seulement si BX +CY = |α|X et CX −BY = |α| Y ce qui s′ecrit

encore:

B CC −B

XY

= |α|

XY

i.e., |α| ∈ Sp D.

116

Complements–sans demonstration. On a vu dans le premier exemple nume-rique qu′une matrice peut ne pas avoir de valeurs co-propres. Neanmoins, on a letheoreme suivant:

Theoreme 7. Toute matrice A ∈ M2n+1(C) admet une infinite de valeurs co-propres. (Difference avec les valeurs propres!)

Pour demontrer ce theoreme on montre que pour une telle A matrice le polynomecaracteristique de la matrice AA est un polynome reel ayant au moins une valeurpropre λ reelle positive ou nulle. Le corollaire et la Proposition 1 permettront alorsde conclure.

Les theoremes suivants concernent la co-trigonalisabilite et la co-diagonalisabilitedes matrices:

Theoreme 8. Pour A ∈ Mn(C) les deux assertion suivantes sont equivalentes:

a) il existe P ∈ G Ln(C) telle que A = PTP−1

ou T est une matrice triangulaire,i.e. la matrice A est co-trigonalisable;

b) les valeurs propres de la matrice AA sont reelles positives ou nulles, i.e. SpAA ⊂R+. De plus, la matrice P peut etre choisie telle que P−1 =t P (P unitaire).

Corollaire. La matrice A est symetrique si et seulement si A est unitairement

co-diagonalisable, i.e. ∃U unitaire telle que A = Udiag(λi)U−1

= Udiag(λi)tU .

Theoreme 9. Pour A ∈ Mn(C) les deux assertion suivantes sont equivalentes:

a) il existe P ∈ G Ln(C) telle que A = PDP−1

, ou D est une matrice diagonale,i.e. A est co-diagonalisable;

b) la matrice A verifie les trois conditions suivantes:i) AA est diagonalisable,ii) SpAA ⊂ R+,iii) rgA = rgAA.

References1. J.-M. Arnaudies, H. Fraysse - Cours de mathematiques, Tome 1. Algebre, Editions

Dunod, Paris, 1987.

2. J. Fresnel - Algebre des matrices, Editions Hermann, Paris, 1997.

3. J. Fresnel - Espaces quadratiques, euclidiens hermitiens, Editions Hermann, Paris,1999 (pour les complements).

Vizitati pagina web a revistei Recreatii Matematice:

http://www.recreatiimatematice.ro

117

Proprietati ale primitivelor functiilor periodiceGheorghe IUREA1

Abstract. It is outlined the fact that a small number of elementary properties of periodic func-tions are sufficient for solving various problems proposed at contests and olympiads of mathematics.

Keywords: periodic functions, the primitive of a function.

MSC 2000: 26A42.

In anul 2003, la examenul de bacalaureat a fost propusa urmatoarea

Problema. limx→∞

1

x

Z x

0| sin t| dt este: a) ∞; b) 1; c) 0; d)

2

π.

Este o problema cunoscuta, dar dificila. Un raspuns se poate obtine astfel: fie

sirul xn =1

Z nπ

0| sin t| dt, n ∈ N∗; cum

Z nπ

0| sin t| dt =

n−1Xk=0

Z (k+1)π

kπ| sin t| dt =

n−1Xk=0

Z π

0| sin y| dy = 2n, obtinem xn =

2

π. Prin urmare, limita ceruta, daca exista,

este egala cu2

π.

In aceasta nota demonstram cateva proprietati ale primitivelor functiilor periodicesi totodata indicam un mod de determinare a primitivelor acestor functii.

Propozitie. Fie f, F : R → R, unde f este o functie continua, neconstanta si

periodica de perioada T > 0, iar F (x) =

Z x

0f(t) dt. Atunci:

a)

Z (k+1)T

kTf(t) dt =

Z T

0f(t) dt, pentru orice k ∈ Z;

b)

Z x+T

xf(t) dt =

Z T

0f(t) dt, pentru orice x ∈ R;

c) F (x) =h xT

iF0(T ) +F0

x− T

h xT

i,∀x ∈ R, unde F0 este primitiva lui F pe

[0;T ] care se anuleaza ın zero;d) exista a ∈ R astfel ıncat functia G : R → R, G(x) = F (x)− ax sa fie periodica

si neconstanta;

e) limx→∞

1

x

Z x

0f(t) dt =

1

T

Z T

0f(t) dt;

f) functia F nu admite asimptota oblica spre ∞.

Demonstratie. a) Cu substitutia y = t−kT , obtinem

Z (k+1)T

kTf(t) dt =

Z T

0f(y) dy.

b) Consideram functia g : R → R, g(x) =Z x+T

xf(t) dt = F (x+T )−F (x); atunci

g′(x) = f(x + T ) − f(x) = 0. Rezulta ca g este constanta, deci g(x) = g(0) pentruorice x ∈ R si de aici concluzia.

1Profesor, Liceul Teoretic ”Dimitrie Cantemir”, Iasi

118

c) Pentru x ≥ 0, fie n =h xT

i∈ N, deci nT ≤ x < (n + 1)T . Avem ca

F (x) =

Z T

0f(t) dt+

Z 2T

Tf(t) dt+ ...+

Z nT

(n−1)Tf(t) dt+

Z x

nTf(t) dt. Cu substitutia t−

nT = y, obtinem

Z x

nTf(t) dt =

Z x−nT

0f(y) dy, si folosind a), F (x) = n

Z T

0f(t) dt+Z x−nT

0f(y) dy. Cum x − nT ∈ [0;T ), rezulta ca F (x) = nF0(T ) + F0(x − nT ).

Pentru x < 0, alegem n = −h xT

i∈ N, deci −nT ≤ x < −(n − 1)T si atunci

F (x) =−TR0

f(t) dt+

Z −2T

−Tf(t) dt+ ...+

Z −nT

−(n−1)Tf(t) dt+

Z x

−nTf(t) dt. Cu substitutia

t + nT = y ın ultima integrala si folosind punctul a), F (x) = −n

Z T

0f(t) dt +Z x+nT

0f(y) dy. Dar x + nT ∈ [0;T ), deci F (x) = −nF0(T ) + F0(x + nT ). In

concluzie F (x) =h xT

iF0(T ) + F0

x− T

h xT

i, pentru x ∈ R.

d) Conform punctului b), F (x+T )−F (x) =

Z T

0f(t) dt, prin urmare G(x+T )−

G(x) = F (x+T )−F (x)−aT =

Z T

0f(t) dt−aT . Alegand a =

1

T

Z T

0f(t) dt, rezulta

ca G(x + T ) = G(x), pentru orice x ∈ R, prin urmare, pentru a determinat, G esteperiodica de perioada T .

DacaG este constanta, atunciG′(x) = 0, pentru orice x ∈ R. Rezulta ca f(x)−a =0, pentru orice x ∈ R, contradictie. In concluzie, functia G nu este constanta.

e) limx→∞

R x0 f(t) dt

x= lim

x→∞

G(x)

x+ a

= a, deoarece G fiind continua si periodica,

este marginita.

f) Conform punctului e), limx→∞

F (x)

x= a. Cum G este periodica si neconstanta,

rezulta ca nu exista limx→∞

G(x). Prin urmare, F nu admite asimptota oblica spre ∞.

Nota. 1. Rezultatele de la punctele a) si b) sunt clasice. Punctul c) poate fi folositpentru determinarea primitivelor functiilor periodice. Punctele d), e) si f) fac obiectulunei probleme semnate de prof. univ. dr. Radu Gologan.

2. Folosind punctul e) al propozitiei demonstrate, obtinem ca:

limx→∞

1

x

Z x

0| sin t| dt = 1

π

Z π

0| sin t| dt = 2

π.

Prezentam cateva probleme, ca aplicatii directe ale propozitiei demonstrate.1. Determinati primitivele functiei f : R → R, f(x) = | sinx|.Solutie. f este periodica de perioada π. Primitiva lui f care se anuleaza ın 0 este

F0(x) = 1− cosx si atunci primitiva pe R este F (x) = 2hxπ

i+ 1− cos

x− π

hxπ

i.

Prin urmare, functia F (x) = 2n + 1 − (−1)n cosx, x ∈ [nπ; (n + 1)π), n ∈ Z, esteprimitiva lui f , care se anuleaza ın zero.

119

2. Se considera functiile f : R → R, f(x) =1

3 + cosxsi F : R → R, F (x) =Z x

0f(t) dt.

a) Sa se calculeze limx→∞

F (x).

b) Sa se arate ca graficul functiei F nu are asimptota catre +∞.(Variante BAC 2007, enunt partial)

Solutie. a) Evident ca f(x) ≥ 1

4, pentru orice x ∈ R, deci F (x) ≥ x

4, pentru

orice x ∈ R si de aici obtinem ca limx→∞

F (x) = ∞.

b) Este o consecinta imediata a punctului a) si a punctului f) din propozitie.Observatie. f este periodica de perioada T = 2π. Primitiva lui f pe [0; 2π], care

se anuleaza ın zero, este

F0(x) =

8>>>>><>>>>>:1√2arctg

tgx

2√2, x ∈ [0;π)

π

2√2, x = π

1√2arctg

tgx

2√2

+π√2, x ∈ (π; 2π]

si atunci primitiva lui f care se anuleaza ın zero este

F (x) =

8>>>>>>><>>>>>>>:

1√2arctg

tgx

2√2

+nπ√2, x ∈ [2nπ; (2n+ 1)π)

(2n+ 1)π

2√2

, x = (2n+ 1)π

1√2arctg

tgx

2√2

+(n+ 1)π√

2, x ∈ ((2n+ 1)π; (2n+ 2)π]

, n ∈ Z.

3. Se considera functiile f : R → R, f(x) = 3+x(1−x) si F : R → R, F (x) =Z x

0f(t) dt.

a) Sa se calculeze limx→∞

F (x).

b) Sa se arate ca exista a ∈ R, astel ıncat functia G : R → R, G(x) = F (x)− axsa fie periodica.

(Variante BAC 2007, enunt partial)Solutie. a) Deoarece f(x) ≥ 3, pentru orice x ∈ R, obtinem ca F (x) ≥ 3x, pentru

orice x ∈ R si de aici limx→∞

F (x) = ∞.

b) Cum f este periodica de perioada T = 1, din punctul d) al propozitiei demon-

strate rezulta ca exista a =

Z 1

0f(x) dx =

19

6cu proprietatea ceruta.

120

Nota. Primitiva functiei f pe [0; 1], care se anuleaza ın zero, este F0(x) = 3x +x2

2− x3

3. Prin urmare, primitiva functiei f care se anuleaza ın zero este: F (x) =

19

6[x] + 3(x− [x]) +

(x− [x])2

2− (x− [x])3

6.

4. Fie f : R → R o functie continua si periodica de perioada T > 0. Aratati ca

limn→∞

Z b

af(nx) dx =

b− a

T

Z T

0f(x) dx, pentru orice a, b ∈ R.

Solutie. Cu substitutia nx = t, obtinem

Z b

af(nx) dx =

1

n

Z nb

naf(t) dt = b ·

1

nb

Z nb

0f(t) dt− a · 1

na

Z na

0f(t) dt si, conform punctului e), urmeaza concluzia.

5. Fie f : [0;T ] → R o functie continua si g : R → R o functie continua siperiodica de perioada T > 0. Atunci

limn→∞

Z T

0f(x)g(nx) dx =

1

T

Z T

0f(x) dx

·Z T

0g(x) dx

(v. [2], [3]).

Solutie. Fie an =

Z T

0f(x)g(nx) dx, n ∈ N. Evident, an =

n−1Xk=1

Z (k+1)Tn

kTn

f(x)g(nx) dx.

Deoarece

Z (k+1)Tn

kTn

g(nx) dx =1

n

Z T

0g(t) dt, aplicand o teorema de medie, obtinem ca

an =n−1Xk=0

f(ξk)

Z (k+1)Tn

kTn

g(nx) dx =n−1Xk=0

1

nf(ξk) ·

Z T

0g(t) dt, ξk ∈

kT

n;(k + 1)T

n

si

cum limn→∞

n−1Xk=0

T

nf(ξk) =

Z T

0f(t) dt, rezulta concluzia problemei.

Ca aplicatie directa a acestui rezultat obtinem problema:

Fie f : R → (0;∞) o functie continua cu perioada 1. Atunci :

limn→∞

Z 1

0f(x)f(nx) dx =

Z 1

0f(x) dx

2

.

(Cristinel Mortici, etapa judeteana, 2003)

6. Fie f : R→R o functie continua si periodica. Daca F este o primitiva a lui f ,

aratati ca sirul an =nX

k=1

F (k)

k2este convergent daca si numai daca F este periodica.

(Florian Dumitrel, Concursul ”Nicolae Coculescu”, 2008)

Solutie. Fie T > 0 o perioada a functiei f . Exista a ∈ R astfel ıncat functia

G : R → R, G(x) = F (x)−ax sa fie periodica. Prin urmare, an =nX

k=1

G(k)

k2+a

nXk=1

1

k.

121

Cum G este marginita, fiind continua si periodica, exista M > 0, astfel ıncat|G(x)| ≤ M , pentru orice x ∈ R. Atunci 0 ≤ G(x) +M ≤ 2M , pentru orice x ∈ R.

Rezulta ca sirul bn =nX

k=1

G(k) +M

k2este convergent, fiind monoton si marginit.

Astfel, an =nX

k=1

G(k) +M

k2−M

nXk=1

1

k2+a

nXk=1

1

k. Prin urmare, sirul (an) este con-

vergent daca si numai daca a ·nX

k=1

1

keste convergent. Cum lim

n→∞

1 +

1

2+ ...+

1

n

=

∞, rezulta ca (an) converge daca si numai daca a = 0 si concluzia se impune.

7. Fie f : R → R o functie continua si periodica de perioada irationala, iar Fo primitiva a sa. Sa se demonstreze ca sirul (F (n) − n)n≥1 este convergent daca sinumai daca f(x) = 1, pentru orice x ∈ R.

(Florian Dumitrel, Concursul ”Nicolae Coculescu”, 2009)Solutie. Daca f(x) = 1, pentru orice x ∈ R, concluzia este evidenta. Reciproc,

fie T ∈ R\Q, T > 0, o perioada a functiei f . Exista a ∈ R astfel ıncat functiaG : R → R, G(x) = F (x)− ax sa fie periodica, de perioada T .

Atunci F (n)− n = G(n) + (a− 1)n si de aici sirul (F (n)− n)n≥1 este convergentnumai daca a = 1. Rezulta ca sirul (G(n))n≥1 are limita. Fie l ∈ R limita acestuisir si x ∈ R. Deoarece T ∈ R\Q, din teorema lui Kronecker rezulta ca multimean − mT | n,m ∈ N este densa ın R, deci exista sirurile (nk)k∈N si (mk)k∈N astfelıncat lim

k→∞(nk −mkT ) = x si (nk −mkT )k∈N este strict crescator.

Sirul (nk) este nemarginit; altfel, cum mkT = (mkT − nk) + nk, k ∈ N, ar rezultaca (mk) este marginit, adica B = nk − mkT | k ∈ N ar fi marginita, ceea ce arcontrazice monotonia sirului (nk −mkT )k∈N. Prin urmare (considerand, eventual, unsubsir al sirului (nk)), lim

k→∞nk = ∞.

Cum G(nk) = G(nk − mkT ) → G(x) si (G(nk))k∈N este un subsir al sirului(G(n))n∈N, rezulta ca G(x) = l. Deci F (x) = l + x si f(x) = 1, pentru orice x ∈ R.

Bibliografie

1. L. Niculescu - O metoda de calcul a primitivelor unei functii periodice, G.M.-8/1991,281-284.

2. D. Popescu, F. Popovici - O generalizare a lemei lui Riemann, Recreatii Mate-matice nr. 1/2002, 12-13.

3. D. Popescu - Asupra unui sir de integrale Riemann, Recreatii Matematice nr.2/2009, 98-100.

4. N. Pavelescu - Primitivele unor functii periodice, G.M.-10/2002, 371-379.

5. - Subiectele si variantele de bacalaureat, 2003 - 2007.

122

O problema de reprezentare

Marian TETIVA1

Abstract. This note is a very quick overview of some wellknown and simple representationproblems of a certain type; accessible explanations are given for the steps leading to their solutions.

Keywords: natural number, representation in base 2.

MSC 2000: 11A67.

Prin probleme de reprezentare ıntelegem aici acele probleme ın care se cere sa sedemonstreze ca orice numar de un anumit tip (ıntreg, rational, natural impar etc.)poate fi scris ıntr-o anumita forma. De exemplu: sa se arate ca orice numar natural(nenul) poate fi scris ca suma de puteri distincte ale lui 2. Recunoasteti acest enunt?(Probabil ca da.) Intr-o alta formulare, putem zice acelasi lucru astfel: oricare arfi numarul natural n ≥ 1, exista numerele naturale k ≥ 0 si a0, a1, . . . , ak ∈ 0, 1astfel ıncat n = a0 + a12 + · · · + ak2

k. De exemplu 1 = 1, 3 = 1 + 1 · 2, 22 =0+1 · 2+1 · 22 +0 · 23 +1 · 24 = 2+22 +24. (Acum ati recunoscut sigur enuntul carestabileste reprezentarea ın baza 2). Iata si alte exemple, din cele mai des ıntalnite.

Problema 1. Orice numar natural n poate fi reprezentat ıntr-o infinitate demoduri ın forma

±12 ± 22 ± . . .±m2,

pentru anumite numere naturale m si anumite alegeri ale semnelor plus si minus.Aceasta ınseamna ca trebuie sa dovedim ca, dat fiind numarul natural n, putem

gasi un numar natural m si o alegere a semnelor plus/minus astfel ıncat egalitateaanterioara sa fie adevarata (iar asta se ıntampla pentru o infinitate de numere naturalem; ın unele probleme se cere sa aratam unicitatea reprezentarii - de exemplu, oricenumar natural este ıntr-un singur mod suma unor puteri distincte ale lui doi, daca,desigur, facem abstractie de ordinea termenilor -, ın altele se cere sa se arate ca existao infinitate de reprezentari etc.). De exemplu avem:

0 = 12 − 22 − 32 + 42 − 52 + 62 + 72 − 82

si daca va gınditi bine de tot de unde provine aceasta egalitate neasteptata si (aparent)greu de gasit, aproape ca ati rezolvat problema 1. Sa mai mentionam aici ca, dim-potriva, avem urmatorul enunt:

Problema 2. Sa se arate ca un numar natural n poate fi reprezentat ın forma

n = ±1± 2± . . .± n

pentru o anume alegere a semnelor plus si minus daca si numai daca n da restul 0sau 1 la ımpartirea cu 4.

1Profesor, Colegiul National ”Gheorghe Rosca Codreanu”, Barlad

123

(Aceasta nu e tocmai o problema de reprezentare ın sensul precizat la ınceput, dare clar ca face parte din acelasi cerc de idei.)

Urmatoarea este, poate, mai putin cunoscuta, dar nu mai putin frumoasa; propusade Mihai Onucu Drimbe ın Gazeta Matematica [2]:

Problema 3. Sa se arate ca orice numar rational pozitiv poate fi reprezentatıntr-o infinitate de moduri ın forma

2

2

±1

· . . . ·m

2

±1

pentru anumite numere naturale m ≥ 2 si anumite alegeri ale semnelor plus si minus.(Enuntul original se refera la numere ıntregi pozitive, dar se poate si asa.)

Exista o larga varietate de asemenea probleme, pentru care, desigur, nu avemloc aici, iar ca sa va faceti o idee despre importanta lor ın matematica, gınditi-vadoar ca teorema fundamentala a aritmeticii - fundamentala, da? - este o teorema dereprezentare a oricarui numar natural ≥ 2 ca produs de numere prime.

Ajungem la problema despre care vom vorbi mai departe.

Problema 4. Orice numar ıntreg impar are o infinitate de reprezentari de forma

±1± 2± . . .± 2m

pentru anumite numere naturale m si anumite alegeri ale semnelor plus si minus.

De ea vrem sa ne ocupam ın aceasta nota. Nu ınainte de a va sfatui sa reflectatiasupra celor enuntate mai sus, sau sa le cautati solutiile prin carti; se gasesc ınnenumarate: [3], [4], [5]; totusi (repet, daca am mai spus-o - si am mai spus-o!),nu faceti asta ınainte de a munci singuri la ele: chiar daca nu veti gasi independentrezolvari complete, nimic nu poate ınlocui efortul personal, iar satisfactiile, cele reale,nu vin (orice ar spune unii) decat de pe urma acestuia.

Sa observam, mai ıntai, ca o reprezentare a unui numar natural ın aceasta formagenereaza o infinitate de reprezentari similare ale aceluiasi numar. E suficient saınlocuim pe 2m cu −2m + 2m+1 pentru a obtine, din n = ±1± 2± . . .± 2m,

n = ±1± 2± . . .± (−2m + 2m+1) = ±1± 2± . . .± 2m−1 ∓ 2m ± 2m+1,

adica (asa cum am anuntat) o alta reprezentare. O secunda de gındire si ne damseama ca egalitatea 2m = −2m + 2m+1 este, de fapt, un caz particular pentru

2m = −2m − 2m+1 − . . .− 2m+p−1 + 2m+p

(unde p ≥ 1 poate fi orice numar natural), deci reprezentarea n = ±1± 2± . . .± 2m

produce, cu aceasta formula, o infinitate de reprezentari:

n = ±1± 2± . . .± 2m−1 ∓ 2m ∓ 2m+1 ∓ . . .∓ 2m+p−1 ± 2m+p

124

(pentru fiecare p cate una). Astfel ca tot ce ne mai ramane de facut este sa dovedimca fiecare numar impar n (de ce impar? pentru ca este evident ca numerele pare nupot avea asemenea reprezentari) are macar o reprezentare cum cere enuntul. Desigur,putem rezolva problema doar pentru numerele ıntregi impare pozitive, caci schimbareatuturor semnelor ıntr-o reprezentare a lui n conduce la o reprezentare a lui −n. Cumobtinem deci o scriere ca (sa zicem)

233 = −1− 2 + 22 − 23 + 24 + 25 + 26 + 27?

Pornim de la 233 = 2 · 117− 1 si folosim scrierea

117 = −1 + 2− 22 + 23 + 24 + 25 + 26.

De unde am stiut-o pe aceasta? Evident, din 117 = 2 · 59− 1 si

59 = 1− 2 + 22 + 23 + 24 + 25;

pe care am aflat-o pornind de la

59 = 2 · 30− 1 = 22 · 15− 1 = −1 + 22 · (1 + 2 + 22 + 23) =

= −1 + 22 + 23 + 24 + 25 = 1− 2 + 22 + 23 + 24 + 25.

Acum e timpul pentru

Exercitiul 1. Aratati (prin inductie) ca orice numar natural impar admite oreprezentare de forma

±1± 2± . . .± 2m

(pentru un anumit numar natural m si o anumita alegere a semnelor plus si minus).Deduceti ca orice numar ıntreg poate fi reprezentat (ıntr-o infinitate de moduri) ınforma

±2p ± 2p+1 ± . . .± 2q

pentru anumite numere naturale p ≤ q si anumite alegeri ale semnelor plus si minus.Ipoteza de inductie se refera la posibilitatea reprezentarii ın forma ceruta a tuturor

numerelor mai mici decat, sa zicem, 2k − 1 si conduce la existenta unei reprezentaripentru 2k − 1. Pentru asta ıl scriem pe k ın forma 2s(2t − 1) si folosim identitateafundamentala mentionata si mai sus: 1 = −1 − 2 − . . . − 2u + 2u+1 etc. Aceasta esolutia pe care (cel putin eu) am ıntalnit-o cel mai des. Mi s-a parut ıntotdeauna preacomplicata (marturisesc ca nici nu am ınteles-o bine de prima data; de fapt, e usor deınteles, dar cınd vrei s-o explici cuiva te lovesti mereu de dificultati la alegerea lui usi asa mai departe, iar inductia nu face ca lucrurile sa fie mai simple: dar acesta e unfel de a spune ca n-ai patruns ın esenta problemei). De aceea am cautat mereu altavarianta (iar prin carti n-am gasit, chiar daca - nu ma ındoiesc - si cele ce urmeazatrebuie sa mai fi fost gındite de altcineva si, probabil exista si ın carti, altele decatcele cu care m-am ıntalnit eu).

Au trecut ani. Intr-o zi, explicand unui elev solutia de mai sus si vazand cat degreu accede la ıntelegerea ei (ın ciuda faptului ca eu, ıntre timp, chiar am priceput-o

125

si i-o aratam cu exemple si tot ce trebuie), m-am gandit: trebuie sa mai fie si altaposibilitate! Si singura idee care mi-a venit a fost sa ma leg de reprezentarea ın bazadoi.

Intr-adevar, privind numarul 233 scris ca suma de puteri ale lui 2:

233 = 1 + 23 + 25 + 26 + 27

mi-am dat seama imediat cum trebuie sa fac: era suficient sa-l ınlocuiesc aici pe 1 cu−1− 2 + 22 si pe 23 cu −23 + 24, ca sa obtin scrierea lui 233 ın forma cautata. (Erabanal!) Iata ınca un exemplu:

273 = 1 + 24 + 28 = −1− 2− 22 + 23 − 24 − 25 − 26 + 27 + 28.

E atat de banal, ıncat, practic, intra ın categoria ”demonstratii fara cuvinte”! Noi vompropune, totusi, cititorului interesat sa formalizeze aceasta demonstratie, chiar pentrusituatia generala prezentata ın exercitiul anterior (e un fel de a spune: ”generala”;propriu-zis, nu avem o generalizare efectiva, ci doar una de circumstanta).

Exercitiul 2. Fie n un ıntreg pozitiv, scris n = 2n1 + 2n2 + . . . + 2nk (unden1 < n2 < . . . < nk) ca suma de puteri distincte ale lui 2. Sa se arate ca exista p ≤ qnumere naturale si exista o alegere a semnelor plus si minus astfel ıncat

n = ±2p ± 2p+1 ± . . .± 2q.

Deduceti ca acest enunt este valabil pentru orice ıntreg nenul n (de ce nu si pentruzero?).

(Desigur, p = n1 si q = nk, dar pentru q avem o infinitate de posibilitati; parteaa doua rezulta, cum am mai spus, doar prin schimbarea semnelor).

Cu asta s-ar parea ca am terminat, dar nu e asa:

Propozitie. Fie n si m numere naturale nenule, cu n impar si 1+2+. . .+2m > n.Atunci exista o alegere a semnelor plus si minus astfel ıncat

n = ±1± 2± . . .± 2m.

Demonstratie. Conform ipotezei, numarul 1+2+ . . .+2m−n este par si pozitiv.

De aceea1

2(1 + 2 + · · ·+ 2m − n) este un ıntreg pozitiv, prin urmare el poate fi scris

ca suma de puteri ale lui 2:

1

2(1 + 2 + . . .+ 2m − n) = 2n1 + 2n2 + . . .+ 2nk ,

cu n1 < n2 < · · · < nk numere naturale. Deoarece1

2(1 + 2 + . . . + 2m − n) <

1

2(1+2+ . . .+2m) =

1

2(2m+1−1) < 2m, exponentii n1, n2, . . . , nk sunt din multimea

0, 1, . . . ,m. Atunci n = 1+ 2+ . . .+ 2m − 2(2n1 + 2n2 + . . .+ 2nk) este scrierea pecare o cautam.

126

De exemplu, sa spunem ca vrem sa gasim o exprimare a lui 233 ca suma de forma

±1 ± 2 ± . . .± 210. Pentru asta calculam1

2(1 + 2 + . . .+ 210 − 233) = 907 si scriem

acest numar ca suma de puteri ale lui 2 907 = 1+ 2+23 +27 +28 +29. Avem atunci233 = 1+ 2+ . . .+210 − 2(1+ 2+ 23 +27 +28 +29) == −1− 2+ 22 − 23 +24 +25 +26 − 27 − 28 − 29 + 210.

E limpede ca lumina zilei ca aceasta demonstratie conduce la un rezultat ge-neral. Chiar asa si este: M.O. Drimbe a descoperit aceasta teorema (generala) si apublicat-o [1] ınca din 1983! Aceasta ıntalnire peste ani mie ımi reveleaza ınca o data(daca mai era nevoie) nesfarsita frumusete a matematicii.

Bibliografie

1. M.O. Drimbe - O problema de reprezentare a numerelor ıntregi, GM 10-11/1983,382-383.

2. M.O. Drimbe - Problema C:371, GM 1/1984, p. 42.

3. A. Engel - Probleme de matematica - strategii de rezolvare, Editura Gil, Zalau, 2006.

4. L. Panaitopol, Alexandru Gica - Probleme de aritmetica si teoria numerelor. Ideisi metode de rezolvare, Editura Gil, Zalau, 2006.

5. W. Sierpinski - Elementary Theory of Numbers, (Translated from Polish by A.Hulanicki), Warszawa, 1964.

Raspuns la ”recreatia” de la pag. 112.

In reprezentarea lor ıntr-o baza convenabil aleasa, numerele indicate folosesc numaicifra 1:

1 = 1 (ın orice baza b ≥ 2)3 = 11213 = 111385 = 11114

781 = 11 11159 331 = 111 1116

137 257 = 1 111 11172 396 745 = 11 111 111848 427 561 = 111 111 1119

127

Un episod ineditdin matematica romaneasca interbelica

Temistocle BIRSAN 1, Dan TIBA2

Unele publicatii matematice dintre cele doua razboaie mondiale poarta siglaInstitutul Matematic Roman cu deviza pitagorica Mundum Regunt Numerisi anul de ınfiintare 1928.

Sub egida acestei institutii academice s-a desfasurat o activitate variata, sumandun numar important de realizari. Institutul Matematic Roman si-a continuatactivitatea chiar si ın primii ani ai celui de-Al Doilea Razboi Mondial.

Directorul IMR, si ın acelasi timp animatorul principal al activitatilor sale, a fostRodolphe Neculai Raclis (care deseori semna Rodolphe Raclis sau NeculaiRaclis). Adresa de corespondenta indicata era str. Porumbaru 70, Bucuresti, maitarziu Aleea Vulpache 21, devenita ulterior str. Ankara 5, adica acasa la R.N. Raclis.

In perioada la care ne referim, scoala matematica romaneasca se afirmase sicapatase deja o recunoastere si ın afara granitelor tarii. Legea ınvatamantului din 1928a dat o lovitura ınvatamantului matematic prin desfiintarea sectiei reale prevazuta delegea Haret (1898) pentru cursul superior al liceelor. O consecinta imediata a aces-tei masuri a fost scaderea dramatica a cunostintelor de matematica ale elevilor dinscolile secundare. In acest context, alaturi de alte societati si publicatii ale timpului,IMR a jucat un rol important ın sustinerea ınvatamantului matematic romanesc sidezvoltarea aptitudinilor si gustului pentru matematica.

IMR nu a avut ca scop desfasurarea unei activitati sistematice de cercetare stiinti-fica, asa cum se face astazi ın institutele cu nume similar. Si-a asumat atributeleunei case editoriale publicand trei reviste, cursuri universitare si manuale scolare, darmembrii sai au fost totodata realizatori si redactori ai acestora sau, ın multe cazuri,autori ai celor din urma.

Revista Universitara Matematica este, cronologic, prima revista publicatade IMR si se adreseaza candidatilor la licenta si elevilor scolilor tehnice superioare.Au aparut doar doua volume: vol.I din 1929, format din patru fascicule, si vol.II din1937, o singura fascicula.

Cea de-a patra fascicula din 1929 este dedicata ın ıntregime memoriei luiTraian Lalescu, ilustrul nostru matematician disparut prematur ın acel an, si cu-prinde: un portret al acestuia, scrisorile de condoleante primite de la Emile Picardsi Vito Volterra, precum si cuvintele de doliu trimise de E. Pangrati, Gh. Titeica,D. Pompeiu, V. Valcovici s.a.

In celelalte fascicule, ın cea mai mare parte a lor, sunt prezentate chestiuni deexamen: enunturi si solutii ale subiectelor date la diferite examene de la Facultatilede stiinte din toata tara sau Scoala Politehnica din Bucuresti, iar ın fascicula din 1937

1Prof. dr., Univ. Tehnica ”Gh. Asachi” Iasi2Cercet. st., Inst. de Matematica al Academiei ”S. Stoilow”, Bucuresti

128

sunt publicate si subiecte date ın anii anteriori la examenul de capacitate, urmate apoide solutii detaliate; fapt notabil, se publica si componenta comisiei de examinare,totalul candidatilor si numarul si numele celor admisi. Sunt interesante unele datestatistice ın aceasta privinta: la Facultatea de Stiinte din Bucuresti, la examenulde geometrie analitica din iunie 1927 cu profesorul Gheorghe Titeica, asistat de DanBarbilian, doar 18 candidati au fost admisi din 71 ınscrisi; la Facultatea de Stiintedin Iasi, ın iunie 1928, un singur candidat a fost admis la examenul de astronomie(Gheorghe Gheorghiev, cunoscutul geometru de mai tarziu).

Sunt publicate un numar mic de note matematice, dar multe materiale de infor-mare: lista lucrarilor de licenta de la Facultatea de Stiinte din Cluj, specialitateamatematica, din anii 1924-1928 (vol.I, nr.2), un raport al lui Anton Davidoglu asupradesfasurarii examenului de capacitate ın care se fac recomandari privind recrutareacorpului profesoral secundar (vol.II), subiectele discutate ın sedintele Cercului Revis-tei Universitare (la Bucuresti) ın ianuarie 1929 etc. (unul dintre conferentiari fiindacademicianul (de mai tarziu) Nicolae Teodorescu, cu subiectul Introducere ın studiulecuatiilor integrale de tip Volterra). Dintre articolele omagiale mentionam Cincuante-narul lui Emile Picard scris de Traian Lalescu (vol.1, nr.2, 1929) si Cateva cuvinte ınlegatura cu lectiile D. Paul Montel la Universitatea din Cluj de Gheorghe Calugareanu(vol.1, nr.3, 1929).

Mai cunoscuta este revista Numerus (1935 -1943), cu prima aparitie la 15 mai1935, cu un continut si un public vizat mentionate ın subtitlu: Revista de matematicielementare pentru ınvatamantul secundar, normal, profesional si militar. Apareaproximativ lunar ın fascicule, numite caiete; zece caiete formeaza un volum, iarcaietele sunt numerotate ın continuare. Secretari de redactie si cu un aport impor-tant ın aparitia revistei au fost Cezar Cosnita, Virgil Claudian, Traian Angelescus.a.

Revista a publicat un numar impresionant de probleme propuse, ordonate pe ma-terii si clase, de subiecte date la bacalaureat ın liceele de pe tot cuprinsul tarii, ca side subiecte propuse la examenele de capacitate, la probele de matematica elementara.

IMR a organizat concursuri anuale pentru rezolvitorii de probleme propuse ınNumerus. Aceste concursuri au purtat, succesiv, numele unor matematicieni romanisau francezi: Spiru Haret, Traian Lalescu, Edouard Goursat, Henri Poincare,Gheorghe Titeica, Nicolae Botea (colaborator important al IMR, decedat prematur),Dimitrie Pompeiu, Andrei Ioachimescu. In anii de dinainte de razboi, erau atrasi multiparticipanti din toata tara, care erau toti listati ın revista, iar premiantii (premiilepurtau numele concursului) erau ınscrisi ın liste speciale si primeau drept recompensapublicatii ale IMR. Iata si doua informatii care ne ajuta sa retraim momentul istoric:”ın septembrie 1939 s-au prezentat ın toata tara 3521 candidati si au fost declaratibacalaureati 2108”; ”din cauza concentrarii directorului, secretarilor de redactie si co-laboratorilor, rezultatul concursului George Titeica la care au luat parte 615 elevi sieleve se publica ın Caietul 59” (octombrie 1940).

Mai apar ın fasciculele revistei unele note matematice si materiale variate depopularizare si umanizare a matematicii: portrete ale unor ınaintasi celebri, arti-cole aniversare, aforisme, fantezii matematice. Este de remarcat excelenta calitate

129

a portretelor lui Euclid, Arhimede, Descartes, Newton, Euler, Poincare, precum siS. Haret, Gh. Titeica, D. Pompeiu, N. Botea, A. Angelescu etc., care erau deseoriınsotite de biografia si prezentarea operei lor. Sunt evocate figuri de mari matemati-cieni ca R. Descartes (dupa o conferinta tinuta de Gh. Titeica la Institut Francaisdes Hautes Etudes en Roumanie, 1937), S. Haret (dupa discursul de receptie al luiGh. Titeica, la Academia Romana, 1914), I. Newton si L. Euler (de I. Ionescu),T. Lalescu, Arhimede, H. Poincare, Gh. Titeica (de Dan Barbilian), P. Fermat, G.Galilei (traduceri).

In anii razboiului au aparut ın revistaNumerus unele accente politizante nepotri-vite, care nu aveau legatura cu matematica, obiectul revistei.

Dintre articolele deosebite publicate mentionam lucrarile Rationamentul aritmeticsi Adevarata valoare de Dimitrie Pompeiu, articolele Despre ipoteza Fermat siDespre numerele pitagorice ale lui Gabriel Sudan, Problema celor patru culori de IonArmean, descrierea Congresului Interbalcanic de la Bucuresti din septembrie 1937 saua sarbatoririi lui Paul Montel ın 1938 la Universitatea din Cluj.

Revista ın limba franceza Annales Roumaines de Mathematiques, de fapto colectie (biblioteca) de carti sau memorii consacrate din literatura matematica siavand autori romani, a publicat sau republicat sub forma de fascicule (numite caiete)urmatoarele: La geometrie du triangle de Traian Lalescu (caietul 1), Sur les couplestransformables de Alexandru Pantazi (caietul 2), La derivee areolaire de NicolaeTeodorescu (caietul 3), Coordonnees barycentriques de Cezar Cosnita (caietul 4)si Recherches sur le grand theoreme de Fermat de R.N. Raclis (caietul 5). CartileLa geometrie du triangle si Coordonnees barycentriques, care aparusera anterior frag-mentat ın caietele revistei Numerus, au fost copublicate ın Annales Roumaines deMathematiques si de editura Vuibert, Paris, ın anii 1938, 1941; prima este deschisacu o scrisoare a lui E. Picard si o prefata semnata de Gh. Titeica, iar cea de-a douaeste prefatata de D. Pompeiu. In 1958 a aparut si editia ın limba romana a cartii luiT. Lalescu, Geometria triunghiului, care ne-a ajutat si ne-a ıncantat pe multi dintrenoi. Cu o intentie asemanatoare, ın Numerus, vol. 4-6, a fost publicata traducereaprimelor trei carti ale Elementelor lui Euclid; nu s-a continuat, deoarece ıntre timpa aparut traducerea integrala, ın Biblioteca Gazetei Matematice.

O alta componenta importanta a activitatii IMR este publicarea de cursuri uni-versitare si manuale scolare grupate respectiv ın Biblioteca Universitara si ColectiaNumerus. In centrul acestei activitati se afla R.N. Raclis, care a editat, reeditat,revizuit sau alcatuit dupa note de curs un numar ınsemnat de tratate si cursuriuniversitare profesate la Facultatea de Stiinte sau Scoala Politehnica din Bucuresti:Curs de geometrie analitica de Gheorghe Titeica, Curs de analiza infinitezimalade Anton Davidoglu, Probleme de geometrie analitica de Dan Barbilian, Curs degeometrie descriptiva de Ermil Pangrati si Gheorghe Nichifor, Curs de algebrasuperioara de R. Raclis si multe altele.

In Colectia Numerus au aparut, ın mai multe editii, manualele: Geometrie deR. Raclis, V. Claudian si C. Cosnita (pentru clasele de liceu: 2,3,4,5,6), Arit-metica deR. Raclis si Traian Angelescu (pentru clasele: 1,2,3), Probleme deslegatede examen de R. Raclis, V. Claudian si C. Cosnita s.a.

130

R. Raclis a editat o parte importanta a operei didactice a lui Traian Lalescu ınpublicatiile IMR. In prefata la Culegere de probleme de geometrie descriptiva, editiaa II-a, 1935, el spune: La cinstea de a fi fost elevul lui Traian Lalescu si colaboratorullui, la Facultatea de Stiinte din Bucuresti, ın ultimii lui patru ani, se adauga cinsteade a mi se ıncredinta, ın mod exclusiv, publicarea tuturor lucrarilor ilustrului meuMaestru.

Mentionam si faptul ca IMR a instituit Medalia Traian Lalescu pentru come-morarea la 15 iunie 1939 a zece ani de la moartea acestuia; medalia era decernataelevilor bacalaureati care sumau un punctaj ridicat la concursurile anuale ale revisteiNumerus, dar si profesorilor ce au pregatit un numar mare de elevi premiati la acesteconcursuri. In fine, a existat si un Premiu Traian Lalescu ce se acorda celor maidistinsi licentiati ın stiinte matematice sau ingineri diplomati.

Prin publicistica bogata si variata, concursuri, cercuri, congrese, aniversari sicomemorari, premii si medalii, cat si prin atragerea colaborararii unor ilustri mate-maticieni, din tara si strainatate, Institutul Matematic Roman a avut o contributieremarcabila la ridicarea nivelului ınvatamantului romanesc, ıntreaga sa activitateputand fi un model de urmat chiar si astazi.

Bibliografie

1. - Annales Roumaines de Mathematiques -colectie.

2. - Revista Universitara Matematica, vol.I (1929) si vol.II (1937).

3. - Revista Numerus-colectie.

4. G.St. Andonie - Istoria matematicii ın Romania, vol.I (1965), pp. 365-390; vol.II(1967), pp. 43-49, Ed. Stiintifica, Bucuresti.

5. N. Mihaileanu - Revistele de matematici elementare (pp.181-187, pp.222-223,p.225), Ed. Gil, Zalau, 1995.

6. *** - Traian Lalescu, un nume peste ani, Curtea Veche, Bucuresti, 2007.

LAMPAS UTILITATIS, prima candela, arata ca nu putem ımpartasi mate-matica multimii oamenilor decat daca ne oprim mai ıntai asupra utilitatii ei.

LAMPAS DECORIS, candela frumusetii; adevaratul succes ın transmitereacunostintelor de matematica este posibil numai daca stim ca matematica este totatat de frumoasa pe cat este de utila.

LAMPAS IMAGINATIONES, candela a treia; ın matematica, niciodata nus-a facut o descoperire fara impulsul imaginatiei, care deschide calea pe care logicas-o urmeze.

LAMPAS POESIS, candela poeziei. Matematicile – spune Ion Barbu/Dan Bar-bilian - pun ın joc puteri sufletesti, nu mult diferite de cele solicitate de poezie si arta,iar cei care n-au simtit poezia matematicii ar fi mai bine sa n-o mai profeseze.

LAMPAS MISTERII, candela a cincea, dezvaluie lumii unul din farmecelestiintei: slujitorii ei au fost confruntati, ın fiece epoca, cu mari probleme, carora,odata rezolvate, le-au luat locul altele.

131

SEBASTIAN ANITA

(1941-2010)

Este aproape o cutezanta sa speri ca vei gasicuvintele potrivite pentru a istorisi cariera depeste patru decenii a profesorului de matematicaSebastian Anita. Vom ıncerca ınsa ın randurileurmatoare sa prezentam, atat cat ne sta ın putinta,realizarile acestui eminent discipol al scolii dematematica de la Iasi, la randul sau creator depasiuni pentru matematica ın randul celor care auavut sansa de a-i fi fost elevi sau de a-i fi cititcartile scrise cu har.

S-a nascut la 3 septembrie 1941, ın satulBasarabi, comuna Preutesti, judetul Suceava, ınfamilia lui Sevastian si a Anicai Anita, gospodaride frunte ai satului, parinti care au sadit dragosteade carte ın sufletele celor sase copii ai lor. Aabsolvit, ın anul 1959, Liceul ”Nicu Gane” dinFalticeni, scoala cu renume a ınvatamantului mol-dav. In anul 1964 a obtinut licenta ın matematica la Facultatea de matematica aUniversitatii ”Alexandru Ioan Cuza” din Iasi si, ın acelasi an, si-a ınceput activitateala Grupul Scolar de Chimie nr. 3 din Iasi. A fost apoi transferat, ın anul 1967, laLiceul Teoretic ”Al.I. Cuza”, scoala tanara, dar cu un prestigios corp profesoral sicu elevi avand dorinta de ınvatatura. In acest timp, profesorul Sebastian Anita aparcurs cu siguranta si reusita deplina examenele carierei didactice, obtinand gradulI ın anul 1977.

Din anul 1982 a fost transferat la Liceul National (astazi Colegiul ”National” Iasi).In momentul plecarii sale de la liceul Cuza s-a ıntamplat un lucru despre care nu avemcunostinta ca ar avea precedent: o ıntreaga clasa si-a cerut transferul, nedorind sa sedesparta de profesorul iubit.

Apreciat unanim pentru calitatea actului didactic practicat, Sebastian Anita aavut ca dominanta a personalitatii sale permanenta dorinta de autodepasire. Ca oanecdota, ın anul 2001, ın prag de pensionare, a participat activ, ımpreuna cu maitineri colegi, la un stagiu de formare pentru profesorii examinatori, ımpartasind dinexperienta sa, dar ın acelasi timp urmarind constiincios cursurile predate.

Drumul spre ınalta performanta a elevilor sai l-a pregatit printr-o asidua munca deautoperfectionare, precum si printr-o permanenta colaborare cu elevii. A fost un pro-motor al pregatirii centralizate a tinerilor cu aptitudini pentru matematica, sustinandlectii ın cadrul cursurilor saptamanale adresate acestora ın anii ’70. Recunoasterea

132

autoritatii ın domeniu este atestata de calitatea de membru al Comisiei Centralea Olimpiadei de Matematica ın anii 1970-1983, 1990, 1994, 1996. In 2003-2005, acontribuit la nasterea si la bunul mers al celui mai prestigios concurs iesean, Concur-sul National ”Alexandru Myller”. Elevii profesorului Sebastian Anita au avut, de-alungul timpului, realizari deosebite la olimpiadele si concursurile scolare. Marturiestau cele 20 de premii si mentiuni obtinute de acestia la etapa finala a Olimpiadei,precum si nenumaratele premii de la diverse concursuri interjudetene sau nationale.Performante exceptionale au avut Sebastian Anita (fiul profesorului), multiplu pre-miat al Olimpiadei de Matematica si calificat ın loturile nationale ın perioada 1980-1983, precum si Marius Pachitariu, care a obtinut medalia de aur la Balcaniada deMatematica a Juniorilor ın anul 2002.

Corolar al consacrarii profesionale a profesorului Sebastian Anita sunt si nume-roasele lucrari de specialitate publicate. Multi dintre performerii anilor ’80 au ınvatatsi au aprofundat geometria studiind Bazele rationamentului geometric sau Planulsi spatiul euclidian, aparute la Editura Academiei ın 1983, respectiv 1986, carti dereferinta ın domeniul geometriei elementare, la care profesorul Sebastian Anitaeste coautor. In anii ’90 au aparut, tot ın colaborare, Competenta si performanta ıngeometrie, la Paralela 45, sau Siruri recurente ın liceu, la GIL.

Pentru rezultatele de exceptie ale muncii sale, a fost distins cu titluri, diplome,decoratii, scrisori de multumire si de apreciere; amintim doar Ordinul National”Pentru Merit” ın grad de Cavaler, acordat de presedintele Romaniei ın anul 2000.

Intregirea portretului de dascal al profesorului Sebastian Anita impune, cumeste firesc, cuvinte respectuoase pentru familia sa: sotia Rodica Anita, profesor dematematica, si cei doi copii, Sebastian Anita, olimpic redutabil, astazi distins cadruuniversitar si Alice Anita, profesor de matematica la aceeasi scoala unde a predat tatalei, Colegiul ”National” Iasi. Dealtfel, acest liceu a constituit pentru profesorul Sebas-tian Anita centrul de interes ın cea mai rodnica perioada a sa ca dascal. In momentulpensionarii, la data de 1 septembrie 2005, gandurile sale despre anii ındelungati ıncare a cunsocut suprema ımplinire sunt o marturie plina de caldura sufleteasca pentruSCOALA, careia i s-a dedicat neconditionat si definitoriu: Cei mai minunati ani aivietii mele au fost anii de ınvatamant, mai ales cand i-am petrecut ın acest prestigioslacas de cultura, care este Colegiul ”National”. Aici au fost anii de munca asidua, deımpliniri, de idealuri concretizate ın rezultatele obtinute la concursuri si olimpiade.Mai presus ınsa de cunostintele complexe si profunde predate, s-a situat acel procesde formare a unor caractere demne si oneste. Cu acest prilej, ımi iau ramas bun dela acest Colegiu si rog pe fostii colegi sa nu uite ca destinul lor este de a se ınalta totmai sus pe culmile cunoasterii si ale culturii, ca lamentatiile, conflictele, ındoielilevor fi depasite de cei care nu sunt infirmi spiritual, ca Noul este vesnica tinerete.

In ziua de 27 februarie 2010, Sebastian Anita a trecut ın lumea vesniciei, ınsotitde regretele sincere ale fostilor sai elevi si colegi. In pleiada generatiilor de profesoriai Nationalului si ai Iasului, Sebastian Anita ramane un nume de referinta si un reperal ascensiunii pe drumul cunoasterii si al consacrarii depline.

Prof. Vasile BUZNOSUColegiul ”National” Iasi

133

Concursul de matematica ”Al. Myller”Editia a VIII-a, Iasi, 8 aprilie 2010

Clasa a VII-a

1. Fie x si y doua numere reale cu proprietatea ca x+y ≥ 2. Aratati ca x2+y2 ≥x+ y. Cand are loc egalitatea?

2. Sa se determine numerele prime a, b, c si A, stiind ca A = a4 + b4 + c4 − 3.

3. Fie a1, a2, . . . , a2010 ∈ −1, 0, 1, astfel ıncat a1 + a2 + . . .+ a2010 = 1.a) Calculati a31 + a32 + . . .+ a32010.b) Determinati numarul de valori distincte pe care le poate lua suma S = a21 +

a22 + . . .+ a22010.

4. Se considera triunghiul ABC ın care AB = AC si fie M mijlocul segmentuluiBC. Punctele D si E sunt picioarele perpendicularelor din M pe dreptele AC, respec-tiv AB, iar H este mijlocul segmentului [DE]. Fie punctele P,Q ∈ BC astfel ıncatMQ = MP = MD si P ∈ (BM). Aratati ca punctul H este ortocentrul triunghiuluiAPQ.

Clasa a VIII-a

1. Fie x si y doua numere reale cu proprietatea ca x+y ≥ 2. Aratati ca x3+y3 ≥x+ y. Pentru care valori ale numerelor x si y are loc egalitatea?

2. Piramida triunghiulara regulata VABC cu varful V are latura bazei de 6 cm siınaltimea de

√6cm. Planul perpendicular ın C pe dreapta AC intersecteaza dreapta

AV ın punctul P . Calculati distanta de la P la planul (V BC).

3. Fie multimea A = 1, 2, 3, ..., 37. Aratati ca, oricum am considera 13 elemente

distincte din A, exista doua printre acestea, a si b, cu proprietatea ca4

5<

a

b<

5

4.

4. Fie p ≥ 2 un numar prim. Sa se arate ca orice numar natural nenul admite unmultiplu de forma p2a − pa, unde a ∈ N∗.

Clasa a IX-a

1. Sa se arate ca, pentru orice numar real x ≥ 25

2, intervalul [x, 2x] contine trei

ıntregi distincti ın progresie geometrica.Mihai Baluna

2. Se considera o multime A cu n ≥ 3 elemente si 2n−1 submultimi distincte alelui A. Se stie ca oricare trei dintre aceste submultimi au intersectia nevida. Sa searate ca intersectia tuturor submultimilor date este nevida.

134

3. Fie n ∈ N, n ≥ 2 si o multime x1, x2, . . . , xn de numere reale cu proprietatea

|xi − xj | ≥ 1, ∀1 ≤ i < j ≤ n. Sa se arate ca x21 + x2

2 + . . .+ x2n ≥ n(n− 1)(n+ 1)

12.

4. Fie M ∈ (BC), N ∈ (AC), P ∈ (AB) picioarele a trei ceviene concurente ıntriunghiul ABC. Paralela prin N la AB taie dreapta PM ın E, iar paralela prin M laAB taie dreapta PN ın F . Sa se arate ca dreptele MN , EF si PC sunt concurente.

Gabriel Popa si Paul Georgescu

Clasa a X-a

1. Daca x, y ∈ (1, 2], demonstrati ca logx(3y − 2) + logy(3x− 2) ≥ 4.Radu Sava

2. Daca z este un numar complex de modul 1, aratati ca√3 ≤ |1+z|+|1−z+z2| ≤

13

4. Cand se realizeaza egalitatile?

3. Determinati numerele reale x care pot fi scrise sub forma

x =1

a1a2 . . . an+

a1a2a3 . . . an

+a2

a3a4 . . . an+ . . .+

an−2

an−1an+

an−1

an,

unde n, a1, a2, . . . , an sunt numere naturale nenule cu a1 < a2 < . . . < an.Gheorghe Iurea

4. Fie P multimea punctelor planului, iar f : P → P o functie care duce drepteleın drepte si astfel ıncat orice patrulater convex este dus ıntr-un patrulater convex deacelasi perimetru. Demonstrati ca f este izometrie.

Paul Georgescu si Gabriel Popa

Clasa a XI-a

1. Dati un exemplu de functie f : (0,∞) → (0,∞) cu proprietatea ca

f(f(f(f(x)))) =41x+ 40

40x+ 41, pentru orice x > 0.

2. Fie A si B doua matrice din Mn(C) cu proprietatea ca AB este nilpotenta siABA = On. Aratati ca exista o matrice C ∈ Mn(C) astfel ıncat ABAC = On siAC = On.

3. Determinati functiile continue f : R → R cu proprietatea ca 2f(x + 1) =f(x) + 4f(2x), oricare ar fi x ∈ R.

Dinu Serbanescu

4. Fie f : (0,∞) → R o functie avand proprietatea lui Darboux, astfel ıncatpentru orice a ∈ R, ecuatia f(x) = a are un numar finit de solutii. Aratati ca functiaf are limita la infinit.

Dinu Serbanescu

135

Clasa a XII-a

1. Fie f : [a, b] → R o functie integrabila cu proprietatea ca pentru orice x ∈ [a, b),

exista y ∈ (x, b) astfel ıncat

Z y

xf(t)dt > 0. Aratati ca

Z b

af(x)dx > 0.

Calin Popescu

2. Fie f : [0,∞) → R o functie crescatoare. Aratati caZ x

0(2t− x)f(t)dt ≥ 0, oricare ar fi x ∈ R.

Marian Andronache

3. Fie n un numar natural, n ≥ 2, si G ⊂ M3(C) un grup cu n elemente ınraport cu ınmultirea matricelor, cu proprietatea ca tr(AB) = tr(A)tr(B), oricare arfi A,B ∈ G. Aratati ca:

a) elementul neutru al lui G este diferit de I3;b) G este izomorf cu grupul (Un, ·) al radacinilor complexe de ordinul n ale unitatii.

Dinu Serbanescu

4. Fie A un inel finit comutativ cu cel putin trei elemente si a =Xx∈A

x7, b =Xx∈A

x8.

Aratati ca cel putin unul dintre elementele a si b este neinversabil.Marian Andronache

Raspuns la exercitiul de la pag. 106.

Numarul are sase cifre si se termina ın s. Evident, s ∈ 0, 1, 4, 5, 6, 9.Din prima scadere care este prezenta ın algoritmul de extragere a radacinii patrate,

rezulta ca prima cifra a radacinii poate fi 0, 1, 2 sau 3. Cum 0, 1 si 2 se elimina usor(ın aceste cazuri patratul radacinii ar avea cel mult cinci cifre), deducem ca primacifra a radacinii este 3.

Tinand seama de algoritmul extragerii radacinii patrate, a doua scadere implicaegalitatea ∗ ∗ s = 6s · s. Cu aceasta, constatam ca s /∈ 0, 1, 4, 9, deci s ∈ 5, 6.

Daca s = 5 si avem ın vedere a treia scadere, obtinem ∗ ∗ ∗s = 70t · t, de undet = 5. Gasim ca radacina patrata numarul 355, care, ınsa, nu este solutie: patratul3552 = 126025 nu are cifre distincte (a = i = 5).

Analog, daca s = 6, vom avea ∗ ∗ ∗6 = 72u · u si, deci, u = 4 sau u = 6.Asadar, gasim numerele 364 si 366. Cum 3662 = 133956 nu are cifre distincte, iar3642 = 132496 are cifre distincte, conchidem ca numarul 364 este solutia problemei,iar Raclis ≡ 132496.

136

Concursul de matematica ”Florica T. Campan”Editia a X-a, Iasi, 2010

Se aude (mai des decat ar intra ın umbra unor posibile justificari) ca matematicienii ar fireci, ne-iubitori, severi si inumani. Prostia este atat de mare ıncat nici nu merita combatuta;moare sufocata de propria osanza.

Concursurile ”Florica T. Campan” au ımplinit 10 ani. Nu numaram editii: judetene,interjudetene, adresate si celor foarte mici etc. Fiecare exprima prinos de afectiuni: aleFloricai pentru matematica si pentru cei ce o ındragesc, ale propunatorilor si corectorilor,ale sponsorilor, ale participantilor.

10 ani de afectiune reciproca! Doar o mama care si-a crescut odrasla are o imaginesugestiva a unei astfel de perioade! Pe langa satisfactii apar mereu pozne, boli, influentenegative, temeri de evolutii strambe. Parintii concursurilor ”FTC” se arata ıncrezatori ınodrasla: merge pe drum drept si bun.

In lumea asta exista si zmei rai, mai urati decat cei din povesti. Nu le pasa ca lumea sauscoala se vestejesc; socotesc important doar ce curge spre buzunare proprii. Copii! Nu vatemeti de Bau-Bau! Cititi mai jos cum ultimul FTC pregateste palosuri si buzdugane contrazmeilor inconstienti. Nu va grabiti sa deveniti discipoli si slugi ale zmeilor. Intariti-va acum,ascutiti palose si pregatiti-va de noi concursuri. Poate va veni o vreme cand veti fi destul detari sa ridicati buzdugane contra unor rele.

Prof.dr. Dan Branzei

Etapa judeteana, 20 februarie 2010

Clasa a IV-a

1. Suma a patru numere este 2021. Daca ıl ımpartim pe primul la al doilea, peal doilea la al treilea si pe al treilea la al patrulea obtinem, de fiecare data, catul 2 sirestul 1. Aflati numerele.

Doina Lacramioara Nechifor

2. Catalin, Lucian si Gabriel au ımpreuna 29 de portocale. Aflati cate portocaleare fiecare dintre ei, stiind ca Lucian are de trei ori mai multe portocale decat Cataliniar Gabriel are mai multe portocale decat Catalin si mai putine decat Lucian.

Catalin Budeanu

3. In figura alaturata, ın locul literelor trebuie puse toate cifrele nenule, fiecare

1

2

3

a

b

c d

e

f

cate o singura data; cifrele 1, 2 si 3 sunt deja fixate.a) Aflati suma celor patru numere de pe fiecare latura a

triunghiului si justificati cum ati obtinut-o, stiind ca suma depe fiecare latura este aceeasi.

b) Inlocuiti literele cu cifre pentru a obtine o astfel de suma.c) Cate astfel de distributii ale numerelor 4,5,6,7,8,9 pe

aceasta figura exista, tinand cont ca suma numerelor de pe fiecare latura sa fie aceeasi?

4. (Problema suplimentara) Un patrat de hartie are latura de 7 cm. Desenati cumse pot decupa din acest patrat patru triunghiuri identice si un patrat. (Gasiti cat maimulte modalitati!)

Dan Branzei

137

Clasa a V-a

1. Ciocolatele sunt ambalate ın cutii de cate 6, 9 si 20 de bucati. In cate moduriputem cumpara 33 de ciocolate? Dar 43? (Nota: Cutiile nu se desfac.)

Nelu Tudorache

2. Se considera multimea A = 2, 3, 4, . . . , 11, 12.a) Care este cel mai mic numar de numere care trebuie excluse din A, astfel ıncat

numerele ramase sa poata fi ımpartite ın doua multimi care au proprietatea ca sumanumerelor din prima multime este egala cu suma numerelor din a doua multime?

b) Care este cel mai mic numar de elemente care trebuie excluse din A, astfelıncat numerele ramase sa poata fi ımpartite ın doua multimi care au proprietatea caprodusul numerelor din prima multime este egal cu produsul numerelor din a douamultime?

3. a) Consideram noua numere diferite de 10. Aratati ca printre acestea existacel putin cinci numere care sunt fie mai mici decat 10, fie mai mari decat 10.

b) Un elev cumpara mai multe carti. Constata ca toate cartile au preturi diferite,niciuna nu costa 10 lei si ın orice grupa de cinci carti gaseste doua cu suma preturiloregala cu 20 lei. Aratati ca elevul a cumparat cel mult 8 carti si dati un exemplu delista de preturi pentru carti care ındeplineste toate conditiile date.

Gheorghe Iurea si Nelu Tudorache

Clasa a VI-a

1. Fie a, b, c ∈ N impare, d1 = (a, b, c), iar d2 =

a+ b

2,b+ c

2,c+ a

2

. Aratati

ca d1 = d2. (Notatia (a, b, c) desemneaza cel mai mare divizor comun al numerelora, b, c.)

2. Gigel si Costel se antreneaza zilnic 1 ora 53 minute 24 secunde pentru a par-ticipa la Turul Romaniei la ciclism. Antrenamentul ıl efectueaza pe o pista circularacu lungimea de 2268 m, miscandu-se uniform. Ei pleaca ın acelasi timp si din acelasiloc A, ın sensuri contrare. Dupa ce s-au ıntalnit prima data ın punctul B, lui Gigeli-au mai trebuit 81 de secunde iar lui Costel 144 de secunde pentru a ajunge ın A.

a) Cu ce viteza s-a deplasat fiecare biciclist?

b) Ce distanta a parcurs fiecare biciclist ıntr-o zi de antrenament?

Artur Balauca

3. Itsy si Bitsy sunt doi copii extraterestri de pe planeta Zizilon, ajunsi accidentalpe Terra, primul ın localitatea A si celalalt ın localitatea M . Prin statie, li s-a co-municat sa se ındrepte spre localitatea B. Localitatile A,M,B sunt legate printr-undrum drept, iar M se afla la mijlocul distantei dintre A si B. Cei doi se deplaseazanumai ın salturi, lungimile salturilor fiind programate de fiecare dupa anumite reg-uli. Itsy porneste din A si aterizeaza succesiv ın punctele A1, A2, A3, ..., Ak, dupa

urmatoarea regula: AA1 =1

nAB,A1A2 =

1

2AA1, A2A3 =

1

2A1A2, ..., Ak−1Ak =

1

2Ak−2Ak−1, unde n este un numar natural mai mare ca 1. Bitsy porneste din M

138

si aterizeaza succesiv ın punctele M1,M2, ...,Mk, dupa regula: MM1 =1

2mMB,

M1M2 =1

2MM1,M2M3 =

1

2M1M2, ...,Mk−1Mk =

1

2Mk−2Mk−1, unde m este un

numar natural mai mare decat 1.a) Este posibil ca Itsy sa aterizeze ın M?b) Aratati ca nu este posibil ca Bitsy sa aterizeze ın B.c) Aratati ca nu exista valori ale lui k astfel ıncat punctele Ak si Mk sa coincida.

Ioan Ticalo

Clasa a VII-a

1. a) Aflati toate numerele ıntregi x pentru care3x+ 2

4x− 3si

4x− 3

3x+ 2sunt simultan

numere ıntregi.

b) Fie F (x) =nx+ n− 1

(n+ 1)x− n, n ∈ N, n > 1. Demonstrati ca exista cel putin doua

numere naturale impare a si b astfel ıncat F (a) = b si F (b) = a.Claudiu Stefan Popa

2. Doua patrate de hartie sunt suprapuse unul peste celalalt astfel ıncat sa coin-cida, pe o suprafata plana presupusa nemarginita. Se alege un punct O pe patratulsituat deasupra. Se taie patratul de deasupra dupa segmentele determinate de O sivarfurile lui si fiecare triunghi astfel obtinut se asaza pe suprafata plana, astfel ıncatlatura care era si latura a patratului sa ramana lipita de latura patratului ramas, iarvarful opus ei sa fie simetricul lui O fata de aceeasi latura a patratului pe suprafataplana (nu se ia ın considerare grosimea celor doua patrate).

a) Ce legatura exista ıntre aria unuia dintre cele doua patrate si aria figurii geo-metrice obtinute?

b) Ce figura geometrica F se obtine prin procedeul expus ın enunt?c) Daca OA1 ·OA4 = OA2 ·OA3, unde A1, A2, A3, A4 sunt proiectiile lui O pe cele

patru laturi consecutive, demonstrati ca punctul O se afla pe o diagonala a patratuluisi precizati proprietatile caracteristice suplimentare ale figurii geometrice F .

Claudiu Stefan Popa si Al. Gabriel Mırsanu

3. Trapezul ABCD are baza mare AB, O este punctul de intersectie al diago-nalelor iar D′ este simetricul lui D fata de O. Demonstrati ca:

a) daca ^ACD ≡ ^BDC, atunci ABCD este trapez isoscel;b) daca AC ⊥ BD si AD′ ⊥ BC, atunci ABCD este trapez isoscel.

Claudiu Stefan Popa si Constantin Chirila

Clasa a VIII-a

1. a) Descompuneti ın factori expresia N (x, y) = x5 − x4y − 13x3y2 + 13x2y3 +36xy4 − 36y5.

b) Aratati ca nu exista valori ıntregi ale numerelor x si y pentru care N (x, y) = 77.

2. Fie A,B,C si D patru puncte necoplanare ın spatiu, astfel ıncat AB = CD.Patru bondari zboara pe traiectorii rectilinii, paralele, trecand fiecare prin cate unul

139

dintre cele patru puncte. Oricare bondar ısi alege traseul ın asa fel ıncat unghiurileformate de traiectoria sa cu dreptele AB, respectiv CD, sa fie egale.

a) Demonstrati ca distanta dintre traiectoriile bondarilor care trec prin A, re-spectiv prin B, este egala cu distanta dintre traiectoriile bondarilor care trec prin C,respectiv prin D.

b) Aratati ca niste bondari inteligenti ısi pot alege drumul ın cel putin 220+20102

moduri, astfel ıncat sa fie respectate cerintele problemei.Gabriel Popa si Cristian Lazar

3. In fiecare patratel al unei table de sah este scris cate un numar natural. Fiecarenumar n utilizat pentru completarea tablei apare pe tabla de cate n ori, iar pe primalinie exista, ordonate strict crescator, toate numerele folosite.

a) Aratati ca, dintre cele opt numere de pe prima linie, cel mult sase sunt pare.b) Determinati cel mai mare numar care poate aparea pe tabla.c) Demostrati ca exista o singura modalitate de completare a primei linii astfel

ıncat produsul numerelor de aceasta linie sa fie impar.Recreatii Matematice 1/2010

Etapa interjudeteana, 17 aprilie 2010

Clasa a IV-a

1. Tatal, mama si cei trei copii ai lor au ımpreuna 82 de ani. Varstele copiilor suntexprimate prin trei numere naturale consecutive pare. Aflati varsta fiecaruia stiindca, la nasterea celui de-al doilea copil, fiecare dintre parintii lui avea de 13 ori varstaprimului copil.

2. Avem scrise ın ordine primele 2010 numere naturale nenule. Eliminam dinacest sir o parte din numere dupa urmatorul procedeu: taiem primul numar, sarimal doilea numar, taiem urmatoarele doua numere, sarim urmatoarele doua numere,taiem urmatoarele trei numere, sarim urmatoarele trei numere s.a.m.d., pana cand lapasul n raman mai putin de n numere.

a) Verificati daca 2010 a fost taiat sau nu.b) Cate numere au ramas netaiate?

Catalin Budeanu

3. In cele noua patratele ale patratului mare alaturat sunt scrise numere distincte

a b 2ae 4 d6 f 12

mai mici decat 24. Aflati literele corespunzatoare, stiind ca sumanumerelor de pe fiecare linie si de pe fiecare coloana este aceeasi.

Clasa a V-a

1. Avem la dispozitie un numar nelimitat de jetoane, pe fiecare dintre ele fiindscris unul din numerele 5, 7 sau 11. Un numar natural n se numeste norocos dacaexista un numar de jetoane astfel ıncat suma numerelor scrise pe ele sa fie egala cu n.

a) Aratati ca 13 nu este norocos.b) Aratati ca 14, 15, 16, 17 si 18 sunt numere norocoase.c) Aratati ca orice numar natural n ≥ 14 este norocos.

140

2. Intr-un supermarket, ciocolatele de un anumit tip sunt asezate pe un standın 25 de cutii a cate 11 bucati, iar clientii ısi aleg ciocolatele, pentru a le cumpara,dintr-o cutie la ıntamplare. Aratati ca, ın orice moment, exista 3 cutii care continacelasi numar de ciocolate.

3. Pentru fiecare numar natural nenul n se noteaza cu s(n) numarul de zerouricu care se termina numarul 1 · 2 · 3 · . . . · n.

a) Calculati s(2010)− s(2000).b) Determinati cel mai mic numar natural n cu proprietatea ca s(n+10)−s(n) =

2010.

Clasa a VI-a

1. a) Sa se arate ca 2n > 2n−1 + 2n−2 + 2n−3, ∀n ∈ N, n ≥ 3.b) Se aleg la ıntamplare 9 divizori distincti ai numarului 22010 si se aseaza ın cele

noua patratele ale unei table 3 × 3, ıntr-o ordine oarecare. Sa se arate ca sumelenumerelor de pe fiecare linie, coloana sau diagonala ale tablei sunt distincte.

Marian Pantiruc

2. Aflati numerele naturale de forma xyz ın baza 10, stiind ca produsul cifrelor loreste un cub perfect de forma abc, iar numerele x2+xy+xz, y2+xy+yz, z2+xz+yzsunt direct proportionale, respectiv, cu numerele y + z, x+ z si x+ y.

Artur Balauca

3. Avem un cadru de tipul celui din desen, cu x > 15 si un echer dreptunghic.A

B D C

E

xx

F 600

600

Sa se precizeze daca se poate aseza unghiul drept alecherului ın unul dintre punctele A,B,C,D,E, F ast-fel ıncat fiecare cateta a echerului sa mai treaca princate un punct din cele sase numite mai sus si sa sejustifice.

Al. Gabriel Mırsanu si Dan Branzei

Clasa a VII-a

1. Intors de la Concursul Florica T. Campan, un copil este ıntrebat de mama sace a facut.

- Totul, mai putin o problema. Sa ti-o zic: se face o operatie, aceeasi, cu fiecaredoua din trei numere reale si, de fiecare data, rezultatul obtinut este un numarrational. Se cerea sa se demonstreze ca toate cele trei numere sunt rationale.

- Bine, bine-spune mama-despre ce operatie e vorba?- Pai ... nu tin minte, dar intru pe internet si gasesc subiectele acolo. Dupa aceea,

ma lasi sa joc un joc nou online?- Sigur, repede la internet pentru orice informatie! Tu memorie interna nu mai ai?- Imi aduc aminte doar ca era ori adunare, ori ınmultire.- Ei, atunci nu e greu. Uite, eu stiu deja ce operatie era ın enuntul problemei si

cum sa o rezolv.Aflati si voi cum a gandit mama baiatului si justificati raspunsul.

Claudiu Stefan Popa si Al. Gabriel Mırsanu

141

2. O lucrare poate fi efectuata, separat, de patru muncitori astfel: de primulmuncitor ın 2a ore, de al doilea muncitor ın 2b ore, iar al treilea si al patrulea

efectueaza aceeasi lucrare, fiecare, ın a + b ore (a, b ∈ R, a >1

2, b >

1

2, a = b).

Se formeaza doua echipe ın care muncitorii lucreaza ımpreuna: ın prima intra primulsi cel de al doilea muncitor, iar ın a doua intra al treilea si al patrulea. Cercetati caredintre cele doua echipe termina lucrarea mai ıntai.

Claudiu Stefan Popa

3. PuncteleM si N sunt mijloacele laturilor [AB], respectiv [CD], ale patrulateru-lui ABCD, AB∥CD, iar O este punctul de intersectie a diagonalelor sale. PuncteleE si F sunt simetricele punctului O fata de M , respectiv N , iar AC ∩ DE = P,BD ∩ CE = Q, AC ∩BF = R si BD ∩AF = S.

a) Presupunand ca AB = CD, demonstrati ca PQRS este paralelogram, PPQRS ≤PABCD

2si APQRS ≤ AABCD

4.

b) Aceleasi cerinte, presupunand ca CD < AB.Claudiu Stefan Popa si Constantin Chirila

Clasa a VIII-a

1. Fie n numar natural nenul; vom spune ca un numar real strict pozitiv x esten-insignifiant daca x < 10−n. Stabiliti, cu justificari, daca urmatoarele propozitiisunt adevarate sau false:

P1 : ”Exista numere 2010-insignifiante care nu sunt 2000-insignifiante.”P2 : ”Orice numar 2000-insignifiant este 2010-insignifiant.”P3 : ”Daca x si y sunt 2010-insignifiante, atunci suma x+y este 2010-insignifianta.”P4 : ”Daca x si y sunt 2000-insignifiante, atunci produsul xy este 2010-insignifiant.”

Dorel Luchian

2. Vom spune ca un numar natural n este 2010-acceptabil daca un cub de laturan poate fi ımpartit ın cel putin 2010 cuburi mai mici, toate avand laturile numerenaturale si vom spune ca este 2010-remarcabil daca un cub de latura n poate fi ımpartitın exact 2010 cuburi mai mici, toate avand laturile numere naturale.

a) Demonstrati ca numarul 12 nu este 2010-acceptabil.b) Aratati ca 13 este numar 2010-acceptabil si 2010-remarcabil.c) Demonstrati ca exista o infinitate de numere 2010-remarcabile.

Gabriel Popa si Cristian Lazar

3. Pe Marte, moneda folosita ın schimburile comerciale se numeste mart. BancaPlanetara Martiana bate monede avand ca valori orice numar ıntreg pozitiv de marti(exista monede de 1 mart, 2 marti, 3 marti etc.). Un martian perseverent strange,ban cu ban, o gramada de 2010 monede, a caror valoare totala este de 4018 marti.

a) Care este cea mai mare valoare posibila a unei monede din gramada?b) Demonstrati ca ın gramada se afla cel putin doua monede de 1 mart.c) Aratati ca un martian inteligent poate ımparti monedele ın doua gramezi mai

mici, astfel ıncat valoarea totala a monedelor din fiecare gramajoara sa fie de 2009marti.

Cristian Lazar si Gabriel Popa

142

Concursul de matematica ”Student pentru o zi”Editia a III-a, Suceava, 20 martie 2010

Problema 1. Se considera functiile f, g : (0, 3) → R, f(x) =lnx

3− xsi g(x) =

ln(3− x)

x, pentru orice x ∈ (0, 3).

a) Sa se calculeze

Z e

1(3− x)f(x)dx.

b) Sa se arate ca

Z 2

1f(x)dx =

Z 2

1g(x)dx.

c) Sa se arate ca limt→0

Z 1

tg(x)dx = ∞.

Problema 2. Se considera polinomul definit prin P (X) = X3 + b6X2 +c11X + b6,cu coeficientii ın Z2010.

a) Sa se arate ca polinomul are cel putin trei radacini distincte ın Z2010.

b) Sa se arate ca polinomul are o radacina de forma ×10k − 1, k ∈ Z, diferita deÔ2009.c) Sa se arate ca polinomul are cel putin sase radacini distincte ın Z2010.

Catalin Tigaeru, Suceava

Problema 3. Fie M multimea functiilor f : [0, 1] → R continue, cu proprietatea

ca

Z 1

0f2(x)dx =

Z 1

0f(x)dx.

a) Determinati valorile lui a si b pentru care functiile f, g : [0, 1] → R, f(x) = ax+1

si g(x) =1

2+

Éx

2+ b apartin multimii M .

b) Aratati ca multimea M este infinita.

c) Determinati maxZ 1

0f(x)dx|f ∈ M.

Ion Bursuc, Suceava

143

MARIA POPA

(1927-2010)

La ınscrierea ın gimnaziu, am ıntalnit ın scoala o doamna blanda, pe care am rugat-o sa ma ajute sa ma ınscriu ın clasa unde preda profesoara Maria Popa. Doamnaa zambit uimita si mi-a raspuns ca tocmai ea era aceea. Astazi, matematicieni,ingineri, profesori universitari sau medici, cu totii ne amintim cum ne ındruma sadevenim rezolvitori ai Gazetei Matematice, cum ne dezvolta creativitatea la cercurilede matematica sau ne stimula ın publicarea de note originale ın revistele scolare.

Absolventa a Universitatii din Bucuresti, promotia 1951, Maria Popa a fostdiscipola unor profesori de prestigiu: Gr. Moisil, M. Nicolescu, Gh. Vranceanu, S.Stoilow, H. Hulubei s.a. Mediul stiintific ın care s-a format a ajutat-o sa transmitasi elevilor sai o gandire corecta, riguroasa si profunda. Mi-o amintesc la catedraLiceului ”Dimitrie Cantemir” din Iasi, unde a profesat chiar de la ınfiintarea acestuiliceu, ın 1964, si pana la pensionarea sa, ın 1982. Maria Popa a obtinut cu eleviisai numeroase rezultate remarcabile la fazele finale ale Olimpiadelor Nationale deMatematica.

Nu putem ıncheia fara a spune despre Maria Popa ca a fost un exemplu demnde urmat nu numai pe plan profesional ci si familial. Cei doi copii ai sai, prin talentulmostenit ın matematica si educatia primita, s-au remarcat ca valorosi matematicieni.

Cunoscandu-i puternica caldura sufleteasca cu care-si ınconjura elevii (ca si petoti cei apropiati), am sentimentul ca si ın acest an 2010, al trecerii sale ın nefiinta,tot ne mai ıncalzeste ınca pe toti cu dragostea sa protectoare.

Prof. univ. dr. Cornelia-Livia BEJAN

Noutati din activitatea matematica scolara ieseana

• Dupa 2006, Iasul a gazduit din nou, ın aprilie 2010, Olimpiada Nationala deMatematica. (Afirmam ca mai eficient.)

• Din 2007, fara ıntrerupere, Iasul organizeaza continuarea extinsa a Concursului”Adolf Haimovici”, Concursul National al Liceelor cu profil Economic si Tehnologic,adevarata Olimpiada Nationala de Matematici Aplicate.

• In iunie 2009 si iunie 2010, Iasul a organizat pregatiri ale echipelor balcanice dejuniori ale Romaniei. (Actiune finantata de Fundatia ”Dinu Patriciu”.)

• Intre 14-16 iunie 2010 a fost pregatit la Iasi un lot al sperantelor olimpice, elevidin toata tara care au ıncheiat clasa a VII-a cu rezultate notabile la ONM.

• Concursul ”Al. Myller”, Concursul ”Florica T. Campan”, Concursul ”RecreatiiMatematice” si Olimpici pentru o zi au capatat caracter de regularitate. Citam aicisi concursurile Sperante Olimpice si Micii matematicieni ın organizarea Pascanilorsi Harlaului. In 2010 acestora li s-a adaugat un nou concurs interesant, Concursul”Henri Coanda” (evaluand ın paralel si creativitatea ın domeniul fizicii).

144

Solutiile problemelor propuse ın nr. 2/2009

Clasele primare

P.174. Mirela are un mar, o para si o portocala. Mama ıi spune sa aseze fructelepe doua farfurii identice, astfel ıncat pe fiecare farfurie sa fie cel mult doua fructe. Incate moduri poate aseza Mirela cele trei fructe?(Clasa I) Inst. Maria Racu, Iasi

Solutie. Mirela poate aseza cele trei fructe astfel: (mar, para)-(portocala); (mar,portocala)-(para); (para, portocala)-(mar), deci exista trei modalitati de aranjare.

a a+1 a+2 a+3

P.175. Scrieti toate numerele mai mici ca 27 care se pot des-compune sub forma indicata alaturat.(Clasa I) Diana Tanasoaie, eleva, Iasi

Solutie. Avem 1 + 2 + 3 = 6, 27− 6 = 21. Trebuie sa aflam toate numerele maimici ca 21 care se pot scrie sub forma a + a + a + a. Acestea sunt 20; 16; 12; 8; 4; 0.Numerele cerute de problema sunt 26, 22, 18, 14, 10, 6.

P.176. Intr-o bomboniera sunt cinci bomboane cu fructe si sapte bomboane cuciocolata. Care este cel mai mic numar de bomboane pe care trebuie sa-l luam dinbomboniera, fara sa ne uitam, pentru a avea cel putin doua bomboane cu ciocolata?(Clasa a II-a) Alexandru Dumitru Chiriac, elev, Iasi

Solutie. Situatia cea mai nefavorabila este cand luam ıntai toate cele cinci bom-boane cu fructe. Trebuie sa luam minimum 7 bomboane.

P.177. Cum masuram 1 litru de apa folosind doua vase negradate, unul de 5 litri,iar celalalt de 8 litri?(Clasa a II-a) Mariana Nastasia, eleva, Iasi

Solutie. Umplem vasul de 8 litri, iar din acesta pe cel de 5 litri. Golim vasul de5 litri si turnam ın el cei 3 litri ramasi ın vasul de 8 litri. Umplem din nou vasul de 8litri si turnam din el 2 litri peste cei 3 litri din vasul de 5 litri. Golim vasul de 5 litrisi ıl umplem din cei 6 litri ramasi ın vasul de 8 litri. In vasul de 8 litri ramane 1 litrude apa.

P.178. Aratati ca, daca restul este o cincime din scazator, atunci descazutul seımparte exact la 6. Care este cel mai apropiat descazut de numarul 100 cu aceastaproprietate?(Clasa a III-a) Mirela Cucoranu, eleva, Iasi

Solutie. Din formula scaderii, D− S = R, deducem D = S +R = 5R+R = 6R,de unde rezulta ca descazutul se ımparte exact la 6. Cel mai apropiat descazut denumarul 100, avand aceasta proprietate, este 102.

P.179. Se dau produsele: a× b = 60, a× c = 70, a× d = 95. Stiind ca b+ c+ deste de noua ori mai mare decat a, sa se afle valoarea lui a.(Clasa a III-a) Andreea Amarandei, eleva, Iasi

145

Solutie. a × b + a × c + a × d = 225 ⇒ a × (b + c + d) = 225 ⇒ a × (9 × a) =225 ⇒ a× a = 225 : 9 ⇒ a× a = 25 ⇒ a = 5.

P.180. Aratati ca din sirul 7, 28, 31, 46, 61, 100 nu putem extrage patru numere acaror suma sa se ımparta exact la trei.

(Clasa a III-a) Dragos Iacob, elev, Iasi

Solutie. Observam ca: 7 = 2 · 3+ 1, 28 = 9 · 3+ 1, 31 = 10 · 3+ 1, 46 = 15 · 3+ 1,61 = 20 · 3 + 1, 100 = 33 · 3 + 1. Suma a patru numere, din cele de mai sus, este deforma p · 3 + 4, care nu se ımparte exact la 3.

P.181. Un triunghi si un patrat au acelasi perimetru, exprimat printr-un numarnatural. Care este cea mai mica valoare a perimetrului? Cate valori posibile aleperimetrului sunt cuprinse ıntre 100 si 200?

(Clasa a IV-a) Andreea Alexa, eleva, Iasi

Solutie. a) Daca a este latura triunghiului, iar b este latura patratului, atunci3 · a = 4 · b, deci b trebuie sa se ımparta exact la 3. Pentru b = 3 obtinem perimetrulminim, care este 12.

b) Avem 100 < 12 · 9 < 12 · 10 < . . . < 12 · 16 < 200. Intre 100 si 200 sunt cuprinse8 valori ale perimetrului.

P.182. Aflati cea mai mica valoare a lui k astfel ıncat1

21+

2

21+

3

21+ · · ·+ k

21sa fie un numar natural.

(Clasa a IV-a) Ionela Baragan, eleva, Iasi

Solutie.1

21+

2

21+

3

21+ . . .+

k

21=

1 + 2 + 3 + . . .+ k

21=

(1 + k) · k : 2

21. Pentru

k = 6 se obtine1

21+

2

21+ . . .+

6

21= 1, iar pentru valori mai mici ale lui k, sumele

sunt subunitare.

P.183. Se considera noua numere naturale a, b, c, . . . , i. Media aritmetica a nu-merelor a si b este 1, media numerelor c, d si e este 5, iar media numerelor f, g, h sii este 11. Aflati media aritmetica a numerelor a, b, c, . . . , i si 9.

(Clasa a IV-a) Ionel Nechifor, Iasi

Solutie. a+ b+ c+ d+ e+ f + g + h+ i+ 9 = 2 · 1 + 3 · 5 + 4 · 11 + 9 = 70, iar70 : 10 = 7.

Clasa a V-aV.109. Aflati catul si restul ımpartirii numarului 3 · 22009 la 5 · 22007.

Damian Marinescu, Targoviste

Solutie. Deoarece 3·22009 = (5+1)·22008 = (5·22007)·2+22008, iar 22008 < 5·22007,catul cautat este 2, iar restul este 22008.

V.110. Determinati patru numere naturale x, y, z, t cu proprietatea ca 2x−1

+ 3 · 22y+1 + 5 · 23z+2 + 11 · 25t+1 = 2009.

Catalina Dragan, Galati

Solutie. Trecand numarul 2009 ın baza 2, avem ca 2009 = 11111011001(2) =210 + 29 + 28 + 27 + 26 + 24 + 23 + 20. Pe de alta parte, 2x−1 + 3 · 22y+1 + 5 ·23z+2 + 11 · 25t+1 = 2x−1 + (2 + 1) · 22y+1 + (22 + 1) · 23z+2 + (23 + 2 + 1) · 25t+1 =

146

2x−1 +22y+2 +22y+1 +23z+4 +23z+3 +25t+4 +25t+2 +25t+1. Identificand, rezulta cat = 1, z = 2, x = y = 1.

V.111. Demonstrati ca numarul 20200 are 261 de cifre ın scrierea ın baza 10.Geanina Havarneanu, Iasi

Solutie. Trebuie sa demonstram ca 10260 ≤ 20200 < 10261, deci ca 1060 ≤ 2200 <1061. Pentru prima inegalitate, observam ca 103 = 1000 < 1024 = 210, prin urmare1060 = (103)20 < (210)20 = 2200. Demonstram acum a doua inegalitate:

2200 = 22 · 2198 = 4 · (233)6 = 4 · 85899345926 < 10 · (1010)6 = 1061

(calculul lui 233 a fost efectuat cu ajutorul unui calculator de buzunar).

V.112. Demonstrati ca multimea A =

§x =

3n+ 4

4n+ 3

n ∈ N, 1000 ≤ n ≤ 2009

ªare

1010 elemente.Daniela Munteanu, Iasi

Solutie. Trebuie sa demonstram ca, daca vom da valori distincte lui n, vom obtine

valori distincte ale lui x. Pentru aceasta, sa presupunem ca avem3n1 + 4

4n1 + 3=

3n2 + 4

4n2 + 3;

atunci (3n1 + 4)(4n2 + 3) = (4n1 + 3)(3n2 + 4), de unde 12n1n2 + 9n1 + 16n2 + 12 =12n1n2 + 16n1 + 9n2 + 12. Rezulta ca n1 = n2 si rezolvarea este completa.

V.113. Daca S = 1 +1

2+

1

3+ . . .+

1

2009, demonstrati ca S >

13

2.

Al. Gabriel Mırsanu, Iasi

Solutie. Cum1

3=

1

4+

1

12si

1

12>

39

2010>

1

2010+

1

2011+ . . .+

1

2048, deducem

ca S > 1 +1

2+

1

4+

1

4+

1

5+ . . .+

1

2048. Utlizand acum procedeul uzual, obtinem ca

S > 1 +1

2+

1

4+

1

4

+

1

8+

1

8+

1

8+

1

8

+ . . .+

1

2048+ . . .+

1

2048| z 1024 de termeni

!= 1+

1

2+

1

2+

1

2+ . . .+

1

2=

13

2.

V.114. Se considera ın plan cinci drepte distincte, care ımpart planul ın maimulte regiuni. Aratati ca oricum am alege 2009 puncte din plan care nu apartindreptelor, vor exista cel putin 126 de puncte dintr-o aceeasi regiune.

Nicolae Ivaschescu, CraiovaSolutie. Numarul maxim de regiuni ın care dreptele ımpart planul se obtine

atunci cand ele sunt concurente doua cate doua, fara ınsa sa existe trei care trecprintr-un acelasi punct. In acest caz, obtinem 16 regiuni; presupunand ca ın fiecarear exista cel mult 125 de puncte, numarul total de puncte ar fi cel mult egal cu125 · 16 = 2000, absurd. Ramane atunci adevarata concluzia problemei.

V.115. O multime de numere naturale A = a1, a2, . . . , a9 are elementele aran-jate strict crescator; media aritmetica a primelor doua elemente este 1, media urma-toarelor trei este 5, iar media ultimelor patru este 11. Cate astfel de multimi exista?

Ionel Nechifor si Gabriel Popa, Iasi

147

Solutie. Primele doua elemente nu pot fi decat a1 = 0, a2 = 2. Suma urmatoarelortrei este 15, iar 3 ≤ a3 < a4 < a5; avem posibilitatile (a3, a4, a5) ∈ (3, 4, 8); (3, 5, 7);(4, 5, 6). Daca a5 = 8, atunci (a6, a7, a8, a9) ∈ (9, 10, 11, 14); (9, 10, 12, 13). Canda5 = 7, pe langa cele doua cazuri dinainte, mai obtinem ınca sapte, anume (8, 9, 10, 17);(8, 9, 11, 16); (8, 9, 13, 14); (8, 10, 11, 15); (8, 10, 12, 14); (8, 11, 12, 13). Daca a5 = 6,convin cele noua posiblitati enuntate, la care se adauga ınca 14 : (7, 8, 9, 20); . . . ;(7, 8, 14, 15); (7, 9, 10, 18); . . . ; (7, 8, 13, 15); (7, 10, 11, 16); (7, 10, 12, 15); (7, 10, 13, 14);(7, 11, 12, 14). In total, numarul multimilor A este 2 + 9 + 23 = 34.

Clasa a VI-aNota. (Titu Zvonaru, Comanesti) In solutia problemei VI.95 din RecMat

2/2009, pg. 138, se spune ca din relatia a1+1004a2 = 2009, unde a1, a2 ∈ N∗, rezultaca a1 = 1, a2 = 2. De fapt, mai exista si posibilitatea a1 = 1005, a2 = 1, cand obtinema3 = 3 · 1005, a5 = 5 · 1005, . . . , a2007 = 2007 · 1005, a4 = 2, a6 = 3, . . . , a2006 = 1003,a2008 = 1004.

VI.109. Determinati a, b, c, d ∈ N pentru care 2a+ 3b+ 5c+ 7d = 87, daca:a) a, b, c, d sunt numere prime;b) a, b, c, d sunt patrate perfecte.

Nicolae Ivaschescu, CraiovaSolutie. Observam ca d nu poate lua valori prea mari (cel mult 7 la punctul a),

cel mult 9 la b)). Dam, pe rand, toate valorile posibile lui d si facem consideratiiasupra marimii lui c etc. Obtinem solutiile:

a) (a, b, c, d) ∈ (2, 3, 5, 7); (2, 11, 3, 5); (3, 7, 5, 5); (3, 19, 2, 2); (5, 7, 7, 3); (7, 3, 3, 7);(11, 2, 2, 7); (11, 3, 7, 3); (11, 5, 3, 5); (13, 5, 5, 3);

b) (a, b, c, d) ∈ (0, 0, 16, 1); (0, 25, 1, 1); (1, 4, 9, 4); (4, 9, 9, 1); (16, 1, 9, 1); (16, 9, 0, 4);(16, 16, 0, 1); (36, 1, 1, 1).

VI.110. Determinati perechile de numere naturale care au suma 2009 si produsulmultiplu al numarului 2009.

Dan Popescu, Suceava

Solutie. Fie a, b ∈ N astfel ıncat a + b = 2009, iar ab...2009. Cum 2009 = 72 · 41,

rezulta ca 41|a sau 41|b. Fie a = 41a1; obtinem ca b = 2009−41a1 = 41(49−a1), decib = 41b1, cu a1+b1 = 49. Din 72 ·41|412 ·a1b1, deducem ca 72|a1b1 si, tinand seama side conditia precedenta, gasim ca (a1, b1) ∈ (0, 49); (7, 42); (14, 35); (21, 28); (28, 21);(35, 14); (42, 7); (49, 0). Corespunzator, obtinem opt perechi (a, b).

VI.111. Demonstrati ca numarul A = 40! ·1 +

1

2+ . . .+

1

40

este natural,

divizibil cu 2009 · 72 (unde 40! = 1 · 2 · . . . · 40).Mihai Haivas, Iasi

Solutie. Desfacand paranteza si observand ca fiecare dintre numitorii 2, 3, . . . , 40divide 40!, deducem ca A ∈ N. Apoi, avem:

A = 40! ·1 +

1

40+

1

2+

1

39+ . . .+

1

20+

1

21

= 41 ·

40!

40+

40!

2 · 39+ . . .+

40!

20 · 21

.

Cum 40! se divide cu 75, iar numitorii fractiilor se divid cel mult cu 7, rezulta caA = 41 · 74 ·B = 2009 · 72 ·B, cu B ∈ N.

148

VI.112. Fie a, b, c, d ∈ N∗ astfel ıncat ad+ bc = bd. Demonstrati ca

a2009

b2009+

a2008c

b2008d+ . . .+

a2c

b2d+

ac

bd+

c

d∈ N.

Catalin Budeanu, Iasi

Solutie. Fie Sn = xn+xn−1y+xn−2y+ . . .+xy+y, n ∈ N∗, unde x =a

b, y =

c

d;

atunci Sn = xn−1(x+y)+xn−2y+ . . .+xy+y = xn−1+xn−2y+ . . .+xy+y = Sn−1.Rezulta ca Sn = S1 = x+ y = 1, deci S2009 = 1.

VI.113. Dupa doua reduceri succesive, pretul unui frigider scade de la 2000 lei la1620 lei. Stiind ca cele doua reduceri sunt proportionale cu preturile ramase ın urmalor, aflati pretul frigiderului dupa prima reducere.

Ciprian Baghiu, IasiSolutie. Prima data pretul se reduce cu p%, ajungand la x lei, iar a doua oara

se reduce cu q%. Din ipoteza, avem cap% · 2000

(1− p%) · 2000=

q% · x(1− q%) · x

, de unde

obtinem ca p = q. Rezulta ca1− p

100

2· 2000 = 1620, deci

1− p

100

2=

9

10

2

,

prin urmare p = 10. Asftel, x =90

100· 2000 = 1800 lei.

VI.114. Pe laturile [BC], [AC], [AB] ale triunghiului isoscel ABC (AB = AC)

consideram punctele D,E, respectiv F , astfel ıncat m(ÕBAD) = 2m(ÕEDC) si m(ÕDAC)

= 2m(ÕFDB). Demonstrati ca AEF este isoscel.Doru Buzac, Iasi

Solutie. Fie a = m(ÕEDC), b = m(ÕFDB); atunci m(ÕBAD) = 2a, iar m(ÕCAD) =

F

B

A

E

D C

2b. Masurile unghiurilor ÒB si ÒC sunt, fiecare, egale cu1

2[180 −

(2a + 2b)] = 90 − a − b. Cum ÕADC este unghi exterior tri-

unghiului ABD, obtinem ca m(ÕADC) = 2a + 90 − a − b =

90 + a − b. Pe de alta parte, m(ÕADC) = m(ÕADE) + a si

atunci m(ÕADE) = 90 − b. In triunghiul ADE, vom avea ca

m(ÕAED) = 180 − 2b − (90 − b) = 90 − b = m(ÕADE), prinurmare AD = AE. Analog se arata ca AF = AD si astfel rezulta concluzia problemei.

VI.115. Dreptele a si b sunt perpendiculare pe segmentul [AB] ın A, respectivın B. Consideram punctele C ∈ (AB), M ∈ a, N,P ∈ b astfel ıncat ıntre oricaredoua dintre triunghiurile ACM,BCN si BCP exista cate o congruenta. Stiind ca

m(ÕBPC) = 25, determinati masurile unghiurilor triunghiului MNP.Andrei Nedelcu, Iasi

Solutie. Intre triunghiurile BCN si BCP, singura congruenta posibila este

BCN ≡ BCP. Daca ACM ≡ BCN ≡ BCP , atunci ÖACM ≡ ÕBCPsi rezulta ca punctele M,C,P sunt coliniare. In triunghiul MNP, mediana NC

este egala cu jumatatea laturii pe care cade, prin urmare m(ÖMNP ) = 90, iar

m(ÖNMC) = 65.

149

In cazul ın care AMC ≡ BCN = BCP , avem ca m(ÖACM) = m(ÕCNB) =

90 − m(ÕNCB), de unde m(ÖMCN) = 90. Triunghiul CNM va fi dreptunghic

isoscel, deci m(ÖCNM) = m(ÖCMN = 45. In CMP isoscel, avem m(ÖMCP ) =

360 − 90 − 2 · 65 = 140, astfel ca m(ÖCMP ) = m(ÖCPM) = 20. Deducem

ca m(ÖMNP ) = 45 + 25 = 70, m(ÖNMP ) = 45 + 20 = 65, iar m(ÖNPM) =25 + 20 = 45.

Clasa a VII-aVII.109. Fie ABCD dreptunghi, O mijlocul lui [AC], M ∈ (AO), N ∈ (OC),

P = BM ∩ AD si Q = BN ∩ CD. Demonstrati ca O este centrul de greutate al

triunghiului BPQ daca si numai daca OM = ON =1

6AC.

Petru Asaftei, IasiSolutie. Notam K = PQ ∩ BD. Daca O este centru de greutate ın BPQ,

atunci K va fi mijlocul lui [PQ], iar KO =1

2OB =

1

2OD. In patrulaterul OPDQ,

diagonalele se ınjumatatesc si m(ÒD) = 90, prin urmare OPDQ este dreptughi.Obtinem ca P si Q sunt mijloace pentru [AD], respectiv [CD], deci M si N vor

fi centre de greutate ın BAD, respectiv BCD. Rezulta ca OM =1

3OA =

1

6AC

si, analog, ON =1

6AC. Reciproca se demontreaza urmarind rationamentul invers.

VII.110. Masurile unghiurilor A,B si C ale triunghiului ABC sunt direct propor-tionale cu 5, 4 si 3, iar BC = (2+2

√2+2

√3) cm. Demonstrati ca perimetrul si aria

triunghiului sunt numeric egale.Constantin Apostol, Rm. Sarat

Solutie. Obtinem imediat ca m( bA) = 75, m(ÒB) = 60, iar m(ÒC) = 45. Daca

ducem ınaltimea AD si notam BD = x, din ABD dreptunghic cu m(ÕBAD) =30 rezulta ca AB = 2x, AD = x

√3, iar din ADC dreptunghic isoscel gasim

ca CD = x√3, AC = x

√6. Astfel, BC = x + x

√3 = 2 + 2

√2 + 2

√3, de unde

x =2 + 2

√2 + 2

√3

1 +√3

= 2 +√6−

√2, prin urmare AB = (4 + 2

√6− 2

√2)cm, AC =

(2√6 + 6 − 2

√3)cm, iar AD = (2

√3 + 3

√2 −

√6)cm. In concluzie, PABC = AB +

BC +AC = (12 + 4√6)cm si AABC =

BC ·AD

2= (12 + 4

√6)cm2.

VII.111. Fie ABC un triunghi si punctele D ∈ (AC), E ∈ (AB), P = BD ∩

CE. DacaDA

DC= k, demonstrati ca k

PC

PE− (k + 1)

PD

PB= 1.

Neculai Roman, Mircesti (Iasi)Solutie. Cu teorema lui Menelaus ın ACE si transversala B − P − D,

A

B C

ED

P

obtinem caDA

DC·PC

PE·BE

BA= 1, de unde k·PC

PE=

AB

BE. Aplicand

acum Menelaus ın ABD cu transversala C−P −E, deducem

caEB

EA· CA

CD· PD

PB= 1 ⇔ CA

CD· PD

PB=

EA

BE, prin urmare

(k + 1)PD

PB=

EA

BE. Scazand membru cu membru relatiile

150

obtinute, rezulta concluzia problemei.

Nota autorului. Daca BD este mediana, atunci k = 1, prin urmarePC

PE−

2 · PD

PB= 1. Aceasta relatie are loc oricare ar fi ceviana CE, ın particular si atunci

cand CE este bisectoare. Am gasit astfel o generalizare a problemei 8.4 din FoaieMatematica 4/2002.

VII.112. Fie ABCD trapez cu baza mare [AB], E = AD ∩ BC, O =AC ∩ BD, iar OP∥AB, cu P ∈ (AD). Demonstrati ca CP si CE sunt bisectoarele

(interioara, respectiv exterioara) unghiului ÕACD, daca si numai daca AB = AC.

Claudiu Stefan Popa, Iasi

Solutie. Presupunem ca AB = AC. Din asemanarileAPO ∼ ADC siOAB

A B

CD

OP

E∼ OCD, obtinem caAP

PD=

AO

OC=

CD

AB=

CD

CA.

Reciproca teoremei bisectoarei arata ca CP este bisec-

toarea unghiului ÕACD. Cum ÕECD ≡ ÕCBA (corespon-

dente pentru AB∥CD, BC secanta) si ÕCBA ≡ ÕACB

(ABC isoscel), deducem ca ÕECD ≡ ÕACB, prin ur-

mare ÕECP ≡ ÕBCP . Rezulta astfel ca CP⊥CE, deci

CE va fi bisectoarea exterioara a unghiului ÕACD.

Reciproc, daca CP si CE sunt bisectoarele unghiuluiÕACD, atunci PC⊥BE si ÕPCD ≡ ÕPCA. Urmeaza ca ÕECD ≡ ÕBCA si, cum ÕECD ≡ÕCBA, atunci ÕACB ≡ ÕABC, adica ABC este isoscel cu AB = AC.

VII.113. a) Demonstrati ca√b2 − bc+ c2 ≥

√3 · bc√

b2 + bc+ c2, ∀b, c ∈ R∗

+.

b) Considerand un triunghi ABC cu m( bA) = 120, AB = c, AC = b, interpretatigeometric inegalitatea de la a).

Dan Mocanu, elev, Iasi

Solutie. a) Cum b, c sunt pozitive, inegalitatea revine la (b2+c2−bc)(b2+c2+bc) ≥3b2c2, adica (b2 + c2)2 − b2c2 ≥ 3b2c2, altfel spus b2 + c2 ≥ 2bc, ceea ce este evidentadevarat.

b) Din teorema cosinusului, obtinem ca BC2 = b2 + c2 + bc. Fie [AM ] si [AD]

mediana, respectiv ınaltimea din A. Observam ca AM2 =2(b2 + c2)−BC2

4=

b2 + c2 − bc

2, deciAM =

1

2

√b2 + c2 − bc, iarAD =

2S

BC=

bc sinA

BC=

1

√3bc√

b2 + c2 + bc.

Astfel, inegalitatea din enunt arata ca mediana AM este cel putin egala cu ınaltimeaAD.

VII.114. Demonstrati ca produsul a doua numere naturale nenule consecutive nupoate fi egal cu produsul altor patru numere naturale consecutive.

Mihai Craciun, Pascani

Solutie. Presupunem, prin absurd, ca exista m,n ∈ N astfel ıncat n(n + 1) =m(m+1)(m+2)(m+3); atunci n2+n+1 = (m2+3m+1)2. Insa n2 < n2+n+1 <

151

(n + 1)2, ∀n ∈ N∗, prin urmare numarul n2 + n + 1 nu poate fi patrat perfect.Contradictia la care am ajuns arata ca este adevarata concluzia problemei.

VII.115. Demonstrati ca1

n+ 1+

1

n+ 2+ · · ·+ 1

4k · n> k, ∀n, k ∈ N∗.

Cosmin Manea si Dragos Petrica, Pitesti

Solutie. Pentru ınceput, observam ca Sa =1

a+ 1+

1

a+ 2+. . .+

1

4a=

1

a+ 1+ . . .

+1

2a

+

1

2a+ 1+ . . .+

1

4a

> a· 1

2a+2a· 1

4a=

1

2+1

2= 1, ∀a ∈ N∗. Cum suma din

enunt este Sn+S4n+S42n+ . . .+S4k−1n, rezulta ca ea depaseste 1 + 1 + . . .+ 1| z k termeni

= k.

Clasa a VIII-aVIII.109. Fie ABCDA′B′C ′D′ un cub de muchie a. Notam cu E,F,G,H,K,L

mijloacele muchiilor AB,BC,CC ′, C ′D′, D′A′, respectiv A′A. Calculati volumul poli-edrului B′EFGHKL.

Adrian Corduneanu, Iasi

Solutie. Se arata usor ca punctele E,F,G,H,K,L sunt coplanare. Fiecare dintre

A B

CD

AB

CD

L O

G

F

H

E

K

segmentele [EF ], [FG], . . . , [LE], [OE], [OF ], . . . , [OL]

are lungimeaa√2

2, deci EFGHKL este un hexagon

regulat de centru O. Mai observam ca B′E =

B′F = . . . = B′L =a√5

2; ın concluzie, poliedrul

B′EFGHKL este o piramida hexagonala regulata,

avand aria bazei 6 · EF 2√3

4=

3a2√3

4si ınaltimea

B′O =a√3

2. Volumul este

1

3· 3a

2√3

4· a

√3

2=

3a3

8.

VIII.110. Fie V ABCD o piramida patrulatera regulata. Notam cu u =

m( Û(V BC), (ABC)), v = m( Û(V BC), (V CD)) si t = m( Û(V BC), (V AD)). Stabilitidaca printre numerele u, v, t pot exista perechi de numere egale. (In legatura cuVIII.98 din RecMat 2/2008.)

Claudiu Stefan Popa, Iasi

Solutie. Ca ın solutia problemei VIII.98, prezentata ın RecMat 2/2009, pg. 142,

A B

CD

NM

P

O

Vfie u = m(ÖVMN), v = m(ÕBPD) si t = m(ÖMVN),unde M,N sunt mijloacele lui [BC], respectiv [AD],iar P este piciorul perpendicularei din B pe V C (si,totodata, al perpendicularei din D pe V C). In loculamintit s-a demonstrat ca v > u, prin urmare nuputem avea u = v. Vom arata ca ın nicio piramidapatrulatera regulata nu putem avea v = t. Evident,poate avea loc egalitatea u = t, ın cazul ın care

V O =AB ·

√3

2, deci cand VMN este echilateral.

152

Presupunem, prin absurd ca v = t; atunci ÖOVM ≡ ÕOPB si, cum cele doua

unghiuri sunt ascutie, am obtine ca tgÖOVM = tgÕOPB, deciOM

VO=

OB

OP⇔ OM

VO=

V C

V O⇔ OM = V C. Insa, ın triunghiul dreptunghic OV C, avem ca V C > OC, iar ın

triunghiul dreptunghic OMC avem ca OC > OM, prin urmare V C > OM ın oricepiramida patrulatera regulata. In concluzie, nu este posibil ca doua dintre cele treinumere sa fie egale.

VIII.111. Fie ABC un triunghi de laturi a, b, c, astfel ıncat b + c = a√2.

Demonstrati ca triunghiul este ascutitunghic daca si numai daca b si c sunt distincte

si se afla ın intervalul

a√2

4,3a

√2

4

.

Romanta Ghita si Ioan Ghita, Blaj

Solutie. Unghiul bA este ascutit daca si numai daca a2 < b2 + c2 ⇔ (b+ c)2

2<

b2 + c2 ⇔ (b− c)2 > 0, deci atunci cand b = c. Unghiul ÒB este ascutit daca si numai

daca b2 < a2 + c2 ⇔ b2 <(b+ c)2

2+ c2 ⇔ b2 − c2 < 2c2 +2bc ⇔ b− c < 2c ⇔ b < 3c.

Analog se arata ca ÒC este ascutit daca si numai daca c < 3b. Cum b + c = a√2,

urmeaza concluzia problemei.

VIII.112. Fie x, y ∈ R∗ astfel ıncat xy,x

ysi y

Èx2 + (x+ 1)2 + x2(x+ 1)2 sunt

toate numere rationale. Demonstrati ca x si y sunt tot numere rationale.

Dan Nedeianu, Drobeta Tr. Severin

Solutie. Observam ca x2 + (x + 1)2 + x2(x + 1)2 = x4 + 2x3 + 3x2 + 2x + 1 =(x2+x+1)2, prin urmare y(x2+x+1) ∈ Q si, cum xy ∈ Q, rezulta ca y(x2+1) ∈ Q.

Insa x2 = (xy) · xy∈ Q, deci x2 + 1 ∈ Q si obtinem ca y ∈ Q∗, apoi ca x ∈ Q∗.

VIII.113. Daca a, b, c ∈ R∗+, demonstrati ca

1

a+1

b+1

c=2

1

a+ b+

1

b+ c+

1

c+ a

daca si numai daca a = b = c.

D.M. Batinetu-Giurgiu, Bucuresti

Solutie. Identitatea din enunt se scrie echivalent sub forma

1

a+

1

b− 4

a+ b

+

1

b+

1

c− 4

b+ c

+

1

c+

1

a− 4

c+ a

= 0 ⇔ (a− b)2

ab(a+ b)+

(b− c)2

bc(b+ c)+

(c− a)2

ca(c+ a)= 0.

Aceasta din urma egalitate are loc daca si numai daca (a−b)2 = (b−c)2 = (c−a)2 = 0,i.e. a = b = c.

VIII.114. Demonstrati ca oricare ar fi numerele naturale impare m,n cu m >n+ 2, exista numere naturale x, a, b astfel ıncat x = a(a+m) = b(b+ n).

Titu Zvonaru, Comanesti

Solutie. Observam ca a(a+m) = b(b+n) ⇔a+

m

2

2−b+

n

2

2=

m2 − n2

4⇔

a+ b+m+ n

2

a− b+

m− n

2

=

m2 − n2

4. Facem alegerea (avem aceasta liber-

153

tate!) a+b+m+ n

2=

m2 − n2

4, a−b+

m+ n

2= 1, deci a =

(m− n− 2)(m+ n− 2)

8,

b =(m− n− 2)(m+ n+ 2)

8. Ramane sa demonstram ca a, b sunt numere naturale.

Deoarece m si n sunt ambele impare, numerele m− n− 2, m+ n− 2, m+ n+ 2sunt toate pare. Daca m = M4 + 1, n = M4 + 3 sau invers, atunci m− n− 2 = M4,deci a, b ∈ N. Daca m = M4 +1 si n = M4 +1 sau m = M4 +3 si n = M4 +3, atuncim+ n− 2 = M4, m+ n+ 2 = M4, deci a, b ∈ N, ceea ce ıncheie solutia problemei.

VIII.115. Demonstrati ca 5(a2 + b2)2 ≤ 4a4b4 + (a+ b)4,∀a, b ∈ [1,+∞).Lucian Tutescu si Ion Visan, Craiova

Solutie. Dupa efectuarea ridicarilor la putere si reducerea termenilor asemenea,inegalitatea din enunt se scrie sub forma a4 + b4 + a2b2 ≤ a4b4 + a3b + ab3. Fara arestrange generalitatea, putem presupune ca 1 ≤ a ≤ b; atunci a4 ≤ a3b, b4 ≤ a4b4,iar a2b2 ≤ ab3. Sumand ultimele trei inegalitati, obtinem ceea ce dorim. Egalitatease atinge pentru a = b = 1.

Clasa a IX-aIX.101. Prin inductie matematica se arata ca are loc inegalitatea lui Bernoulli

(1) (1 + x)n ≥ 1 + nx,

∀n ∈ N, n ≥ 2 si ∀x ∈ [−1,∞), egalitatea fiind atinsa pentru x = 0. Aratati ca:a) daca n = 2k, k ∈ N∗, atunci (1) are loc ∀x ∈ R;b) daca n = 2k + 1, k ∈ N∗, atunci (1) are loc ∀x ∈ [−2,∞);c) daca n = 3, atunci (1) are loc ∀x ∈ [−3,+∞), cu egalitate cand x ∈ −3, 0,

iar pentru x ∈ (−∞,−3), (1) are loc cu sens contrar.Dorin Dutkay, Orlando (U.S.A.) si Florin Popovici, Brasov

Solutie. a) Daca x ∈ (−∞,−1), atunci 1 + x < 0, prin urmare (1 + x)2k > 0 >1 + x > 1 + 2kx, deci are loc (1).

b) Daca x ∈ [−2,−1), atunci −1 ≤ 1 + x < 0, de unde |1 + x| ≤ 1. Obtinem ca|1+x|n ≤ |1+x|, ∀n ∈ N∗, deci (1+x)2k+1 = −|1+x|2k+1 ≥ −|1+x| > 1+(2k+1)x.

c) Pentru n = 3, avem ca (1) ⇔ 1 + 3x + 3x2 + x3 ≥ 1 + 3x ⇔ x2(3 + x) ≥ 0.Rezulta ca (1+x)3 = 1+3x ⇔ x ∈ −3, 0, (1+x)3 > 1+3x ⇔ x ∈ (−3, 0)∪ (0,∞)si (1 + x)3 < 1 + 3x ⇔ x ∈ (−∞,−3).

IX.102. Rezolvati ın R3 sistemul :

x+y+z = 2− 1

x− 1

y− 1

z;x2+y2+z2 = 6− 1

x2− 1

y2− 1

z2;x3+y3+z3 = 2− 1

x3− 1

y3− 1

z3.

Vasile Chiriac, Bacau

Solutie. Cu notatiile u = x+1

x, v = y+

1

y, w = z+

1

z, sistemul devine u+v+w = 2,

u2+v2+w2 = 12, u3+v3+w3 = 8. Cum u2+v2+w2 = (u+v+w)2−2(uv+uw+vw),obtinem ca uv + uw + vw = −4. Apoi, deoarece u3 + v3 + w3 = 3uvw + (u + v +w)(u2 + v2 + w2 − uv − uw − vw), deducem ca uvw = −8. Rezulta ca numereleu, v si w sunt solutiile ecuatiei t3 − 2t2 − 4t + 8 = 0, i.e. (t − 2)2(t + 2) = 0, prin

154

urmare (u, v, w) ∈ (2, 2,−2); (2,−2, 2); (−2, 2, 2). De aici, rezulta ca (x, y, z) ∈(1, 1,−1); (1,−1, 1); (−1, 1, 1).

IX.103. Fie x, y, z ∈ R cu 0 ≤ x ≤ y ≤ z. Daca α ∈ R este astfel ıncatαx+ (1− α)z ≥ 0, demonstrati ca αx+ (1− α)y ≥ 0 si αy + (1− α)z ≥ 0.

Ovidiu Pop, Satu MareSolutie. Daca x = z, atunci x = y = z si concluzia se impune; fie deci x = z.

Notand αx + (1 − α)z = t ≥ 0, rezulta ca α =t− z

x− zsi atunci αx + (1 − α)y =

t(y − x) + x(z − y)

z − x≥ 0, iar αy + (1− α)z =

t(z − y) + z(y − x)

z − x≥ 0.

Altfel, consideram cazurile α ∈ [0, 1], α ∈ (−∞, 0) si α ∈ (1,∞) si comparam celetrei expresii din enunt termen cu termen.

IX.104. Fie A,B,C,D patru puncte ale cercului C(O, r), M = AB ∩ CD,N si P mijloacele coardelor [AB], respectiv [CD], iar Ω cel de-al patrulea varf alparalelogramului NMPΩ.

a) Aratati ca−−→MA+

−−→MB +

−−→MC +

−−→MD = 2

−−→MΩ.

b) Paralelele prin C si D la AB si paralelele prin A si B la CD se taie doua catedoua ın patru puncte ce determina un paralelogram de centru Ω.

c) Ω = O daca si numai daca AB ⊥ CD.Diana Vranceanu, eleva si Dumitru Mihalache, Barlad

Solutie. a) Avem ca−−→MA +

−−→MB +

−−→MC +

−−→MD = 4

−−→MG, unde G este mijlocul

segmentului [NP ] (deci centrul de greutate al patrulaterului ABCD). Insa−−→MΩ =

2−−→MG, de unde rezulta relatia dorita.b) Notam RSTU paralelogramul construit conform enuntului, astfel ıncat A ∈

RU , B ∈ ST , C ∈ RS, D ∈ TU . Atunci AMDU si BMCS sunt paralelograme, prin

urmare−−→MA+

−−→MD =

−−→MU si

−−→MB +

−−→MC =

−−→MS. Notand cu O′ mijlocul segmentului

[SU ], obtinem ca

2−−−→MO′ =

−−→MO +

−−→MS =

−−→MA+

−−→MB +

−−→MC +

−−→MD = 2

−−→MΩ ⇒ O′ = Ω,

deci Ω este centrul paralelogramului RSTU .c) Deoarece ON ⊥ AB, avem ca Ω = O ⇔ ONMP paralelogram ⇔ ONMP

dreptunghi ⇔ AB ⊥ CD.

IX.105. Intr-un triunghi, cu notatiile uzuale, demonstrati echivalenta conditiilor:(i) R = ra; (ii) cosA = cosB + cosC.

Temistocle Bırsan, Iasi

Solutia 1. R = ra ⇔ a

2 sinA= p · tg A

2⇔ a = 4p sin2

A

2⇔ 2 sin2

A

2=

a

a+ b+ c⇔ (a + b + c) cosA = (a cosC + c cosA) + (a cosB + b cosA) ⇔ (a + b +

c) cosA = a(cosB + cosC) + (b+ c) cosA ⇔ cosA = cosB + cosC.

Solutia 2. R = ra ⇔ abc

4S=

S

p− a⇔ 4S2 = abc(p−a) ⇔ 4p(p−b)(p−c) = abc ⇔

(a+b+c)(a2−b2−c2+2bc) = 2abc ⇔ a(a2−b2−c2)+b(a2−b2−c2)+2b2c+c(a2−b2−c2)+2bc2 = 0 ⇔ a(b2+c2−a2) = b(a2+c2−b2)+c(a2+b2−c2) ⇔ cosA = cosB+cosC.

155

Solutia 3. Utilizand formulele ra = 4R sinA

2cos

B

2cos

C

2si cosA − cosB −

cosC = 1−4 sinA

2cos

B

2cos

C

2(Traian Lalescu-Geometria triunghiului; 16.38 si 13.5),

avem:

R = ra ⇔ 1 = 4 sinA

2cos

B

2cos

C

2⇔ cosA− cosB − cosC = 0.

Nota autorului. Problema este consistenta, ın sensul ca exista triunghiuri ıncare sa fie ındeplinite conditiile echivalente din enunt, de exemplu triunghiul isoscelsi dreptunghic ın B sau cel isoscel de laturi a = 2(

√3− 1), b = c = 2.

Clasa a X-a

X.101. Calculati suma S =nX

k=1

arctg 1√2k+1

arcsin√2k+1k+1

.

Bencze Mihaly, Brasov

Solutie. Fie xk = arcsin

√2k + 1

k + 1si yk = arctg

1√2k + 1

; evident ca xk ∈0,

π

2

,

iar yk ∈0,

π

4

. Avem ca tg xk =

sinxkÈ1− sin2 xk

=

√2k + 1

ksi tg 2yk =

2 tg yk1− tg2 yk

=

√2k + 1

k, prin urmare tg xk = tg 2yk. Cum functia tangenta este injectiva pe

0,

π

2

,

deducem ca xk = 2yk, deciykxk

=1

2, ∀k ∈ N∗. Suma ceruta este S =

nXk=1

1

2=

n

2.

X.102. Rezolvati ecuatia1

2x+ log2

x− 7

4

+

7

4= 0.

Eugen Jecan, Dej

Solutie. Consideram functia f : R →7

4,+∞

, f(x) =

1

2x+

7

4. Evident, f este

injectiva (fiind strict descrescatoare) si surjectiva, deci este inversabila, cu inversa

f−1 :

7

4,+∞

→ R, f−1(y) = − log2

y − 7

4

. Ecuatia data se scrie atunci sub

forma f(x) = f−1(x), care revine la f(x) = x (graficele functiilor f si f−1 nu se potintersecta decat pe prima bisectoare). Cum f este strict descrescatoare, iar g(x) = xeste strict crescatoare, ecuatia f(x) = x are cel mult o solutie. Se observa usor cax0 = 2 verifica ecuatia, prin urmare este unica solutie a acesteia.

X.103. Fie S,U,A trei puncte distincte. Rotind vectorul−→SA ın jurul lui S, cu

un arc α ∈ (−π, π), obtinem punctul S′; rotind apoi−→UA ın jurul lui U , cu un arc

β ∈ (−π, π), obtinem U ′, U ′ = S′. Fie M ∈ S′U ′ astfel ıncat−−→S′M = k ·

−−−→MU ′, unde

k ∈ R\0,−1. Demonstrati ca pozitia punctului M nu depinde de A atunci si numaiatunci cand k = 1, β = α± π*.

Diana Vranceanu, eleva si Dumitru Mihalache, Barlad

*Generalizare a problemei comorii din insula a lui G. Gamow, din One, Two, Three . . . Infinity.

156

Solutie. Vom nota cu x afixul punctuluiX, prin raportare la un reper arbitrar dinplanul complex si fie ω = cosα+ i sinα, τ = cosβ + i sinβ. Atunci s′ = s+ ω(a− s),

u′ = u + τ(a − u), prin urmare m =s′ + ku′

1 + k=

s+ ku+ ws− kτu+ a(ω + kτ)

1 + k.

. .

.

.

.

.

S

U

A

S M U

a

Astfel, pozitia punctului M nu depinde de A daca sinumai daca ω + kτ = 0. Cum k = 0 si S′ = U ′, nuvom putea avea ω = 0 sau τ = 0, deci ω + kτ = 0 ⇔k = −ω

τ= − cos(a − β) − i sin(α − β) ∈ R\−1 ⇔

sin(α − β) = 0 si cos(α − β) = 1 ⇔ α − β = ±π sik = 1, adica chiar cerinta problemei.

Problema comorii din insula se obtine pentru α =π

2, β = −π

2.

X.104. Fie p, la, lb, lc semiperimetrul, respectiv lungimile bisectoarelor unui tri-unghi. Determinati numerele reale α si β, ın functie de p, stiind ca solutiile ecuatieix3 − p

√3 · x2 + αx− β = 0 sunt la, lb si lc.

Catalin Calistru, Iasi

Solutie. Se stie ca l2a =4bc

(b+ c)2· p(p − a), prin urmare l2a ≤ p(p − a). Rezulta

ca l2a + l2b + l2c ≤ p(p − a+ p− b + p− c) = p2, de unde, aplicand CBS, deducem ca(la+lb+lc)

2 ≤ 3(l2a+l2b+l2c) ≤ 3p2, deci la+lb+lc ≤ p√3, cu egalitate cand la = lb = lc.

Insa, cum la, lb, lc sunt solutiile ecuatiei din enunt, avem ca la+lb+lc = p√3. Obtinem

ca triunghiul este echilateral, iar la = lb = lc =p√3

3. Din relatiile lui Viete gasim ca

α =X

lalb = p2, iar β = lalblc =p3√3

9.

X.105. Determinati cel mai mare numar real α astfel ıncat inegalitatea

sinx+ sin y

2≤ α ·

sin x+3y4 + sin 3x+y

4

2+ (1− α) sin

x+ y

2sa fie adevarata pentru orice x, y ∈ [0, π].

Marian Tetiva, BarladSolutie. Transformand sumele ın produse, inegalitatea se scrie sub forma

sinx+ y

2cos

x− y

2≤ α · sin x+ y

2cos

x− y

4+ (1− α) sin

x+ y

2.

Insa x, y ∈ [0, π], deci sinx+ y

2≥ 0 (cu egalitate cand x = y = 0 sau x = y = π),

deci ne ramane

(1) cosx− y

2≤ α cos

x− y

4+ 1− α, ∀x, y ∈ [0, π]

(pentru cazurile x = y = 0 si x = y = π, (1) se verifica direct). Observam cax− y

4∈h−π

4,π

4

i, ∀x, y ∈ [0, π], astfel ca t = cos

x− y

4parcurge intervalul

1√2, 1

,

157

iar (1) devine

(2) 2t2 − αt− 2 + α ≤ 0, ∀t ∈

1√2, 1

.

Considerand t =1√2, obtinem ca α ≤ 2 +

√2. Pentru α = 2 +

√2 relatia (2) devine

2

t− 1√

2

(t− 1) ≤ 0, ∀t ∈

1√2, 1

si este evident adevarata. Rezulta ca valoarea

maxima dorita a lui α este 2 +√2.

Clasa a XI-a

XI.101. Pentru a ∈ R∗+, calculati lim

x→∞x

1 +a

x

x− ea

.

D.M. Batinetu-Giurgiu, BucurestiSolutie. Avem:

limx→∞

x

1 +a

x

x− ea

= lim

x→∞ea · x · (ex ln(1+ a

x )−a − 1) =

= ea · limx→∞

xex ln(1+ a

x )−a − 1

x ln(1 + ax )− a

(x ln(1 +a

x)− a) =

= ea · 1 · limx→∞

x(x ln(1 +a

x)− a) = ea lim

x→∞x2ln(1 +

a

x)− a

x

=

= a2ea limx→∞

ln(1 + ax )−

ax

(ax )2

= a2ea limy→0y>0

ln(1 + y)− y

y2=

= a2ea limy→0y>0

11+y − 1

2y=

a2ea

2limy→0y>0

−y

y(1 + y)= −a2ea

2.

XI.102. Determinati functiile continue f : R → R cu proprietatea ca f(x2) −f(y2) = (x+ y)f(x− y),∀x, y ∈ R.

Gheorghe Iurea, IasiSolutie. Considerand x = y = 0, obtinem ca 0 = 2x · f(0), deci f(0) = 0. Apoi,

luand y = 0, obtinem ca f(x2) = xf(x), prin urmaref(x2)

x2=

f(x)

x, ∀x ∈ R∗. Astfel,

functia g : R∗ → R, g(x) =f(x)

xare proprietatea ca g(x2) = g(x), ∀x ∈ R2. Pentru

x > 0, rezulta ca g(x) = g(√x) = g( 22

√x) = . . . = g( 2n

√x) si, folosind continuitatea lui

g, deducem ca g(x) = limn→∞

g(x) = limn→∞

g( 2n√x) = g( lim

n→∞2n√x) = g(1), ∀x ∈ (0,∞).

Pentru x < 0, g(x) = g(x2) = g(1), deci g(x) = A, ∀x = 0. Obtinem ca f(x) = Ax,∀x = 0 si, cum f(0) = 0, avem ca f(x) = Ax, ∀x ∈ R.

XI.103. Fie (xn)n≥1 ⊂ R∗+ astfel ıncat lim

n→∞n(xn+1−xn) = a ∈ (1,+∞); definim

sirul (yn)n≥1 prin yn =n lnn

x1 + x2 + . . .+ xn. Calculati lim

n→∞(yn)

n.

Cosmin Manea si Dragos Petrica, Pitesti

158

Solutie. Cum limn→∞

n(xn+1 − xn) = a > 1, rezulta ca exista k ∈ N∗ astfel ıncat

p(xp+1−xp) > 1, ∀p ≥ k, prin urmare xp+1−xp >1

p, ∀p ≥ k. Dand lui p valorile k, k+

1, . . . , n−1 si adunand relatiile obtinute, deducem ca xn−xk >1

k+

1

k + 1+. . .+

1

n− 1,

de unde limn→∞

xn = +∞. Conform lemei Stolz-Cesaro, calculul limitei sirului (yn)n≥1

revine la calculul limitei(n+ 1) ln(n+ 1)− n lnn

xn+1=

ln1 + 1

n

nxn+1

+ln(n+ 1)

xn+1. Limita

primei fractii este1

∞= 0, iar pentru cea de-a doua folosim din nou Stolz-Cesaro.

Obtinem ca limn→∞

yn = limn→∞

ln(n+ 2)− ln(n+ 1)

xn+2 − xn+1= lim

n→∞

ln1 + 1

n+1

n+1

(n+ 1)(xn+2 − xn+1)=

1

a∈ (0, 1), prin urmare lim

n→∞(yn)

n = 0.

XI.104. Calculati limn→∞

15

51+

25

52+

35

53+ . . .+

n5

5n

.

Neculai Roman, Mircesti (Iasi)

Solutie. Notam an =nX

k=1

k5

5k, bn =

nXk=1

k4

5k, cn =

nXk=1

k3

5k, dn =

nXk=1

k2

5k, en =

nXk=1

k

5k, fn =

nXk=1

1

5k. Dezvoltand dupa formula binomului (k+1)i, i = 1, 5, ımpartind

prin 5k+1, dand valori lui k de la 1 la n si sumand, obtinem:

an+1 −1

5=

1

5(an + 5bn + 10cn + 10dn + 5en + fn)(1)

bn+1 −1

5=

1

5(bn + 4cn + 6dn + 4en + fn)(2)

cn+1 −1

5=

1

5(cn + 3dn + 3en + fn)(3)

dn+1 −1

5=

1

5(dn + 2en + fn)(4)

en+1 −1

5=

1

5(en + fn).(5)

Observam ca limn→∞

fn = limn→∞

1

5·1− 1

5n

1− 15

=1

4. Mai observam ca fiecare dintre sirurile

an, bn, . . . , en este convergent (cel mai simplu, folosind criteriul radacinii pentru serii,ınsa se pot face si majorari elementare). Trecand succesiv la limita ın relatile (5), (4),

. . . , (1), obtinem ca limn→∞

en =5

16, limn→∞

dn =15

32; limn→∞

cn =115

128, limn→∞

bn =285

128si,

ın final, limn→∞

an =3535

512.

XI.105. Consideram matricele Ak, Bk ∈ Mk(C), k ∈ N\0, 1, astfel ıncatdetAk = α ∈ C∗, ∀k ∈ N\0, 1. Studiati convergenta sirului (an)n≥2 definit prin

159

an = limx→∞

nXk=2

det(Akx+Bk)

k! · xk.

Catalin Calistru, IasiSolutie. Tinand cont de proprietatile determinantilor, avem:

an = limx→∞

nXk=2

det(Ak · x+Bk)

k! · xk=

nXk=2

limx→∞

det(Ak + 1xBk)

k!

=

=nX

k=2

detAk

k!=

nXk=2

α

k!= α

1 +

1

1!+

1

2!+ . . .+

1

n!− 2

,

prin urmare limn→∞

an = α(e− 2).

Clasa a XII-aXII.101. Rezolvati ecuatia x2 + x+ 1 = 0 ın Z13 si ın Z19, apoi deduceti ca 247

divide (37n−1 − 1)(74

n−1 − 1), oricare ar fi n ∈ N.Mihai Haivas, Iasi si I.V. Maftei, Bucuresti

Solutie. Prin verificari directe, obtinem ca ecuatia data are ın Z13 solutiile b3 sib9, iar ın Z19 are solutiile 7 si 11. Cum X3 − 1 = (X − 1) · (X2 +X + 1), rezulta cab93 = b1 (ın Z13), iar 73= 1 (ın Z19). Atuncib37n−1 − b1 = b3(7−1)(7n−1+...+7+1) − b1 = b0 ⇒ 13|37

n−1 − 1,∀n ∈ N,

74n−1 − 1 = 7

(4−1)(4n−1+...+4+1) − 1 = 0 ⇒ 19|74n−1 − 1,∀n ∈ N.

Obtinem astfel ca 13 · 19(37n−1 − 1)(74

n−1 − 1), ∀n ∈ N.

XII.102. Determinati primitivele functiei f :0,

π

4

→ R, f(x)=

cos 2x

(tg x+ ctg x)2009.

Nicoleta Bran, CraiovaSolutie. Avem caZ

f(x)dx =

Zcos 2x

sin xcos x + cos x

sin x

2009 dx =

Zcos 2x

sin2 x+cos2 xsin x cos x

2009 dx =

=

Zcos 2x

sin 2x

2

2009

dx =1

22010

Zsin2009 2x(sin 2x)′dx =

sin2010 2x

2010 · 22010+ C.

XII.103. Demonstrati ca exista c ∈0,

π

4

pentru careZ π

4

0(etg x − 1)dx ≤ π2

32· etg c

cos2 c.

Bogdan Victor Grigoriu, Falticeni

160

Solutie. Consideram functia f :h0,

π

4

i→ R, f(x) = etg x, pentru care f ′(x) =

1

cos2 xetg x > 0, iar f ′′(x) =

1 + sin 2x

cos4 xetg x > 0, ∀x ∈

h0,

π

4

i. Rezulta ca functia f este

strict crescatoare si convexa; graficul sau avand capetele A(0, 1) si Bπ4, e, ınseamna

ca Gf se afla sub dreapta AB, de ecuatie y =4(e− 1)

πx+ 1. Deducem ca etg x − 1 ≤

4(e− 1)

πx si, integrand aceasta inegalitate pe

h0,

π

4

i, obtinem ca

Z π4

0(etg x − 1)dx ≤

π(e− 1)

8. Pe de alta parte, aplicand lui f teorema lui Lagrange, gasim c ∈

0,

π

4

pentru care

f(π4 )− f(0)π4 − 0

= f ′(c), adica4(e− 1)

π=

1

cos2 c· etg c. Combinand ultimele

doua relatii, rezulta concluzia problemei.

XII.104. Determinati functiile derivabile f : [0, 1] → R pentru care f(0) = 0, iarf ′(x) = f(ax),∀x ∈ [0, 1], cu a ∈ [0, 1] fixat.

Gheorghe Iurea, Iasi

Solutie. Functia f este continua pe compactul [0, 1], deci este marginita: ∃M > 0astfel ıncat −M ≤ f(x) ≤ M. Cum f ′(x) = f(ax), ∀x ∈ [0, 1], cu a ∈ [0, 1], ınseamnaca −M ≤ f ′(x) ≤ M, ∀x ∈ [0, 1], de unde −M

R x0 dt ≤

R x0 f ′(t)dt ≤ M

R x0 dt, adica

−Mx ≤ f(x) ≤ Mx, ∀x ∈ [0, 1]. Obtinem ca −Max ≤ f ′(x) ≤ Max si, dupa o noua

integrare, rezulta ca −Max2

2≤ f(x) ≤ Ma

x2

2. Continuand procedeul, deducem

inductiv ca −M · a · a2 · . . . · an−1xn

n!≤ f(x) ≤ M · a · a2 · . . . · an−1x

n

n!, ∀n ∈ N∗,

∀x ∈ [0, 1]. Trecand la limita dupa n → ∞, obtinem ca f(x) = 0, ∀x ∈ [0, 1] si aceastafunctie verifica toate conditile din enunt.

XII.105. Fie f : (0,∞) → R o functie semiconvexa (fx+ y

2

≤ f(x) + f(y)

2,

∀x, y ∈ (0,∞)).

a) Demonstrati ca, pentru orice x ∈ (0,∞), sirul (fn(x))n≥1 definit prin fn(x) =

n

f

x+

1

n

− f(x)

este monoton.

b) Deduceti ca pentru orice x ∈ (0,∞), sirul (en(x))n≥1 definit prin en(x) =1 +

1

nx

n

este crescator.

Dan Stefan Marinescu si Viorel Cornea, Hunedoara

Solutie. a) Daca f este semiconvexa, se arata prin inductie ca fx1 + . . .+ xn

n

f(x1) + . . .+ f(xn)

n, ∀n ∈ N∗, de unde f(rx + (1 − r)y) ≤ rf(x) + (1 − r)f(y),

161

∀x, y ∈ (0,∞), ∀r ∈ Q ∩ [0, 1]. Fie x ∈ (0,∞) fixat; atunci

fn+1(x)− fn(x) = (n+ 1)

f

x+

1

n+ 1

− f(x)

− n

f

x+

1

n

− f(x)

=

= (n+ 1)

f

x+

1

n+ 1

− 1

n+ 1f(x)− n

n+ 1f

x+

1

n

≤ (n+ 1)

f

x+

1

n+ 1

− f

1

n+ 1x+

n

n+ 1

x+

1

n

= 0,

prin urmare sirul (fn(x))n≥1 este descrescator.b) Consideram functia f : (0,∞) → R, f(x) = − lg x, care este evident semicon-

vexa. Atunci, sirul de termen general fn(x) = n[lg x− lg(x+1

n)] este descrescator, de

unde sirul (−fn(x))n≥1 este crescator, unde −fn(x) = lg(1 +1

nx)n. Obtinem astfel

ca sirul (en(x))n≥1 este crescator.

Pitagora si scoala pitagorica–miracolul grecescGandirea pitagorica este un moment de rascruce ın istoria cunoasterii. Cunoasterea

empirica este ınlocuita cu o noua atitudine a gandirii ın confruntarea cu realitateaınconjuratoare: rationamentul si demonstratia. Cum s-a petrecut ın mod precis,acum peste 2500 de ani, aceasta schimbare va fi pentru totdeauna un mister. Nici unmatematician – spuneOctav Onicescu –, atat cat aceasta rasa stiintifica va continua saexiste, nu poate renunta la silinta de a ıntelege cum s-a realizat pentru ıntaia oara ıntr-un mod precis si categoric ınlantuirea logica, ce s-a chemat de atunci demonstratie.

De la Pitagora (∼580-∼500) nu s-a pastrat nimic scris. Lui Pitagora si scoliipitagorice, pe frontispiciul careia era scrisa deviza Numerele guverneaza lumea, lesunt atribuite prin traditie rezultate importante care constituie elemente fundamen-tale ale stiintei actuale.

La nord de antica Crotona (astazi Cortona), unde profesa Pitagora, pe drumul ceduce la Metapont (astazi Taranto), unde legenda spune ca a murit Pitagora, existao regiune mica cunoscuta sub numele ei latin de Terra Imaginationis. In aceastaregiune, nu departe de autostrada care serpuieste de-a lungul coastei calabreze, seafla catunul San Mathesis. Putin la stanga, ın afara satului, se gaseste o capelagotica cunoscuta sub numele de Capela Pitagora. In aceasta capela, pe podea, ın fataaltarului, se afla o lespede de marmura alba stearsa de vremuri si pasii vizitatorilor.Literele ramase din inscriptia de pe ea arata ca ın timpurile de demult legenda spunea:AICI ODIHNESC OASELE LUI PITAGORA DIN SAMOS. Un preot tine aprinse,zi si noapte, cinci candele asezate ın cinci firide ın jurul altarului, ca o dovada capentagrama (pentagonul stelat) era semnul de unire al discipolilor lui Pitagora. Acelasipreot dezvaluie vizitatorilor semnificatia acestor candele:

162

Solutiile problemelor pentru pregatireaconcursurilor propuse ın nr. 2/2009

A. Nivel gimnazialNota (Titu Zvonaru, Comanesti). Dupa solutia din RecMat 1/2008, p. 73, a

problemei G124, autorul spune ca ”dupa calcule laborioase, se poate demonstra ca

problema este echivalenta cu inegalitateama

ha≤ R

2r”. De fapt, calculele nu sunt deloc

laborioase:Din patrulaterul A′QAP ınscris ın cercul de diametru AA′, rezulta ca PQ =

AA′·sinA = ma sinA. Folosind formule uzuale, avem cap

sinA=

p · 2Ra

=ha · p · 2R

aha=

ha · p · 2R2S

= ha · Rr. Atunci 4PQ ≤ AB + BC + CA ⇔ 4ma sinA ≤ 2p ⇔ 2ma ≤

p

sinA⇔ ma

ha≤ R

2r.

G166. Demonstrati ca urmatoarele propozitii sunt adevarate.a) ∀n ∈ N, n ≥ 2, ∃x1, x2, . . . , xn ∈ N∗ astfel ıncat x1x2 + x2x3 + . . . + xnx1 =

x1x2 . . . xn.b) ∀n ∈ N, n ≥ 5, @x1, x2, . . . , xn ∈ 2N∗ astfel ıncat x1x2 + x2x3 + . . . + xnx1 =

x1x2 . . . xn.c) ∃x1, x2, . . . , xn ∈ 2N+ 1 astfel ıncat x1x2 + x2x3 + . . .+ xnx1 = x1x2 . . . xn ⇔

n ∈ 2N∗ + 1.Dan Popescu, Suceava

Solutie. a) De exemplu, consideram x1 = 3, x2 = . . . = xn−2 = 1, xn−1 = n+ 2,xn = 1, daca n ≥ 4. Pentru n = 2, x1 si x2 pot fi alese arbitrar, iar pentru n = 3putem lua x1 = x2 = x3 = 3.

b) Se demonstreaza usor (de exemplu, prin inductie) ca 2n−2 > n, ∀n ≥ 5. Dacapresupunem ca exista xi = 2yi, i = 1, n, avand proprietatea din enunt, atunci y1y2 +y2y3 + . . . + yny1 = 2n−2y1y2 . . . yn. Insa 2n−2y1y2 . . . yn > ny1y2 . . . yn > y1y2 +y2y3 + . . .+ yny1 si astfel se ajunge la o contradictie.

Concluzia se pastreaza si pentru n = 2, ınsa nu mai are loc pentru n = 3 (putemconsidera x1 = 2, x2 = x3 = 4), nici pentru n = 4 (alegem x1 = x2 = x3 = x4 = 2).

c) Daca are loc relatia din enunt, cu toate numerele x1, x2, . . . , xn impare, atuncimembrul drept, deci si cel stang, va fi impar. In stanga avem n termeni impari cusuma impara, prin urmare n va fi impar. Reciproc, daca n este impar si n ≥ 3, putemconsidera exemplele de la a).

G167. Fie 1 = d1 < d2 < . . . < dk = n toti divizorii pozitivi ai numarului naturaln. Daca exista i, j cu j > i > 13 si d27 + d2i = d2j , aratati ca n este multiplu de 8.

Titu Zvonaru, ComanestiSolutie. Se stie (sau se arata usor) ca, daca a, b, c ∈ N sunt astfel ıncat a2 =

b2 + c2, atunci abc se divide cu 60; ın cazul nostru rezulta ca d7didj...60, de unde

d1 = 1, d2 = 2, d3 = 3, d4 = 4, d5 = 5, d6 = 6 si 10|n, deci d7 ∈ 7, 8, 9, 10. Dacad7 = 7, atunci d2j − d2i = 49 ⇔ (dj − di)(dj + di) = 49 si, cum dj − di < dj + di si cele

163

doua numere au aceeasi paritate, deducem ca dj − di = 1, dj + di = 49, deci di = 24,

prin urmare n...8. Daca d7 = 8, concluzia este evidenta, iar daca d7 = 9 sau d7 = 10,

cu un rationament analog celui de mai sus, obtinem ca di = 40, respectiv di = 24.

Aceste relatii arata ca n...8, ınsa intra ın contradictie cu faptul ca d7 > 8.

G168. Pentru x, y, z ∈ R∗+, demonstrati ca are loc inegalitatea

x(y + z)

x+ yz+

y(x+ z)

y + xz+

z(x+ y)

z + xy≤ 2

x2

x+ yz+

y2

y + xz+

z2

z + xy

.

Stefan Gavril, Piatra NeamtSolutie. Fara a restrange generalitatea, putem presupune ca 0 < x ≤ y ≤ z;

atunci 0 < xy ≤ xz ≤ yz si 0 <1

z≤ 1

y≤ 1

x, deci

xy

z≤ xz

y≤ yz

x, de unde

0 <1

1 + yzx

≤ 1

1 + xzy

≤ 1

1 + xyz

, adica 0 <x

x+ yz≤ y

y + xz≤ z

z + xy. Folosind

inegalitatea rearanjarilor, rezulta ca

yx

x+ yz+ z

y

y + xz+ x

z

z + xy≤ x

x

x+ yz+ y

y

y + xz+ z

z

z + xy;

zx

x+ yz+ x

y

y + xz+ y

z

z + xy≤ x

x

x+ yz+ y

y

y + xz+ z

z

z + xy.

Adunand aceste doua relatii se obtine inegalitatea dorita.

G169. Demonstrati ca exista o infinitate de numere irationale α cu proprietateaca α3 si α2 + α sunt, de asemenea, irationale.

Gabi Ghidoveanu si Dumitru Mihalache, BarladSolutie. Orice numar α = a

√2, cu a ∈ Q∗, satisface conditiile din problema.

G170. O multime A ⊂ R, de cardinal 2009, are proprietatea ca fiecare element alei este mai mare decat o zecime din suma celor 2008 numere ramase. Aratati ca Acontine cel putin 12 numere negative.

Andrei Nedelcu, IasiSolutie. Presupunem, prin absurd, ca A contine cel mult 11 numere negative, fie

acestea a1, a2, . . . , ak, unde k ≤ 11; completam cu ınca 11 − k numere (nenegative),ak+1, . . . , a11 si fie S = a1 + a2 + . . . + a2009. Din ipoteza problemei, avem ca a1 >S − a110

, a2 >S − a210

, . . . , a11 >S − a11

10, de unde 10(a1+a2+. . .+a11) > 11S−(a1+

a2+. . .+a11), deci a1+a2+. . .+a11 > S. Rezulta de aici ca a12+a13+. . .+a2009 < 0,ceea ce nu este posibil, ıntrucat a12, a13, . . . , a2009 sunt toate nenegative. Astfel,ramane adevarata concluzia problemei.

y

x0

G171. Punctele planului care au, ın raport cu un repercartezian, ambele coordonate numere naturale, le parcurgem ınsensul indicat de sageti ın figura, pornind din origine. Notamcu an,k pozitia punctului de coordonate (n, k) ın sirul obtinut(de exemplu, a0,0 = 1, a0,2 = 4, a2,2 = 13 etc.). Exprimatinumarul an,k ın functie de n si de k.

Lucian Georges Ladunca, Iasi

164

Solutie. Pentru a ajunge din origine ın punctul de coordonate (n, k) vor fi par-curse toate punctele situate pe diagonalele perpendiculare pe prima bisectoare, pecare suma coordonatelor este mai mica decat n + k; aceste puncte sunt ın numar

de 1 + 2 + 3 + . . . + (n + k) =(n+ k)(n+ k + 1)

2. Apoi, trebuie sa ne deplasam

pe diagonala pe care suma coordonatelor este constanta, egala cu n + k. Observamca daca n + k este par, aceasta diagonala este parcursa de sus ın jos, ın ordinea(0, n + k); (1, n + k − 1); . . . ; (n − 1, k + 1); (n, k), deci mai sunt necesare n + 1 de-plasari. In cazul ın care n + k este impar, aceasta diagonala este parcursa invers,

deci vom avea nevoie de k + 1 deplasari. In concluzie, an,k =(n+ k)(n+ k + 1)

2+

1 + (−1)n+k

2n+

1 + (−1)n+k+1

2k + 1.

G172. O tabla dreptunghiulara m × n, m, n ≥ 2, are patratelele unitate de laintersectiile liniilor de ordin impar cu coloanele de ordin impar colorate ın negru,restul patratelelor fiind albe. A recolora o linie (coloana) ınseamna a schimba culoriletuturor patratelelor acelei linii (coloane). Aratati ca tabla nu poate fi transformataıntr-una complet alba prin recolorarea catorva linii si coloane.

Razvan Ceuca, elev, Iasi

Solutie. Problema este de aceeasi natura cu binecunoscutele aplicatii ale prin-cipiului invariantului prezentate, spre exemplu, ın Invarianti si jocuri de I. Boreicosi M. Teleuca, Ed. Gil, Zalau, 2007. Ne fixam atentia asupra coltului tablei aflat laintersectiile liniilor 1 si 2 cu coloanele 1 si 2. Initial, acest colt contine un patratelnegru si trei patratele albe. La o recolorare a unei linii (coloane), numarul patratelelornegre ale coltului ramane 1 sau devine 3. Cum acest numar nu ajunge niciodata sa fie0, ınseamna ca tabla nu poate fi transformata ıntr-una complet alba prin recolorare.

G173. Notam cu T (a, b, c) triunghiul care are laturile de lungimi a, b si c. Demon-strati ca triunghiurile T (b, 2c, 2mb) si T (c, 2b, 2mc) pot fi confectionate (pe rand),dintr-o aceeasi bucata de carton, fara pierdere de material, plecand de la T (a, b, c).

Petru Asaftei, Iasi

Solutie (Daniel Vacaru, Pitesti). Fie ABC un triunghi de laturi a, b,

A

BC

R

S

F

Q

P

T

E

si c, BE si CF mediane, P si Q simetricele lui B fatade E, respectiv A,R si S simetricele lui C fata de F ,respectiv A, iar T cel de-al patrulea varf al paralel-ogramului ABTC. Este clar ca BPQ este de laturib, 2c, 2mb, iarCRS este de laturi c, 2b, 2mc. DeoareceABTC si AQPC sunt paralelograme congruente, iarABE ≡ CPE, rezulta ca BPQ poate fi construitdin paralelogramul ABTC. Analog, CRS poate ficonstruit din acelasi paralelogram.

G174. Se considera triunghiul ABC isoscel cu

m( bA) = 40. Sa se arate ca nu exista puncte P ∈ IntABC pentru care m(ÕPAB) =

30, m(ÕPBC) = 10 si m(ÕPCA) = 35.

Gabriel Popa si Paul Georgescu, Iasi

165

Solutie. Caz I. m(ÒB) = m(ÒC) = 70; atunci m(ÕPAC) = 10, m(ÕPCB) = 35 side aici rezulta ca APC ≡ BPC (L.U.U.), prin urmare AC = BC, ceea ce esteimposibil, triunghiul nefiind echilateral.

Caz II. m(ÒB) = 40, m(ÒC) = 100; atunci PAB este isoscel cu m(ÕPAB) =

m(ÕPBA) = 30. Rezulta ca AP = BP si cum AC = BC, PC = PC, urmeaza ca

PAC ≡ PBC. Deducem ca ÕPCA ≡ ÕPCB, absurd.

Caz III. m(ÒB) = 100, m(ÒC) = 40; atunci m(ÕPAC) = 10, m(ÕPBA) = 90,

m(ÕPCB) = 5 si de aici m(ÕBPC) = 165. Fie AP = 2; din ABP dreptunghiccu unghi de 30, gasim ca BP = 1, AB =

√3, prin urmare BC =

√3. Cu teorema

sinusurilor ın BPC, obtinem:

BC

sinBPC=

BP

sinBCP⇔

√3

sin 165=

1

sin 5⇔ sin 15 =

√3 sin 5.

Notand t = sin 5, rezulta ca 3t − 4t3 = t√3 ⇔ t(3 −

√3 − 4t2) = 0 ⇔ t ∈¦

0,±

È3−

√3

2

©, ceea ce reprezinta o contradictie (singura valoare care ar putea

intra ın discutie ar fi sin 5 =

È3−

√3

2, ınsa

È3√3

2>

1

2= sin 30).

Nota. Dl. Titu Zvonaru, Comanesti, semnaleaza o alta modalitate prin carese pot obtine contradictii ın cele trei cazuri, anume folosirea teoremei lui Ceva subforma trigonometrica.

Spre exemplu, ın cel de-al treilea caz, din sinÕPAC ·sinÕPBA ·sinÕPCB = sinÕPAB ·sinÕPBC · sinÕPCA, ar rezulta ca sin 10 · sin 90 · sin 5 = sin 30 · sin 10 · sin 35, prinurmare sin 35 = 2 sin 5, fals.

G175. Fie ABCD un patrulater ınscris ın cercul de raza R. Demonstrati caAB ·AD + CB · CD ≤ 4R2.

Gheorghe Costovici, IasiSolutie. Notam cu P si Q proiectiile pe BD ale punctelor A, respectiv C si cu

A

B

C

P

S

C

Q

A

D

A′, C ′ punctele diametral opuse lui A, respectiv C. Avemca ABP ∼ AA′D si CBQ ∼ CC ′D, prin urmareAB · AD = AP · AA′ si CB · CD = CQ · CC ′. Adunandaceste relatii, obtinem ca AB ·AD+CB ·CD = AP ·AA′+CQ ·CC ′ = 2R(AP +CQ) ≤ 2R(AS +CS) = 2R ·AC ≤2R ·2R = 4R2, unde S = AC∩BD. Egalitatea se atingecand ABCD este patrat.

Nota. Din prima teorema a lui Ptolemeu, rezulta caAB · CD +AD ·BC = AC ·BD ≤ 2R · 2R = 4R2.

B. Nivel licealL166. Fie ABCD un dreptunghi, iar C un cerc prin A, care intersecteaza (AB),

(AC) si (AD) ın M,N , respectiv P . Aratati ca AM ·AB +AP ·AD = AN ·AC.Gheorghe Iurea, Iasi

166

Solutia 1 (a autorului). Notam α = m(ÕBAC), β = m(ÖANM). Din teo-

rema sinusurilor, avem caAM

sinβ=

AN

sin(π − α− β), deci AM = AN · sinβ

sin(α+ β),

A B

CD

N

M

O

P

S

T

Q

iarAP

sin ÕPNA=

AN

sin ÕAPN. Insa m(ÖMNP ) = 90 (unghi

ınscris ın semicerc), prin urmare m(ÕPNA) =π

2− β si

m(ÕAPN) = π−π2− β

−π2− α

= α+ β, astfel ca

AP = ANcosβ

sin(α+ β). Cum AB = AC · cosα, AD =

AC · sinα, obtinem:

AM ·AB +AP ·AD = AN · sinβ

sin(α+ β)AC · cosα+AN · cosβ

sin(α+ β)AC · sinα =

= AN ·AC · sinβ cosα+ cosβ sinα

sin(α+ β)= AN ·AC.

Solutia 2 (Titu Zvonaru). Fie O centrul cercului C. Notam cuQ,S, T proiectiilelui O pe AB,AC, respectiv AD si fie a = AB, b = BC, u = AQ, v = AT, iarr =

√u2 + v2 este raza lui C. Avem ca AM = 2AQ, AP = 2AT, AN = 2AS si

AS = r cosÕOAS = r·cos(ÕOAB−ÕBAC) = r cosÕOAB cosÕBAC+r sinÕOAB cosÕBAC =

ra√

a2 + b2u

r+ r

b√a2 + b2

v

r=

au+ bv√a2 + b2

. Relatia de demonstrat devine

AQ ·AB +AT ·AD = AS ·AC ⇔ au+ bv =au+ bv√a2 + b2

·p

a2 + b2,

deci este adevarata.Solutia 3 (Titu Zvonaru, Daniel Vacaru). Folosind puterea punctului fata

de cerc si teorema lui Pitagora, obtinem:

AM ·AB = (AB −BM) ·AB = a2 −BM ·BA = a2 − (BO2 − r2)

= a2 + u2 + v2 − [(a− u)2 + v2] = 2au.

Analog se arata ca AP · AD = 2bv iar AN · AC = 2au + 2bv, de unde concluziaproblemei.

Solutia 4 (Cristinel Mortici, Titu Zvonaru). Alegem un sistem cartezian ınraport cu care A(0, 0), B(a, 0), C(a, b), D(0, b), O(u, v). Ecuatia cercului C este (x −u)2 + (y − v)2 = u2 + v2, adica x2 + y2 − 2ux− 2vy = 0. Se obtin usor coordonatele

punctelor M,N,P , care sunt M(2u, 0), P (0, 2v), N

2a(au+ bv)

a2 + b2,2b(au+ bv)

a2 + b2

. De-

oarece

AN ·AC =pa2 + b2

Ê4a2(au+ bv)2 + 4b2(au+ bv)2

(a2 + b2)2= 2au+ 2bv,

re;atia de demonstrat este adevarata.

167

Nota (Cristinel Mortici). Problema poate fi privita ca o extindere a teoremeilui Pitagora, care se obtine cand cercul C este cel circumscris dreptunghiului dat.

L167. Fie ABC un triunghi cu AB > AC. Cercul ınscris ın triunghi este tangentlaturilor BC si AC ın D, respectiv E. Consideram T un punct pe latura [BC] si notamcu J centrul cercului ınscris ın ABT. Daca DE trece prin mijlocul segmentului [CJ ],demonstrati ca triunghiul ATC este isoscel.

Titu Zvonaru, Comanesti

Solutie. Fie M = AJ ∩ BT, P = DE ∩ CJ, x = AT, y = BM. Cu notatiileuzuale, avem: CE = CD = p− c, AE = p− a, BD = p− b, iar MD = BD −BM =

p − b − y. Din teorema bisectoarei, obtinem caBM

MT=

AB

AT, deci MT =

xy

c, iar

AJ

JM=

AT

MT, de unde

AJ

AM=

c

y + c, iar

JM

AM=

y

y + c. Folosind relatia (R1) din

RecMat1/2005, pg. 15, avem caCP

PJ=

AM · CEEA · CD

DM

AJ · CEEA + JM · CD

DM

si, cum P este mijlocul

lui CJ, obtinem succesiv:

AJ

AM· EC

EA+

JM

AM· DC

DM=

EC

EA· DC

DM⇔ c

y + c· p− c

p− a+

+y

y + c

p− c

p− b− y=

p− c

p− a

p− c

p− b− y⇔ pc− bc− cy + py − ay = py + pc− cy − c2

⇔ y =c(b− c)

a, iar BT =

(c− b)(c+ x)

a.

Cu teorema cosinusului ın ABT, deducem:

x2 = c2 +(c− b)2(c+ x)2

a2− 2c

(c− b)(c+ x)2

a2− 2c

(c− v)(c+ x)

a

a2 + c2 − b2

2ac⇔

⇔ a2x2 = a2c2 + (c− b)2(c2 + 2cx+ x2)− (c− b)(a2 + c2 − b2)(c+ x) ⇔⇔ x2[a2 − (c− b)2] + (c− b)[a2 + c2 − b2 − 2c2 + 2bc]x− a2c2 − c2(c2 − 2bc+ b2)+

+ (c2 − 2bc)(a2 + c2 − b2) = 0 ⇔ x2[a2 − (c− b)2]− (b− c)[a2 − (c− b)2−− bc[a2 − (c− b)2] = 0 ⇔ x2 − (b− c)x− bc = 0.

Aceasta ecuatie are unica solutie convenabila AT = b = AC.

L168. Demonstrati ca ın orice triunghi, cu notatiile uzuale, are loc inegalitatea

a

b+ c+

b

c+ a+

c

a+ b≥ 11p2 − 15r2 − 60Rr

6p2 − 6r2 − 24Rr≥ 3

2.

Marius Olteanu, Rm. Valcea

Solutia 1 (a autorului). Este cunoscuta identitatea a2+b2+c2 = 2p2−2r2−8Rr(Traian Lalescu-Geometria triunghiului, 16.54), de unde rezulta imediat ca ab+ bc+ca = p2 + r2 +4Rr. Scazand aceste doua relatii, obtinem ca 3r2 +12Rr = p2 − (a2 +

168

b2 + c2) + (ab + bc + ca) ≤ p2 si astfel este demonstrata inegalitatea din dreapta.Pentru prima inegalitate, pornim de la

x

y + z+

y

z + x+

z

x+ y≥ 13

6− 2(xy + xz + yz)

3(x2 + y2 + z2), ∀x, y, z > 0

(vezi, de exemplu, V. Cartoaje-Algebraic Inequalities, GIL, Zalau, 2006). Inlocuindx = a, y = b, z = z, x2 + y2 + z2 = 2p2 − 2r2 − 8Rr, xy + xz + yz = p2 + r2 + 4Rr siefectuand calculele, urmeaza concluzia problemei.

Solutia 2 (Titu Zvonaru, Comanesti). Are loc egalitatea:

11p2 − 15r2 − 60Rr

6p2 − 6r2 − 24Rr=

26p2 − 15(p2 + r2 + 4Rr)

12p2 − p(p2 + r2 + 4Rr)=

26p2 − 15(Σab)

12p2 − 6(Σab)=

=13(Σa)2 − 30(Σab)

6(Σa)2 − 12(Σab)=

13(Σa2)− 4(Σab)

6(Σa2)=

3

2+

2(Σa2)− 2(Σab)

3(Σa2)

si, cum Σa2 ≥ Σab, inegalitatea din dreapta este demonstrata. Pentru partea stanga,

ar trebui sa aratam ca Σa

a+ b− 3

2≥ Σ(a− b)2

3(Σa2), ınsa vom demonstra o inegalitate

mai tare, anumeX a

b+ c− 3

2≥

X(a− b)2

3(X

a2)+

X(a+ b− c)2(a+ b)

6(a+ b)(b+ c)(c+ a)(X

a2)⇔

(∗)X (a− b)2

2(a+ b)(b+ c)≥

X(a− b)2

3(X

a2)+X (a+ b− c)2

6(a+ c)(b+ c)(X

a2).

Avem:(a− b)2

3(a+ c)(b+ c)− (a− b)2

3a2 + 3b2 + 3c2− (a+ b− c)2

6(a+ c)(b+ c)(a2 + b2 + c2)=

(a−b)2 · 3a2 + 3b2 + 3c2 − 2c2 − 2ab− 2bc− 2ac− a2 − b2 − c2 − 2ab+ 2ac+ 2bc

6(a+ c)(b+ c)(a2 + b2 + c2)

(a− b)22a2 + 2b2 − 4ab

6(a+ c)(b+ c)(a2 + b2 + c2)≥ 0,

prin urmare are loc (*) si solutia problemei este completa.Remarcam ca, ın locul inegalitatii (*), putem demonstra ca

(∗∗)X a

b+ c− 3

2≥

X(a− b)2

3(X

a2)+X (a− b)2

3(a+ c)(b+ c)(X

a2).

Sa mai spunem ca identitateaX a

b+ c− 3

2=X (a− b)2

2(a+ c)(b+ c)se poate verifica

direct; o modalitate de a o obtine este prezentata ın RecMat 2/2008, p. 125.

L169. Care este probabilitatea ca razele cercurilor exınscrise unui triunghi, alesaleator, sa fie laturile unui nou triunghi?

Petru Minut, Iasi

169

Solutie. Alegand convenabil unitatea de masura, putem considera ca lungimilelaturilor triunghiului sunt a = 2x, b = 2y, c = 2, cu 0 < x ≤ y ≤ 1 si x + y > 1.

Razele cercurilor exınscrise sunt ra =S

p− a=

S

−x+ y + 1, rb =

S

p− b=

S

x− y + 1

A

BC

O

si rc =S

p− c=

S

x+ y − 1; se observa ca 0 < ra ≤ rb ≤ rc. In

aceste conditii, ra, rb si rc sunt laturile unui triunghi daca sinumai daca

ra + rb > rc ⇔1

−x+ y + 1+

1

x− y + 1>

1

x+ y − 1⇔

⇔ x2 − 2xy + y2 + 2x+ 2y − 3 > 0.

Consideram un reper cartezian xOy. Domeniul cazurilor favorabile este multimeaD a solutiilor sistemului de inecuatii 0 < x ≤ y ≤ 1, x + y > 1, deci este interiorultriunghiului ABC, unde A(1, 0), B(1, 1) si C(0, 1). Domeniul cazurilor favorabileeste multimea d a punctelor interioare triunghiului ABC si exterioare parabolei P :x2−2xy+y2+2x+2y−3 = 0 (care trece prin A si prin C). Aria suprafetei cuprinsa

ıntre P si axe este

Z 1

0(x− 1 + 2

√1− x)dx =

5

6si atunci aria(d) = 1− 5

6=

1

6. Cum

aria(D) =1

2, rezulta ca probabilitatea ceruta este P =

aria(d)

aria(D)=

1

3.

L170. Fie n ∈ N, n ≥ 2 si a1, a2, . . . , an ∈ R+ cu a1 + a2 + . . . + an = S.Consideram k ∈ N, 1 ≤ k ≤ n − 1 si α1, α2 ∈ R+ cu α1 + α2 = 1. DemonstratiinegalitateaX

1≤i1<...<ik≤n

(ai1 + ai2 + . . .+ aik)α1(S − ai1 − . . .− aik)

α2 ≤ kα1(n− k)α2

nCk

nS.

(In legatura cu 6117 din R.M.T. 1/1987).Paul Georgescu si Gabriel Popa, Iasi

Solutie. Notam cu T membrul stang al inegalitatii din enunt si observam ca

T = kα1 · (n− k)α2

X1≤i1<...<ik≤n

ai1 + ai2 + . . .+ aikk

α1

·S − ai1 − . . .− aik

n− k

α2

.

Aplicand inegalitatea mediilor generalizata, obtinem ca

T ≤ kα1(n− k)α2

X1≤i1<...ik≤n

α1

ai1 + . . .+ aikk

+ α2S − ai1 − . . .− aik

n− k

=

= kα1(n− k)α2

α1

CknS

n++α2

Cn−kn S

n

= kα1(n− k)α2

CknS

n,

deoarece Ckn = Cn−k

n , iar α1 + α2 = 1.

Problema 6117 din R.M.T. se obtine pentru k = 1, n = 3, α1 = α2 =1

2.

170

L171. Pentru x, y ∈ R∗+, demonstrati ca are loc inegalitatea

√x+ 3

È2(x+ y) +

√y ≤ 2(

p3x+ y +

px+ 3y).

Marian Tetiva, BarladSolutia 1 (a autorului). Inegalitatea se poate scrie ın forma echivalenta

2(2È2(x+ y)−

p3x+ y −

px+ 3y) ≤

È2(x+ y)−

√x−√

y.

Daca amplificam cu expresiile conjugate, gasim caÈ2(x+ y)−

√x−√

y =(√x−√

y)2È2(x+ y) +

√x+

√y;

2È2(x+ y)−

p3x+ y −

px+ 3y =

(√3x+ y −

√x+ 3y)2

2È2(x+ y) +

√3x+ y +

√x+ 3y

=

=4(x− y)2

(2È2(x+ y) +

√3x+ y +

√x+ 3y)(

√3x+ y +

√x+ 3y)2

,

astfel ca demonstrarea inegalitatii noastre revine la a arata ca

8(È2(x+ y) +

√x+

√y)(

√x+

√y)2 ≤

≤ (2È2(x+ y) +

p3x+ y +

px+ 3y)(

p3x+ y +

px+ 3y)2.

Aceasta din urma inegalitate rezulta din

2(√x+

√y) ≤

p3x+ y +

px+ 3y

(care, dupa ridicare la patrat, duce la (x− y)2 ≥ 0).Solutia 2 (Titu Zvonaru, Comanesti). Datorita omogenitatii, putem presupune

ca x + y = 1. Notam 2√xy = t si atunci

√x +

√y =

Èx+ y + 2

√xy =

√1 + t, iar

3x2 +3y2 +10xy = 3(x+ y)2 +4xy = 3+ t2. Prin ridicari la patrat ale unor cantitatipozitive, inegalitatea de demonstrat se scrie succesiv:

√1 + t+ 3

√2 ≤ 2(

p3x+ y +

px+ 3y) ⇔ 3 + t+ 6

√2√1 + t ≤ 8

p3 + t2 ⇔

⇔ 9 + 6t+ t2 + 72(1 + t) + 12√2(3 + t)

√1 + t ≤ 192 + 64t2 ⇔

⇔ 4√2(t+ 3)

√t+ 1 ≤ 21t2 − 26t+ 37 ⇔

(32t+ 32)(t2 + 6t+ 9) ≤ (21t2 − 26t+ 37)2 ⇔⇔ 441t4 − 1124t3 + 2006t2 − 2404t+ 1081 ≥ 0 ⇔⇔ (t− 1)2(441t2 − 242t+ 1081) ≥ 0,

care este adevarata, ıntrucat cea de-a doua paranteza are discriminantul negativ.Egalitatea se atinge daca si numai daca x = y.

171

L172. Fie P ∈ Q[X], cu grP = n ≥ 1. Daca P admite o radacina complexa a,avand ordinul de multiplicitate m, cu n < 2m, demonstrati ca a ∈ Q.

Adrian Reisner, ParisSolutie. Fie A ∈ Q[X] polinomul minimal al lui a peste Q, avand gradul p ≥ 1.

Avem ca p = 1 ⇔ a ∈ Q, deoarece ın acest caz A = X − a. Presupunem, prinabsurd, ca a /∈ Q, deci p ≥ 2. Numarul a nu poate fi radacina multipla a lui A,deoarece altfel polinomul A′, de grad p− 1 < p ar admite radacina a si A nu ar maifi minimal. Rezulta ca A are o radacina complexa b = a. Cum toate polinoameleP, P ′, . . . , P (m−1) au radacina a, ele se divid cu A, deci admit si radacina b si astfeldeducem ca multiplicitateam′ a radacinii b a polinomului P verificam′ ≥ m. Obtinemca n ≥ m + m′ ≥ 2m, ceea ce contrazice ipoteza 2m > n. Prin urmare, ramaneadevarata concluzia problemei.

L172. a) Exista functii f : (a, b) → R cu proprietatea ca |f(x) − f(y)| ≥ c,∀x = y ∈ (a, b), unde c este o constanta pozitiva?

b) Exista functii f : (a, b)∩Q → R cu proprietatea ca |f(x)− f(y)| ≥ c, ∀x = y ∈(a, b) ∩Q, unde c este o constanta pozitiva?

Geanina Havarneanu, IasiSolutie. a) Vom arata ca nu exista astfel de functii prin reducere la absurd. Din

densitatea lui Q ın R rezulta ca oricare ar fi x ∈ (a, b), putem gasi numere rationale

(si fie q(x) un astfel de numar, fixat odata cu x) astfel ıncat |f(x)− q(x)| < c

3. Daca

presupunem ca functia f are proprietatea din enunt, pentru x = y din (a, b) obtinemca

|q(x)− q(y)| = |q(x)− f(x) + f(x)− f(y) + f(y)− q(y)| ≥

≥ |f(x)− f(y)| − |f(x)− q(x)| − |f(y)− q(y)| ≥ c

3⇒ q(x) = q(y).

Rezulta ca functia q : (a, b) → Q este injectiva, ceea ce este imposbil, ıntrucat Q estenumarabila, ın timp ce (a, b) este multime nenumarabila.

b) Cum Q este numarabila, exista o functie bijectiva g : Q → N. Pentru c > 0fixat arbitrar, definim f : (a, b) ∩ Q → R, f(x) = cg(x) si aceasta functie verificacerinta problemei: daca x = y, atunci g(x) = g(y) si deoarece g(x), g(y) ∈ N, rezultaca |g(x)− g(y)| ≥ 1, prin urmare |f(x)− f(y)| ≥ c.

Nota. Principial aceeasi solutie a fost data de catre dl. Cristinel Mortici,Targoviste.

L173. Fie a1, a2, . . . , an, b1, b2, . . . , bn numere reale pozitive si a =

nX

i=1

√ai

!2

,

b =

nX

i=1

√bi

!2

. Aratati ca exista x0 > 0 astfel ıncat

"nX

i=1

paix+ bi

#− [

√ax+ b] ∈

0, 1, ∀x > x0.Marian Tetiva, Barlad

Solutia 1 (a autorului). Fie functia f : [0,∞)→R, f(x)=

nX

i=1

paix+ bi

!2

−ax.

172

Ea este derivabila, cu f ′(x)=

nX

i=1

paix+ bi

! nX

i=1

ai√aix+ bi

!−a≥

nX

i=1

√ai

!2

−a

= 0, ∀x ≥ 0 (am utilizat inegalitatea Cauchy-Buniakowski-Schwartz), deci f estecrescatoare pe [0,∞). Urmeaza ca f(0) ≤ f(x) ≤ lim

x→∞f(x), ∀x ∈ [0,∞). Dar f(0) =

b, iar limx→∞

f(x) = limx→0x>0

f

1

x

= lim

x→0x>0

(nX

i=1

pai + bix)

2 − a

x= lim

x→0x>0

nX

i=1

pai + bix

·

nX

i=1

bi√ai + bix

!=

nX

i=1

√ai

! nX

i=1

bi√ai

!= c. Avem deci b ≤ f(x) ≤ c ⇔

√ax+ b ≤

nXi=1

paix+ bi ≤

√ax+ c, ∀x ≥ 0. Deoarece lim

x→∞(√ax+ c−

√ax+ b) = 0,

exista x0 > 0 astfel ıncat 0 <√ax+ c−

√ax+ b < 1, ∀x > x0. Notand n = [

√ax+ b],

obtinem ca n ≤nX

i=1

paix+ bi < n+ 2, ∀x > x0, adica

"nX

i=1

paix+ bi

#∈ n, n+ 1

si problema este astfel rezolvata.

Solutia 2 (Cristinel Mortici, Targoviste). Notam sn(x) =nX

i=1

paix+ bi, x >

0. Vom demonstra ca

(∗) sn(x)−√ax+ b ≥ 0,∀x ≥ 0 si lim

x→∞(sn(x)−

√ax+ b) = 0.

Daca vom dovedi (*), din sn(x) ≥√ax+ b, ∀x ≥ 0, va rezulta ca [sn(x)] ≥ [

√ax+ b],

∀x ≥ 0. Cum limx→∞

(sn(x) −√ax+ b) = 0, exista x0 > 0 astfel ıncat 0 ≤ sn(x) −

√ax+ b < 1, ∀x ≥ x0. In cazul ın care ar exista x ≥ x0 pentru care [sn(x)] ≥

[√ax+ b] + 2, numerele sn(x) si

√ax+ b ar diferi cu mai mult de 1, contradictie.

Ramane ca [sn(x)]− [√ax+ b] ∈ 0, 1, ∀x ≥ x0, deci are loc cerinta problemei.

Sa justificam deci (*). Cu inegalitateaÈu21 + v21 + . . .+

Èu2n + v2n ≥

È(u1 + . . .+ un)2 + (v1 + . . .+ vn)2,

obtinem ca sn(x) ≥È(√a1x+ . . .+

√anx)2 + (

√b1 + . . .

√bn)2 =

√ax+ b. Pentru

demonstrarea limitei, vom folosi o consecinta a formulei lui Taylor:p1 + y = 1 +

1

2y +O(y2).

In cazul nostru,

sn(x)−√ax+ b =

√a1x ·

r1 +

b1a1x

+ . . .+√anx ·

r1 +

bnanx

−√ax

r1 +

b

ax=

173

=√a1x

1 +

b12a1x

+O

1

x2

+ . . .+

√anx

1 +

bn2anx

+O

1

x2

−√ax

1 +

b

2ax+O

1

x2

=

=b1

2√a1x

+ . . .+bn

2√anx

− b

2√ax

+O

1

x√x

→ 0, pentru x → ∞

si cu aceasta solutia problemei este completa.

L175. Aratati ca[n2 ]Xk=0

C2kn Ωk = 2nΩn, n ∈ N,

unde Ωk =(2k − 1)!!

(2k)!!, k ∈ N∗ (se convine ca Ω0 = 1).

Gheorghe Costovici, Iasi

Solutie. Se stie ca

Z π2

0cosk xdx =

Z π2

0sink xdx =

8<: (k − 1)!!

k!!π2 , k par,

(k − 1)!!

k!!, k impar.

Avem, tinand seama de aceste relatii, urmatoarele:

π

2Ωn =

π

2

(2n− 1)!!

(2n)!!=

Z π2

0cos2n xdx =

1

2n

Z π2

0(1 + cos 2x)ndx =

=1

2n+1

Z π

0(1 + cos t)ndt =

1

2n+1

Z π2

0(1 + cos t)ndt+

Z π

π2

(1 + cos t)ndt

=

=1

2n+1

Z π2

0(1 + cos t)ndt+

Z π2

0(1− sinu)ndu

=

=1

2n+1

nXk=0

Ckn

Z π2

0cosk tdt+ (−1)k

Z π2

0sink udu

=

=1

2n+1

nXk=0

[1 + (−1)k]Ckn

Z π2

0cosk tdt =

=1

2n

[n2 ]Xk=0

C2kn

Z π2

0cos2k tdt =

1

2nπ

2

[n2 ]Xk=0

C2kn Ωk,

de unde deducem identitatea din enunt.

174

Probleme propuse1

Clasele primareP.196. Mioara aranjeaza patru margele, doua albe si doua galbene, una langa

alta, pentru a confectiona o salba papusii sale. In cate feluri poate aranja Mioaramargelele?(Clasa I ) Inst. Maria Racu, Iasi

P.197. Dan ımparte o ciocolata astfel: o linie ıntreaga de 4 patratele pentrufratele sau, o coloana ıntreaga de 4 patratele pentru sora sa, iar restul pentru sine.Cate patratele de ciocolata i-au revenit lui Dan?(Clasa I ) Maria Ursu, eleva, Iasi

P.198. Numarati figurile geometrice din desenul alaturat si scrieti:a) numarul triunghiurilor;b) numarul figurilor geometrice care sunt patrate sau dreptunghiuri.

(Clasa a II-a) Andreea Amarandei, studenta, Iasi

P.199. Daca a− 15 = 15− b, poate fi diferenta a− b un numar impar?(Clasa a II-a) Andreea Bızdıga, eleva, Iasi

P.200. Dintr-un numar de 12 bile, 11 au mase egale, iar una are o masa mai maredecat a celorlalte. Care este cel mai mic numar de cantariri prin care se poate depistabila cu masa mai mare?(Clasa a III-a) Ana Cojocaru, Iasi

P.201. Diferenta dintre suma varstelor a doua persoane si diferenta lor este 20 deani. Triplul sumei varstelor este egal cu de 7 ori diferenta varstelor. Care este varstafiecarei persoane?(Clasa a III-a) Valeria Avasılcei, Iasi

P.202. Aflati numerele ab si c astfel ıncat, daca marim cifrele a si b cu cate ounitate, atunci produsul ab · c se dubleaza.(Clasa a III-a) Amalia Munteanu, eleva, Iasi

P.203. Cei doi tigri de la Zoo au suficienta hrana pentru 3 saptamani. Au maifost adusi, ınsa, ınca 5 tigri. Daca fiecare mananca aceeasi cantitate de hrana pe zi,cate zile le va mai ajunge acum hrana?(Clasa a IV-a) Inst. Laura Chirila, Iasi

P.204. Aflati toate numerele naturale n astfel ıncat triplul predecesorului lui nnu depaseste dublul succesorului lui n.(Clasa a IV-a) Andreea Simion, eleva, Iasi

P.205. Fie S suma a zece numere naturale nenule.a) Daca S = 54, aratati ca exista cel putin doua numere egale.b) Aflati zece numere naturale distincte pentru care S = 57. Cate posibilitati

sunt? Justificati!(Clasa a IV-a) Constanta Tudorache si Nelu Tudorache

1Se primesc solutii pana la data de 31 iunie 2011.

175

Clasa a V-aV.123. Un spiridus ıi sopteste Ioanei: ”Pentru a salva lumea de rau, prepara o

potiune din praf de aur si praf de stele. In camera albastra este un dulap urias, cunumeroase rafturi, fiecare continand cate 20 de sticlute numerotate: de la 1 la 20 peprimul raft, de la 21 la 40 pe al doilea etc. Numarul raftului cu sticluta cu praf de aureste egal cu numarul sticlutei cu praf de stele. Suma dintre numarul sticlutei cu prafde aur si numarul sticlutei cu praf de stele este 243”. Care sunt numerele sticlutelorpe care trebuie sa le aleaga Ioana?

Cristina Timofte, Iasi

V.124. Aratati ca numarul A = 100

7

1 · 2+

7

2 · 3+ . . .+

7

99 · 100

+589

1

1 · 3+

1

3 · 5+ . . .+

1

587 · 589

este cel mai mare numar natural de trei cifre distincte.

Anca Chiritescu, Tiganasi (Iasi)

V.125. Se considera numarul a = 1 + 2 + 3 + . . . + 2010 si multimea M = n ∈N|na = k2, k ∈ N. Determinati cele mai mici cinci elemente ale lui M .

Nicolae Ivaschescu, Craiova

V.126. Se considera multimea A = 1, 2, 3, . . . , 2010. Scrieti multimea A careuniune a trei multimi disjuncte doua cate doua, avand acelasi cardinal si aceeasisuma a elementelor.

Mirela Marin, Iasi

V.127. Determinati numerele naturale A pentru care A + S(A) = 2010. (Amnotat cu S(A) suma cifrelor numarului A.)

Catalin Budeanu, Iasi

V.128. Reconstituiti o ımpartire, stiind ca ımpartitorul, catul si restul sunt cifreale deımpartitului.

Ioan Sacaleanu, Harlau

V.129. Fie a, b, c trei numere impare, iar A = 2a+b2 · 3 a+c

2 · 5 b+c2 . Stiind ca 30A

nu este patrat perfect, aratati ca macar unul dintre a, b, c nu este patrat perfect.Andrei Nedelcu, Iasi

Clasa a VI-aVI.123. Stabiliti ın cate moduri ıl putem scrie pe 2010 ca suma de trei numere

naturale nenule, direct proportionale cu trei numere naturale consecutive.Mirela Obreja si Ioan Lungu, Vaslui

VI.124. Intr-o duminica, bunica face clatite pentru nepoti; 40% dintre clatitesunt cu gem, iar restul cu ciocolata. In duminica urmatoare, bunica face cu 10% maimulte clatite cu gem si cu 5% mai putine clatite cu ciocolata. In care dintre duminicia facut bunica mai multe clatite?

Doru Turbatu, Iasi

VI.125. Determinati restul ımpartirii prin 2010 a numarului A = 12011 +22011 +. . .+ 20112011.

Andrei Pasa, elev, Iasi

176

VI.126. Pe tabla sunt scrise numerele 1, 2, 3, . . . , 2010. Andrei sterge de petabla doua numere, ınlocuindu-le cu media lor aritmetica si procedeaza astfel ın modrepetat, pana cand pe tabla ramane un singur numar. Este posibil ca acest ultimnumar sa fie 2009, 25?

Gabriel Popa, Iasi

VI.127. Consideram punctele coliniare distincte A,B,C,D si E astfel ıncat AB =b, AC = a, BD = b− a (2a < b < 3a), E este simetricul lui B fata de D, iar mijloculsegmentului [AC] este punctul E. Aflati numerele a si b, stiind ca BD = 6.

Matei Havarneanu, elev, Iasi

VI.128. Se considera triunghiul isoscel ABC cu m( bA) = 120. Perpendiculara ın

A pe AC intersecteaza bisectoarea unghiului ÒC ın F si latura BC ın E. Paralela prinE la AB taie CF ın M si notam P = AM ∩ BC. Determinati masura unghiuluiÕAPB.

Gabriela Popa, Iasi

VI.129. Se considera triunghiul ABC. Determinati un punct M pe latura [AC],aflat la egala distanta de varful A si de dreapta BC.

Mihaela Cianga, Iasi

Clasa a VII-aVII.123. Determinati numerele abc, scrise ın baza 10, pentru care numarul A =Èabc−

pabc− 28 este natural.

Vasile Chiriac, Bacau

VII.124. Demonstrati ca nu exista numere naturale n pentru care numarul A =√n+

√n+ 2010 sa fie patrat perfect.

Neculai Stanciu, Buzau

VII.125. Demonstrati ca numarul A = 2009 · 20112010 − 2010 · 20112009 + 1 estedivizibil cu 20102.

Tamara Culac, Iasi

VII.126. Daca a, b, c sunt lungimile laturilor unui triunghi, iar p este semiperime-

trul acestuia, demonstrati caa

p− a+

b

p− b+

c

p− c≥ 6.

Ionel Nechifor, Iasi

VII.127. Se considera triunghiul ABC, punctul D pe latura (BC) si romburileBDEF si CDGH cu E,G ∈ (AD), astfel ıncat A,F si H sunt de aceeasi parte adreptei BC, iar AD separa F si H. Demonstrati ca dreptele AD,BH si CF suntconcurente.

Dan Popescu, Suceava

VII.128. Pe latura (AB) a triunghiului ascutitunghic ABC se considera punctulM , iar pe segmentul (CM) punctul N , astfel ıncat BNM ∼ CAM. Demonstratica AN ⊥ BC.

Constantin Apostol, Rm. Sarat

VII.129. Se considera triunghiul ABC cu BC > AC > AB si punctele D,E pelatura (BC) si F pe (AC), astfel ıncat AB = BD = AF, iar AC = CE. Dreapta

177

BF intersecteaza AD si AE ın G, respectiv H. Aratati ca punctele D,E,G si H suntconciclice.

Daniela Bruma, Deleni (Iasi)

Clasa a VIII-aVIII.123. Rezolvati ın R ecuatia 2x2 + 2y2 − 2xy + 2x− 10y + 14 = 0.

Ionica Marcovschi, Pascani

VIII.124. Demonstrati ca numarul

A =(24 + 22 + 1)(34 + 32 + 1) . . . (20104 + 20102 + 1)

3(20102 + 2010 + 1)

este patrat perfect.Bianca Maria Filip, eleva, Iasi

VIII.125. Determinati numerele ıntregi n pentru carepn2 + 3n+ 9

=pn2 + 3n+ 9.

Ionut Stroe, student, Iasi

VIII.126. Fie a, b, c, p ∈ N∗ cu a, b, c distincte si√p ∈ R\Q, iar A = a√p, b

√p,

c√p. Functia f : A → N este definita prin f(x) = x2 + (2c − a− b)x − b − 1. Daca

x, y ∈ A, x < y, demonstrati ca f(x) < f(y).Cosmin Manea si Dragos Petrica, Pitesti

VIII.127. Determinati numerele reale x, y si z, stiind ca 4(x − 1)y2z2 + 4(y −1)z2x2 + 4(z − 1)x2y2 = 3x2y2z2.

Lucian Tutescu si Mariana Marculescu, Craiova

VIII.128. Daca cercurile ınscrise ın trei fete ale unui tetraedru sunt tangentedoua cate doua, atunci cercul ınscris ın a patra fata este tangent primelor trei.

Mihaly Bencze, Brasov

VIII.129. Sa se arate ca din fetele unui cub de muchie l, confectionat din carton,putem construi, fara resturi, fetele a sase cuburi de muchii l1, l, . . . , l6 astfel ıncatl2 = l21 + l22 + . . .+ l26.

Petru Asaftei, Iasi

Clasa a IX-a

IX.111. Let a, b, c be positive real numbers. Prove thata2 + 1

b+ c+b2 + 1

c+ a+c2 + 1

a+ b≥

3.Pedro H.O. Pantoja, Brazil

IX.112. Aratati ca pentru fiecare numar natural n ≥ 2, exista numerele naturale

x1 < x2 < . . . < xn astfel ıncat1

x1+

1

x2+ . . .+

1

xn=

1

2010.

Radu Sava, Iasi

178

IX.113. Coardele AB si CD ale unui cerc de centru O sunt perpendiculare sise intersecteaza ın P . Daca E si F sunt mijloacele segmentelor AC, respectiv BD,aratati ca PE = OF .

Petru Asaftei, Iasi

IX.114. a) Daca O este punctul de intersectie a diagonalelor patrulaterului convexABCD, atunci AB∥CD daca si numai daca AO2+DO2+BC2 = BO2+CO2+AD2.

b) Presupunem ca AB∥CD si notam cu r1, r2 razele cercurilor ınscrise ın tri-unghiurile AOD, respectiv BOC si cu R1, R2 razele cercurilor circumscrise acestortriunghiuri. Aratati ca AD = BC ⇔ r1 = r2 ⇔ R1 = R2.

Claudiu Stefan Popa, Iasi

IX.115. Trei ceviene concurente ımpart un triunghi ABC ın sase triunghiuri maimici, avand razele cercurilor circumscrise R1, R2, . . . , R6. Daca R = R1+R2+. . .+R6,

demonstrati ca macar doua dintre numerele R1, R2, . . . , R6 sunt cel mult egale cu1

6R.

Marius Dragan, Bucuresti

Clasa a X-aX.111. Rezolvati ecuatia [log1+|x|(

√1 + x2 + |x|)] · [log√1+x2+|x|(1 + |x|)] = a,

unde a este un parametru real, iar [t] este partea ıntreaga a lui t.Stefan Gavril, Piatra Neamt

X.112. Se considera multimile finite X,Y si Z astfel ıncat Z ⊂ Y, |X| = 4,|Y | ≥ 5 si |Z| = 3. Determinati |Y |, stiind ca numarul functiilor f : X → Y a carorimagine include Z este 108.

Mihai Haivas si Constantin Chirila, Iasi

X.113. Fie a, b, x1, x2, . . . , xn numerele reale strict pozitive (n ∈ N, n ≥ 2) si σ opermutare a multimii 1, 2, . . . , n. Demonstrati ca

ax1 +b

xσ(1)

. . .

axn +

b

xσ(n)

≤ax1 +

b

x1

. . .

axn +

b

xn

.

Dan Nedeianu, Drobeta Tr. Severin

X.114. Fie z1, z2, z3 ∈ C∗ astfel ıncat |z1| = |z2| = |z3| = 1 si z1 − z2z3, z2 − z3z1si z3 − z1z2 sunt numere reale. Demonstrati ca (z1 − z2)(z2 − z3)(z3 − z1) = 0.

Ionut Ivanescu, Craiova

X.115. Fie ABCD un patrulater convex si punctele P,Q ∈ AC, R,S ∈ BD astfel

ıncatPA

PC=

QA

QC=

AB

CD=

RB

RD=

SB

SD. Daca M si N sunt mijloacele segmentelor

PQ, respectiv RS, demonstrati ca 2MN < PQ+RS.Titu Zvonaru, Comanesti

Clasa a XI-aXI.111. Demonstrati ca 4(x3 + y3) ≥ 9xy|x− y|, ∀x, y ∈ [0,∞).

Lucian Tutescu, Craiova

179

XI.112. Fie sirurile (wn)n≥1 si (pn)n≥1, unde wn =

2n!!

(2n− 1)!!

2· 1

2n+ 1, ∀n ∈

N∗ (sirul lui Wallis), iar pn este al n-lea numar prim. Calculati limn→∞

2wn

π

pn

ln2wnπ

·lnnn

.

Gabriel Mırsanu, Iasi

XI.113. Calculati limita sirului (xn) definit prin: x1 ∈ (0,∞), α ∈ (0, 1)∪ (1,∞),xn+1 = −1 + (1 + αxn)

1/α, ∀n ≥ 1.Gheorghe Costovici, Iasi

XI.114. Fiind date a, b, α, β ∈ R, determinati functiile f : R → R cu proprietatea: x− α y − αf(x)− β f(y)− β

= af(x) + bf(y), ∀x, y ∈ R.

Marius Tiba, elev, Bucuresti

XI.115. Determinati matricele X,Y, Z ∈ M2(Z), avand determinantul 1, stiindca X4 + Y 4 + Z4 = X2 + Y 2 + Z2 + 6I2.

Florin Stanescu, Gaesti

Clasa a XII-aXII.111. Fie x, y, z numere reale nenule. Daca x + y + z = 0 si x5 + y5 + z5 =

x7 + y7 + z7, calculati valoarea expresiei A = x2 + y2 + z2.Mihai Craciun, Pascani

XII.112. Calculati limita sirului (an)n∈N definit prin an =

Z 1

0

pxn + xn+2dx,

∀n ∈ N.Bogdan Victor Grigoriu, Falticeni

XII.113. Calculati limx→∞

Z 1x

1x+1

ctg t2dt.

Silviu Boga, Iasi

XII.114. Se considera functia continua f : [0, 1] → (0, 1]. Demonstrati ca pentru

orice n ∈ N∗, exista un unic xn ∈ (0, 1) astfel ıncat

Z xn

0f(t)dt = n

Z 1

xn

f(t)dt.

Calculati limita sirului (xn) si aratati ca limn→∞

n(1− xn) =1

f(1)

Z 1

0f(t)dt.

Florin Stanescu, GaestiXII.115. Fie F multimea functiilor de doua ori derivabile pe [a, b] cu derivata

de ordin doi continua, pentru care f(a) = α si f(b) = β, unde α si β sunt constante

fixate. Notam I(f) =

Z b

a[f ′(x)]2dx, ∀f ∈ F si J(f) =

Z b

a[f ′(x)(1 + 2f(x))]2dx,

∀f ∈ F . Determinati minI(f)|f ∈ F si minJ(f)|f ∈ F.Adrian Corduneanu, Iasi

180

Probleme pentru pregatirea concursurilor

A. Nivel gimnazial

G186. Fie m,n, x, y numere naturale nenule astfel ıncat m < n,m+ n

(m,n)este

numar par, iar xm = yn. Demonstrati ca x− y nu este patrat perfect.Geanina Havarneanu, Iasi

G187. Consideram numerele naturale a, b, c, d si x, astfel ıncat ac, bd si ad + bcsunt divizibile cu x. Aratati ca bc se divide cu x.

Ciprian Baghiu, Iasi

G188. Determinati tripletele (p, q, r) ∈ Z3 pentru care

[apq + bpq + (ap + bp)q]r = r[apqr + bpqr + (ap + bp)qr], ∀a, b ∈ R.

Temistocle Bırsan, Iasi

G189. Se considera trei sfere avand volumele v1, v2, v3, ariile s1, s2, s3 si lungimilecercurilor ecuatoriale l1, l2, l3. Demonstrati ca

v1l1 + l2

+v2

l2 + l3+

v3l3 + l1

≥ 1

12π(s1 + s2 + s3).

D.M. Batinetu-Giurgiu, Bucuresti

G190. Daca x si y sunt numere irationale astfel ıncat x2 + y, y2 + x si x+ y suntrationale, demonstrati ca xy < 1

4 .Neculai Moraru, Suceava si Silviu Boga, Iasi

G191. Fie A = 1, 2, . . . , n, unde n ∈ N, n ≥ 5. Demonstrati ca A poate fi scrisaca reuniunea a doua multimi disjuncte B si C, astfel ıncat produsul elementelor luiB sa fie egal cu suma elementelor lui C.

Gheorghe Iurea, Iasi

G192. Demonstrati ca ın orice triunghi neisoscel ABC, exista M,N ∈ [BC],P,Q ∈ [AB] si R,S ∈ [AC] astfel ıncat segmentele [AU ], [BT ] si [CV ] sunt laturi aleunui triunghi ascutitunghic, oricare ar fi punctele U ∈ [MN ], V ∈ [PQ] si T ∈ [RS].

Marius Dragan, Bucuresti

G193. Folosind efectiv 24 bete de chibrit, construiti un poligon avand aria de 7unitati, unde unitatea de arie este patratul cu latura de un bat de chibrit. (In legaturacu o problema din ”Amuzamente matematice” de Martin Gardner)

Dumitru Mihalache, Barlad

G194. Fie ABCD un patrulater convex, iar semidreptele [CK si [CL sunt bi-

sectoarele unghiurilor ÕACD, respectiv ÕACB (K ∈ AD,L ∈ AB). Demonstrati cadreapta LK trece prin centrul cercului ınscris ın triunghiul ABD daca si numai dacapatrulaterul ABCD este inscriptibil.

Neculai Roman, Mircesti (Iasi)

181

G195. PuncteleM si P apartin laturii neparalele (AD) a trapezuluiABCD, astfelıncat M ∈ [AP ]. Paralelele prin M si P la baze intersecteaza BC ın N , respectiv Q.Demonstrati ca trapezele ABNM si PQCD au diagonalele respectiv paralele daca sinumai daca CD si PQ sunt direct proportionale cu MN si AB (In legatura cu G105din RecMat 1/2006.)

Claudiu Stefan Popa, Iasi

B. Nivel licealL186. Determinati masurile unghiurilor triunghiului ABC, stiind ca

tgA+ 2ptgB + 3 3

ptgC = 6 6

ptgA+ tgB + tgC.

Catalin Calistru, Iasi

L187. Se considera triunghiul ABC si punctele D,L ∈ (BC), E,M ∈ (CA)si F,N ∈ (AB). Notam P = FE ∩ AL, Q = DF ∩ BM, S = ED ∩ CN .Demonstrati ca daca doua dintre urmatoarele afirmatii sunt adevarate, atunci esteadevarata si a treia:

i) dreptele AD,BE si CF sunt concurente;ii) dreptele AL,BM si CN sunt concurente;iii) dreptele DP,EQ si FS sunt concurente.

Titu Zvonaru, Comanesti

L188. Fie Oxy un reper cartezian, dreapta d : x = 1 si un punct M ∈ d.Punctul P se obtine din M printr-o rotatie de centru O si unghi 2α, unde α este

unghiul orientat ÕyOM . Scrieti ecuatia y = f(x) a curbei descrise de P atunci candM parcurge dreapta d si desenati graficul acestei curbe.

Temistocle Bırsan, Iasi

L189. Un sistem de n ≥ 3 puncte coplanare A1, A2, . . . , An, oricare trei necol-iniare, se numeste bun atunci cand oricare ar fi 1 ≤ i < j < k ≤ n si oricare arfi punctele M ∈ [AiAj ], N ∈ [AjAk] si P ∈ [AkAi], are loc inegalitatea MN2 +NP 2 +MP 2 ≤ AiA

2j +AjA

2k +AkA

2i . Determinati sistemele bune cu numar maxim

de puncte.Vlad Emanuel, student, Bucuresti

L190. Demonstrati ca

8√15 ≤ (

√x+

√x+ 48)(

√16− x+

√1− x) ≤ 36, ∀x ∈ [0, 1].

Gheorghe Iurea, Iasi

L191. Aratati ca, pentru orice numere reale pozitive x, y, z, are loc inegalitatea

(x2 + y2 + z2)

1

(x+ y)(x+ z)+

1

(x+ y)(y + z)+

1

(x+ z)(y + z)

≥ 9

4≥

≥ (xy + xz + yz)

1

(x+ y)(x+ z)+

1

(x+ y)(y + z)+

1

(x+ z)(y + z)

.

Marian Tetiva, Barlad

182

L192. Pe P multimea polinoamelor unitare avand coeficientii ın intervalul [0, 9].Determinati supremumul modulelor radacinilor complexe ale polinoamelor din P.

Adrian Reisner, Paris

L193. Fie A ∈ M3(R) o matrice simetrica, avand urma egala cu determinantul.Demonstrati ca 2 · Tr(An+2) ≥ Tr A · (Tr(An+1)− Tr(An−1)), ∀n ∈ N∗.

Paul Georgescu si Gabriel Popa, Iasi

L194. Fie p ≥ 2 un numar natural si ecuatia1

p√1 + x

+1

p√2 + x

+ . . .+1

p√n+ x

=

p√xp−1. Demonstrati ca, pentru fiecare n ∈ N∗, ecuatia are o unica solutie pozitiva

xn, aratati ca sirul (xn)n≥1 este monoton si calculati limn→∞

n

xn.

Gabriel Dospinescu, student, Paris

L195. Fie (an)n≥1 un sir de numere reale mai mari ca 1, astfel ıncat an+1 ≥2an − 1, ∀n ∈ N∗. Definim xn =

1

a1+

1

a1a2+ . . .+

1

a1a2 . . . an, ∀n ≥ 1.

a) Demonsrati ca sirul (xn)n≥1 este convergent (notam cu x limita sa).

b) Aratati ca sirul (αn)n≥1, unde x = xn +αn

a1a2 . . . an(an+1 − 1), ∀n ≥ 1, este

monoton, convergent si determinati limita sa.Dumitru Mihalache si Marian Tetiva, Barlad

Training problems for mathematical contests

A. Junior highschool level

G186. Let m,n, x, y be four nonzero natural numbers such that m < n,m+ n

(m,n)is an even number, and xm = yn. Prove that x− y is not a perfect square.

Geanina Havarneanu, Iasi

G187. Let us consider the natural numbers a, b, c, d and x such that ac, bd andad+ bc are divisible by x. Show that bc is divisible by x.

Ciprian Baghiu, Iasi

G188. Determine the triples (p, q, r) ∈ Z3 such that

[a p q + b p q + (a p + b p) q ]r = r [a p q r + b p q r + (a p + b p)q r], ∀a, b ∈ R.

Temistocle Bırsan, Iasi

G189. Three spheres are considered with their volumes v1, v2, v3, their areass1, s2, s3 and the lengths of their equatorial circles l1, l2, l3. Prove that

v1l1 + l2

+v2

l2 + l3+

v3l3 + l1

≥ 1

12π(s1 + s2 + s3).

D.M. Batinetu-Giurgiu, Bucuresti

183

G190. If x and y are irrational numbers such that x2 + y, y2 + x and x + y arerational, prove that xy < 1

4 .Neculai Moraru, Suceava and Silviu Boga, Iasi

G191. Let A = 1, 2, . . . , n with n ∈ N, n ≥ 5. Prove that the set A can bewritten as the union of two disjoint sets B and C so that the product of the elementsof B be equal to the sum of elements of C.

Gheorghe Iurea, Iasi

G192. Prove that, in any non-isosceles triangle ABC, six points M,N ∈ [BC],P,Q ∈ [AB] and R,S ∈ [AC] exist such that the segments [AU ], [BT ] and [CV ] arethe sides of an acute triangle, any would be the points U ∈ [MN ], V ∈ [PQ] andT ∈ [RS].

Marius Dragan, Bucuresti

G193. Effectively using 24 matches, build a polygon with its area equal to 7units, where a unit area is the area of a square with one match stick side. (Related toa problem from the book ”Mathematical Amusements” by Martin Gardner).

Dumitru Mihalache, Barlad

G194. Let ABCD be a convex quadrilateral, and let the half lines [CK and [CL

be the angle-bisectors of the angles ÕACD and respectively ÕACB (K ∈ AD,L ∈ AB).Prove that the line LK passes through the centre of the circle inscribed in the triangleABD if and only if the quadrilateral ABCD can be inscribed into a circle.

Neculai Roman, Mircesti (Iasi)

G195. The points M and P lie on the non-parallel side (AD) of the trapezoidABCD, such that M ∈ [AP ]. The parallel lines to the two bases intersect BC atN and at Q, respectively. Prove that the trapezia ABNM and PQCD have theirdiagonals respectively parallel if and only if CD and PQ are directly proportional toMN and AB (Connected with problem G105 of RecMat 1/2006 ).

Claudiu Stefan Popa, IasiB. Highschool Level

L186. Determine the measures of the angles of triangle ABC knowing that

tgA+ 2ptgB + 3 3

ptgC = 6 6

ptgA+ tgB + tgC.

Catalin Calistru, Iasi

L187. Let ABC be a triangle, and let the points D,L ∈ (BC), E,M ∈ (CA) andF,N ∈ (AB). Denote P = FE ∩ AL, Q = DF ∩ BM, S = ED ∩ CN . Provethat if two of the assertions below hold true the third one is also true:

(i) the straight lines AD,BE and CF are concurrent;(ii) the straight lines AL,BM and CN are concurrent;(iii) the straight lines DP,EQ and FS are concurrent.

Titu Zvonaru, Comanesti

L188. Let Oxy be a Cartesian system of coordinates and consider the line d :x = 1 with the point M ∈ d. The point P is obtained from M by a rotation of centre

O and angle 2α, where a is the directed angle ÕyOM . Write the equation y = f(x) of

184

the curve described by P when M runs along the line d and draw the graph of thiscurve.

Temistocle Bırsan, Iasi

L189. A system of n ≥ 3 coplanar points A1, A2, . . . , An, with any three amongthem being not collinear, is said to be good when for any 1 ≤ i < j < k ≤ n and thepointsM ∈ [AiAj ], N ∈ [AjAk] and P ∈ [AkAi], the inequalityMN2+NP 2+MP 2 ≤AiA

2j +AjA

2k +AkA

2i holds. Determine the good systems with a maximum number

of points.Vlad Emanuel, student, Bucuresti

L190. Prove that

8√15 ≤ (

√x+

√x+ 48)(

√16− x+

√1− x) ≤ 36, ∀x ∈ [0, 1].

Gheorghe Iurea, Iasi

L191. Show that any three positive real numbers x, y, z satisfy the inequality

(x2 + y2 + z2)

1

(x+ y)(x+ z)+

1

(x+ y)(y + z)+

1

(x+ z)(y + z)

≥ 9

4≥

≥ (xy + xz + yz)

1

(x+ y)(x+ z)+

1

(x+ y)(y + z)+

1

(x+ z)(y + z)

.

Marian Tetiva, Barlad

L192. Let P be the set of monic polynomials with their coefficients in the interval[0, 9]. Determine the supremum of the modules of the complex roots of polynomialsin P.

Adrian Reisner, Paris

L193. Let A ∈ M3(R) be a symmetric matrix with its trace equal to its deter-minant. Prove that 2 · Tr (An+2) ≥ Tr A · (Tr(An+1)− Tr(An−1)), ∀n ∈ N∗.

Paul Georgescu and Gabriel Popa, Iasi

L194. Let p ≥ 2 be a natural number and consider the equation1

p√1 + x

+

1p√2 + x

+ . . . +1

p√n+ x

=p√xp−1. Prove that, for any n ∈ N∗, this equation has

a unique positive solution xn, show that the sequence (xn)n≥1 is monotonic and

calculate the limit limn→∞

n

xn.

Gabriel Dospinescu, student, Paris

L195. Let (an)n≥1 be a sequence of real numbers that are greater than 1, such

that an+1 ≥ 2an− 1, ∀n ∈ N∗. We define xn =1

a1+

1

a1a2+ . . .+

1

a1a2 . . . an, ∀n ≥ 1.

a) Prove that the sequence (xn)n≥1 converges (and denote by x its limit).

b) Show that the sequence (an)n≥1, where x = xn+αn

a1a2 . . . an(an+1 − 1), ∀n ≥ 1,

is monotonic and convergent, and determine its limit.Dumitru Mihalache and Marian Tetiva, Barlad

185

Pagina rezolvitorilor

CARANSEBESLiceul Teoretic ”Traian Doda”. Clasa a IX-a (prof. DIDRAGA Iacob). MILU

Nicoleta: VII(110,111,113), VIII(113,114), IX(101,102).

CAMPULUNG MUSCELLiceul National cu program de atletism. Clasa a VII-a (prof. POPESCU

IONELA). MARTA Alexandra: V.116, VI(116-121), VII(116-118), VIII(116,117).

CRAIOVAColegiul National ”Fratii Buzesti”. Clasa a VI-a (prof. IONESCU Maria).

VIRLAN Leonard: P(181-183), V(109,112).

HARLAULiceul Teoretic ”Stefan cel Mare”. Clasa a VII-a (prof. SACALEANU Ioan).

NEICU Mara: VI(111-114), VII.110.

IASIScoala nr. 3 ”Al. Vlahuta”. Clasa a II-a (inst. MAXIM Gabriela). CARACAS

Sorin: P(184-187,193); DASCALU Lorena: P(184-187,193); POPESCU Alexandru:P(184-187,193); ROBU Carmen: P(184-187,193); SERBANOIU Alexandru: P(184-187,193). Clasa a III-a (inst. CRACIUNMarilena). POPESCU Claudia: P(184,185,187,193,195). Clasa a III-a (ınv. MARIUTA Valentina). ENEA Codrut-Alexandru:P(185-187,189,191,193); LUNGU Lucian: P(185-187,189,191,193); HERGHILIGIU-HENEA Malina: P(185-187,189,191,193); HREAPCA Alin: P(185-187,189,191,193);HUHU Paula: P(185-187,189,191,193); PERES Cristi: P(185-187,189,191,193); PO-POVICI Teodor-Andrei: P(185-187,189,191,193); ROMILA Andreea-Maria: P(185-187,189,191,193). Clasa a V-a (prof. MARIN Mirela). ANDRIES Alexandra:P(192,193), V(116-118); CRETU Cristiana-Paula: P(192,193), V(116-118); IFTIMEIoana Evelina: P(192,193), V(116-118); SOFIAN Marina Elena: P(192,193), V(116-118). Clasa a VII-a (prof. MARIN Mirela). ASAVEI Alexandra: VII(116,117,121,122), VIII.117; CELMARE Raluca-Iuliana: VII(116,117,121,122), VIII.117; MARCUAnca: VII(116,117,121,122), VIII.117 ; TIBA Stefana-Alexandra: VII(116,117,121,122), VIII.117.

Scoala nr. 11 ”Otilia Cazimir”. Clasa a III-a (inst. PARAIALA Dumitru).POPA Ioana-Maria: P(174-183), V(114,115).

Scoala nr. 13 ”Alexandru cel Bun”. Clasa a II-a (prof. COJOCARIU Ana).ACATRINEI Andra: P(184-188); BEJANMatei: P(184-188); BONCU Tudor: P(184-188); BULEI Iasmina-Ioana: P(184-188); COSTIN Mihaita-Alexandru: P(184-188);MUNTIANU Ioana-Andreea: P(184-188); PERDUN Patricia-Maria: P(184-188);PRISECARU Alexandru: P(184-188); SAMSON Constantin-Catalin: P(184-188);STRATILA Diana-Liliana: P(184-188); STEFAN Tudor: P(184-188); ZAHARIAStefan-Eusebiu: P(184-188).

Scoala nr. 26 ”George Cosbuc”. Clasa I (inst. RACU Maria). CIOPEICASebastian-Andrei: P(174,175,184-186); GROSU Victor-Alessandru: P(174,175,184-186); LUCHIAN Maria-Clara: P(174,175,184-186); PASNICU Cosmin-Constantin:

186

P(174,175,184-186); RAILEANU Razvan-Constantin: P(174,175,184-186); VASILERaluca-Andreea: P(174,175,184-186). Clasa a III-a (inst. VARLAN Elena). A-MARIEI Romeo: P(184-189,191); GHEBAN Andreea: P(184-189,191); PAVALUCAna-Maria: P(184-189,191); PICHIU Cosmin-Andrei: P(184-189,191); TATARU Al-ice: P(184-189,191); TOFAN Ioana-Raluca: P(184-189,191); TIPLEA Iulian: P(184-189,191).

Colegiul ”National”. Clasa a V-a (prof. POPA Gabriel). ANDONE Raluca:P(192-194), V(116-122), VI(120,121); DOMINTE Stefan: V(109-115), VI(109-112).Clasa a VII-a (prof. POPA Gabriel). STOLERU Ingrid: VI(118-121), VII(116,117,119,121), VIII.117, G179. Clasa a X-a (prof. POPA Gabriel). CEUCA Razvan:IX(101,102), X(101,102), G(169,171); MOCANU Dan: IX(101-103,105), X(101-103),G(169,171).

NEGRESTI (VASLUI)Scoala ”M. David”. Clasa a III-a (inst. SIMIUC Petronela). AIPATIOAEI

Ionel: P(184-188); CIOATA Andreia: P(184-188).

PASCANIScoala ”Iordache Cantacuzino”. Clasa a III-a (ınv. MIRON Petru). CRACIUN

Stefana-Maria: P(174-179).

TIGANASI (IASI)Scoala cu cls. I-VIII ”M. Kogalniceanu”. Clasa a III-a (ınv. PATRASCU Car-

men). CAZADOI Ioana Cristina: P(186,188,190,192,193); DUCA Cristina-Mihaela:P(184-188); SANDU Rebeca: P(184,186,188-190). Clasa a V-a (prof. CHIRITESCUAnca). DUCA Mirela Beatrice: P(186,192,193), V(116,120); GANEANU Tudor:P(192,193), V(116,118,120); VERNERMadalina: P(192,193), V(116,118,120), VII.116.Clasa a VI-a (prof. CHIRITESCU Anca). BOROS Paula Mihaela: P192, V(116,118,120), VII.116, DUCA Liliana Daniela: P(192-194), V(116-118,120), VII.116; IACOBOana Alisa: P(192,193), V(116,120); VII.116; PIU Debora Roxana: P(192,193),V(116,118,120), VII.116; PIUMioara Florentina: P(192,193), V(116,118,120), VII.116;Clasa a VII-a (prof. CHIRITESCU Anca). GAVRILAS Marius Alexandru: V(116-118,120), VII.116; GANEANU Stefan: V(116-118,120), VII.116; GHIOANCA Oana:V(116-118,120), VII.116.

Elevi rezolvitori premiati

Scoala nr. 3 ”Al. Vlahuta”, Iasi

1. DASCALU Lorena (cl. a II-a): 2/2009(5pb), 1/2010(5pb), 2/2010(5pb).

2. POPESCU Alexandru (cl. a II-a): 2/2009(5pb), 1/2010(5pb), 2/2010(5pb).

3. ROBU Carmen (cl. a II-a): 2/2009(5pb), 1/2010(5pb), 2/2010(5pb).

4. SERBANOIU Alexandru (cl. a II-a): 2/2009(5pb), 1/2010(5pb), 2/2010(5pb).

187

5. POPESCU Claudia (cl. a III-a): 2/2009(7pb), 1/2010(5pb), 2/2010(5pb).

6. ENEA Codrut-Alexandru (cl. a III-a): 2/2009(7pb), 1/2010(5pb), 2/2010(6pb).

7. MARCU Anca (cl. a VII-a): 2/2009(5pb), 1/2010(6pb), 2/2010(5pb).

8. TIBA Stefana-Alexandra (cl. a VII-a): 2/2009(5pb), 1/2010(6pb), 2/2010(5pb).

Scoala nr. 13 ”Alexandru cel Bun”, Iasi

1. ACATRINEI Andra (cl. a II-a): 2/2009(5pb), 1/2010(5pb), 2/2010(5pb).

2. PERDUN Patricia-Maria (cl. a II-a): 2/2009(5pb), 1/2010(5pb), 2/2010(5pb).

3. PRISECARU Alexandru (cl. a II-a): 2/2009(5pb), 1/2010(5pb), 2/2010(5pb).

4. SAMSON Constantin-Catalin (cl. a II-a): 2/2009(5pb), 1/2010(5pb), 2/2010(5pb).

5. STEFAN Tudor (cl. a II-a): 2/2009(5pb), 1/2010(5pb), 2/2010(5pb).

6. ZAHARIA Stefan-Eusebiu (cl. a II-a): 2/2009(5pb), 1/2010(5pb), 2/2010(5pb).

Scoala nr. 26 ”George Cosbuc”, Iasi

1. AMARIEI Romeo (cl. a III-a): 2/2009(8pb), 1/2010(6pb), 2/2010(7pb).

2. GHEBAN Andreea (cl. a III-a): 2/2009(8pb), 1/2010(6pb), 2/2010(7pb).

3. PICHIU Cosmin-Andrei (cl. a III-a): 2/2009(8pb), 1/2010(6pb), 2/2010(7pb).

4. TATARU Alice (cl. a III-a): 2/2009(6pb), 1/2010(6pb), 2/2010(7pb).

5. TIPLEA Iulian (cl. a III-a): 2/2009(8pb), 1/2010(6pb), 2/2010(7pb).

Scoala ”M. Kogalniceanu”, TIGANASI

1. CAZADOI Ioana-Cristina (cl. a III-a): 2/2009(7pb), 1/2010(5pb), 2/2010(5pb).

2. DUCA Cristina-Mihaela (cl. a III-a): 2/2009(7pb), 1/2010(5pb), 2/2010(5pb).

3. SANDU Rebeca (cl. a III-a): 2/2009(7pb), 1/2010(5pb), 2/2010(5pb).

Colegiul National ”Fratii Buzesti”, Craiova

1. VIRLAN Leonard (cl. a VI-a): 2/2009(5pb), 1/2010(6pb), 2/2010(5pb).

188

Revista semestrială RECREAŢII MATEMATICE este editată de ASOCIAŢIA “RECREAŢII MATEMATICE”. Apare la datele de 1 martie şi 1 septembrie şi se adresează elevilor, profesorilor, studenţilor şi tuturor celor pasionaţi de matematica elementară.

În atenţia tuturor colaboratorilor Materialele trimise redacţiei spre publicare (note şi articole, chestiuni de

metodică, probleme propuse etc.) trebuie prezentate îngrijit, clar şi concis; ele trebuie să prezinte interes pentru un cerc cât mai larg de cititori. Se recomandă ca textele să nu depăşească patru pagini. Evident, ele trebuie să fie originale şi să nu fi apărut sau să fi fost trimise spre publicare altor reviste. Rugăm ca mate-rialele tehnoredactate să fie însoţite de fişierele lor.

Problemele destinate rubricilor: Probleme propuse şi Probleme pentru pregătirea concursurilor vor fi redactate pe foi separate cu enunţ şi demonstra-ţie/rezolvare (câte una pe fiecare foaie) şi vor fi însoţite de numele autorului, şcoa-la şi localitatea unde lucrează/învaţă.

Redacţia va decide asupra oportunităţii publicării materialelor primite. În atenţia elevilor Numele elevilor ce vor trimite redacţiei soluţii corecte la problemele din

rubricile de Probleme propuse şi Probleme pentru pregatirea concursurilor vor fi menţionate în Pagina rezolvitorilor. Se va ţine seama de regulile:

1. Pot trimite soluţii la minimum cinci probleme propuse în numărul prezent şi cel anterior al revistei; pe o foaie va fi redactată soluţia unei singure probleme.

2. Elevii din clasele VI-XII au dreptul să trimită soluţii la problemele propuse pentru clasa lor, pentru orice clasă mai mare, din două clase mai mici şi imediat anterioare. Elevii din clasa a V-a pot trimite soluţii la problemele propuse pentru clasele a IV-a, a V-a şi orice clasă mai mare, iar elevii claselor I-IV pot trimite soluţii la problemele propuse pentru oricare din clasele primare şi orice cla-să mai mare. Orice elev poate trimite soluţii la problemele de concurs (tip G şi L).

3. Vor fi menţionate următoarele date personale: numele şi prenumele, clasa, şcoala şi localitatea.

4. Plicul cu probleme rezolvate se va trimite prin poştă (sau va fi adus direct) la adresa Redacţiei:

Prof. dr. Temistocle Bîrsan Str. Aurora, nr. 3, sc. D, ap. 6, 700 474, Iaşi Jud. IAŞI E-mail: [email protected]

CUPRINS Universitatea ieşeană – la aniversare (C. CORDUNEANU)................................................. 93 Centenarul Societăţii de Ştiinţe Matematice din România ................................................. 96

ARTICOLE ŞI NOTE T. BÎRSAN – Strofoida – câteva proprietăţi elementare..................................................... 97 I. PĂTRAŞCU – O generalizare a teoremei lui Coşniţă .................................................. 102 V. BLENDEA, Gh. BLENDEA – Criteriu pentru calculul unor limite de funcţii ...... 104 D. POPESCU – Asupra unor puncte de concurenţă ale unui triunghi........................... 107

NOTA ELEVULUI R. CEUCĂ – Câteva proprietăţi caracteristice ale triunghiului isoscel ............................ 110

CORESPONDENŢE A. REISNER – Valeurs et vecteurs co-propres d'une application semi-linéaire ............ 113

CHESTIUNI METODICE Gh. IUREA - Proprietăţi ale primitivelor funcţiilor periodice ………………………... 118

CUM CONCEPEM... CUM REZOLVĂM

M. TETIVA – O problemă de reprezentare....................................................................... 123

DIN ISTORIA MATEMATICII T. BÎRSAN, D. TIBA – Un episod inedit din matematica românească interbelică...........128

ŞCOLI ŞI DASCĂLI SEBASTIAN ANIŢA (1941-2010) .................................................................................... 132

CONCURSURI ŞI EXAMENE Concursul de matematică "Al. Myller", ed. a VIII-a, 2010 ............................................. 134 Concursul de matematică "Florica T. Câmpan", ed. a X-a, 2010.................................... 137 Concursul de matematică "Student pentru o zi", ed. a III-a, Suceava, 2010 .................143

PROBLEME ŞI SOLUŢII Soluţiile problemelor propuse în nr. 2/2009........................................................................ 145 Soluţiile problemelor pentru pregătirea concursurilor din nr. 2/2009 ............................... 163 Probleme propuse................................................................................................................... 175 Probleme pentru pregătirea concursurilor ............................................................................ 181 Training problems for mathematical contests ..................................................................... 183 Pagina rezolvitorilor ........................................................................................................... 186 Elevi rezolvitori premiaţi ……………………………………………………...………....187 ISSN 1582 – 1765 8 lei